You are on page 1of 185

..

Anatom ic Sciences
-

~:'\M1id1 of the following represents the secretory


6. Desmosomes function by serving as
."
. product of the parafollicular cells of the thyroid -
A a means for communication between
gland?
cells.
A Iodine B. a means for attachment of adjacent cells.
. 8. Protease C. an outlet for secretory products of a cell.
C. Thyroxine D. a bridge for continuity of cytoplasm
D. Calcitonin between adjacent cells.
E. Thyroglobulin E. a means by which tonofibrils can pass
from one cell into another.

)2.'.'. Parasympathetic innervation controlling


. salivation originates with which of the following
aanial nerves?
7. The growth rate in the cartilage of the epiphyseal
A V, VII, IX,X, andXII plate of a long bone is markedly retarded when
B. V, VII, IX, andX there is a lack of hormone from which of the
C. V, IX, and X following?
D. VII, IX,andXI
E. VII and IX A Hypophysis
B. Adrenals
C. Testes
Peripheral (lower motor neuron) lesions of D. Parathyroids
aanial nerve VII will cause which of the E. Islets of Langerhans
following?

A A contralateral paralysis of the facial


musculature
B. An ipsilateral flaccid paralysis of the facial
musculature 8. The dentist incises the mucous membrane of
C. the floor of the mouth. This incision extends
A contralateral paralysis of only the lower
facial muscles from the molar region to the sublingual caruncle
D. A contralateral paralysis of only the upper (papilla). Which of the following structural
facial muscles groups will be exposed first?
E. An ipsilateral flaccid paralysis of only the
lower facial muscles A Sublingual gland, lingual nerves, and
submandibular duct
B. Hyoglossus and mylohyoid muscles, and
4. The temporalis muscle inserts into the hypoglossal nerve .
c. Lingual nerve, lingual artery, and anterior
A coronoid process. belly of the digastric muscle
B. condylar process. D. Lingual nerve, hypoglossal nerve, and
C. fovea of the mandible. submental branch of the facial artery
D. lateral aspect of the mandibular angle. E. Anterior belly of the digastric muscle,
E. articular disk of the temporomandibular mylohyoid nerve, and submental branch of
joint. the facial artery

5. Which of the following is surrounded partly by


connective. tissue and epithelium, contains
lymphoid follicles, has no sinuses, and is 9. The trigeminal nerve innervates exclusively
penetrated by a number of crypts? which of the following muscles or muscle parts?
A Spleen A Buccinator and masseter
B. Thymus B. Mylohyoid and geniohyoid
C. Lymph node C. Medial and lateral pterygoid
D. Palatine tonsil D. Tensor and levator veli palatini
E. Bursa of Fabricius E. Anterior and posterior bellies of digastric
.r.

3
\
.f
.

10. As the subclavian vein crosses the first rib, it 15. Which of the following nerves is the MOS'
lies likelyto become injured in fractures of th
mid-humeralshaft? .

A anterior to the anterior scalene muscle.


B. posterior to the anterior scalene A Ulnar
muscle. B. Radial
C. posterior to the posterior scalene C. Median
muscle. D. Axillary
D. between the anterior and the posterior E. Musculocutaneous
scalene muscles.
E. between the scalene posterior and the
levator scapulae muscles. 16. Which of the followingtypes of epithelium
well adapted for secretory or absorptive
functions?
11. Which of the following BEST describes the
passage of material through the hepatic A Simple columnar
sinusoids? B. Simple squamous
C. Stratified cuboidal
A Blood passes toward the central vein. D. Stratified squamous
B. Bile passes peripherally toward the. E. Psuedostratified columnar
portal canal.
C. Lymph moves centrally to join the
sublobular duct. 17.> It is possible to distinguish histologically \;1
D. Lymph moves peripherally toward the between the stomach and the duodenum ~

space of Disse. because of the presence of


E. Blood passes peripherally away from.
the central vein. A mucosal glands in the stomach only..
B. submucosal glands in the duodenum..~
only.'
12. In the life cycle of an ameloblast, there are C.' simple columnqr epithelium lining the'
- cells that contain Tomes' processes. These stomach only. .
cells are in which of the followingstages? D. muscularis mucosa in the stomach ..

only.
A Secretory E. smooth muscle in the external
B. Morphogenic musculature of the duodenum only.
C. Organizing
D. Maturative
E. Protective
18. The superior and inferior ophthalmic veins
drain directly or indirectly into the
13. Histologically,the osteoclasts of bone
resorption are typically A frontal vein.
B. cavernous sinus.
A anuclear. C. anterior facial vein.
B. mononuclear. D. internal jugular vein.
C. multinuclear. E. superior petrosal sinus.
D. polymorphonuclear.

14. During an intraoral injection to the mandibular 19. Each of the followingdevelops as an
foramen, the needle passes through the outpocketing of the gut tube EXCEPT one.
mucous membrane and the buccinator Which one is this EXCEPTION?
muscle. As it does so, the needle lies
A Lung
A inferior to the mandibular lingula. B. Liver
B. C. Spleen
superior to the auriculotemporal nerve.
C. lateral to the neck of the mandible. D. Pancreas
D. lateral to the medial pterygoid muscle. E. Gallbladder
E. lateral to the stylomandibular ligament.
4
.

24. Which of the following processes can be seen -


in the photomicrograph below?

of the following nerves contributes to


"
motor innervation of intrinsic muscles of
tongue?
.,
:.

"
it
,0
:1

.. .,'

A Osteoclasia
B. Endochondral bone formation
C. Intramembranous bone formation !i
.,1
D. Secondary center of ossification

Each of the following muscles receives motor


innervation from the ansacervicalis EXCEPT :1
"~
. one. \Nhich one is this EXCEPTION?
.
~

A Omohyoid 25. Which of the following structures leaves an .


~

B. Thyrohyoid impression on the right lung?


C. Geniohyoid
O. Sternohyoid I
A Azygos vein
E. Sternothyroid B. Right vagus nerve J

C. Right phrenic nerve !


D. Descending thoracic aorta ,
~

E. Right common carotid artery

The three divisions of the trigeminal nerve 26. Which of the following represents the main
pass through openings in which of the support of the wall of a bronchus?
following bones?
A Smooth muscle
Frontal
B. Hyaline cartilage
Sphenoid C. Elastic membranes
Temporal D. Dense irregular connective tissue
Parietal
E. Elastic and collagenous connective
Occipital tissue

5
. .
.

27. The diploid number of chromosomes is 32. Each ofthe followingstructures is an opening
maintained in proliferating somatic cells by into the pterygopalatine fossa EXCEPT one.
which of the followingprocesses? Which one is this EXCEPT/OfV?

A Meiosis A Facial canal


B. Mitosis B. Pterygoid canal
C. Budding C. Pharyngeal canal
D. Amitosis D. Sphenopalatine foramen
E. Cytokinesis E. Pterygomaxillary fissure

28. Tendons are comprised of which of the


followingtypes of collagenous connective 33. The hepatic veins drain blood from the liver
tissue? into the

A Areolar A inferiorvena cava.


B. Reticular B. superior vena cava.
C. Dense regular C. azygos vein.
- D. Dense irregular D. portal vein.
E. superior mesenteric vein.

.29. -which of the followingnerves innervates the


trapezius muscle? . .
34. Calcium ions are sequestered by
A Radial
B. Axillary A T-tubules.
C. B. sarcoplasm.
Accessory .

D. Thotacodorsal C. sarcosomes.
-- E. long thoracic D. myofibrils.
E. sarcoplasmic reticulum.

-
30. The lateral pterygoid muscle insert~ into
which of the following?
35. Which of the followingtwo muscles attach to
A Condylar process only the pterygomandibular raphe?
B. Medialaspect of the mandibularramus
;.~
C. Articular disk of the temporomandibular A Buccinator and palatopharyngeus
joint only B. Medial pterygoid and palatopharyngeus
D. Articular disk of the temporomandibular C. Levator veli palatini and
joint and neck of the mandible palatopharyngeus
E. Articular disk of the temporomandibular D. Buccinator and superior pharyngeal
- joint and coronoid process constrictor

31. Ifthe facial nerve were to be cut just after it 36. Which of the following features distinguishes
exited the stylomastoid foramen, it would bone from osteoid?
cause loss of innervation to which of the
following? A Bone is not resorbed.
B. Bone has a mineralized matrix.
A lacrimal gland C. Osteoid contains fewer lacunae per unit
B. Mylohyoidmuscle area.
C. Submandibular gland D. Osteoid contains a different type of fiber
D. Orbicularis oculi muscle in its matrix.
E. Anterior belly of the digastric muscle
6
-....
,.,.

37. Which of the following explains why the Barr 42. Some medications can be absorbed through
body found in certain epithelial cells is the mucosa of the tongue's ventral surface
significant? and through the mucosa of the floor of the
mouth. This absorption can take place in
A these areas because the mucosa is
It suggests mitotic activity.
B. It indicates protein synthesis.
C. It indicates a metaplasmic change. A covered by pseudostratified squamous
D. It is a symptom of nuclear epithelium.
disintegration. .
B. covered by simple squamous
E. It assists in differentiating between the epithelium with a vascular lamina
sexes. propria.
c. covered by thin nonkeratinized stratified
squamous epithelium with a thin lamina
propria.
D. covered by keratinized stratified
38. squamous epithelium that contains
The vagus nerve supplies parasympathetic
fibers to which of the following? numerous capillaries.
E. pierced by the ducts of numerous minor
salivary glands through which some
A Descending colon
types of medication are easily
B. Ascending colon absorbed.
C. Sigmoid colon
D. Rectum I
E. Anus

43. Pain fibers from the posterior third of the J


tongue travel with which cranial nerve?
39. The lymph vessels that drain both dental A
arches connect directly with which of the
Vagus
B. Facial
following nodes? C. . Trigeminal
D. Glossopharyngeal
A Submandibular
E. Hypoglossal
B. Deep cervical
C. Sublingual I
D. Retropharyngeal I
E. Superficial cervical I
44. A patient who has damage to the right
hypoglossal nerve will protrude the tongue
I

40. Which of the following strata of the epidermis


A upward.
is the LEAST cytodifferentiated? B. downward.
C. straight forward.
A Basale D. toward the right side.
B. Corneum E. toward the left side.
C. Spinosum I
D. Granulosum ~
~
",
45. In terms of wall thickness, the relative
.
J:

proportion of smooth muscle is GREATEST in


41. The roots of the brachial plexus are derived
which of the following?
from the ventral rami of
A Veins
A the cervical plexus. B. Venules
B. the spinal accessory nerve. C. Arterioles
C. spinal nerves C3 through C5 D. Capillaries
D. spinal nerves C3 through C7. E. Large arteries
E. spinal nerves C5 through T1.
..
.

46. The superior laryngeal artery pierces the 51. The buccinator muscle is supplied by which of
thyrohyoid membrane in company with which the following nerves?
of the following laryngeal nerves?
A Facial
A Inferior laryngeal B. Lingual
B. Internal laryngeal c. Spinal acessory
C. External laryngeal D. Inferior alveolar
D. Superior laryngeal E. Buccal nerve of the trigeminal
E. Recurrent laryngeal

52. In which of the following are numerous


47. The vertebral artery is a branch of the afferent lymphatic channels found?

A brachiocephalic artery. A Spleen


- B. external carotid artery. R Thymus
C. internal carotid artery. C. Lymph nodes
D. subclavian artery. D. Palatine tonsils
E. thyrocervical trunk. E. Pharyngeal tonsils

-'
48. The muscle that is the prime mover in left 53. The submucosa is present in each of the
lateral excursion is the foHowing EXCEPT one. Which one is this
EXCEPTION?
-
A right masseter.
B. left medial pterygoid. A Colon
C. right medial pterygoid. B. Stomach
- D. left lateral pterygoid. C. Jejunum
E. right lateral pterygoid. D. Gallbladder
E. Duodenum

49. Each of the following structures lies between


the hyoglossus and the mylohyoid muscles" 54. Each of the following structures is bound by a
EXCEPT one. Which one is this EXCEPTION? membrane EXCEPT one. Which one is this
-
EXCEPTION?
A Lingual nerve
B. Lingual artery A Nucleolus
C. Sublingual gland B. Lysos~me
D. Submandibular duct C. Nucleus
- E. Hypoglossal nerve D. Mitochondrion
E. Pinocytotic vesicle

50. Which of the following represents the function 55. The hypophysis is situated in a fossa of which
of the interstitial cells (of Leydig) in the testis? of the following bones?
-
A Form the acrosome A Ethmoid
B. Secrete testosterone B. Frontal
C. Inhibit spermatogenesis C. Temporal
D. Supply nutrients to sex cells D. Sphenoid
E. Support the germinal epithelium E. Palatine

8
,

56. White matter of the spinal cord consists 61. The core of a ciliumis composed of
chiefly of which of the following?
A microvilli.
A Perineurium B. microtubules. .'
B. Myelinated axons C. microfibrils.
C. Unmyelinated axons D. tonofilaments.
D. Nerve cell bodies E. microfilaments.
E. Loose connective tissue

57. The thoracic duct empties directly into the


junction of the 62. Each of the followingenters the orbit by way of
the superior orbital fissure EXCEPT one.
A superior vena cava and the azygos vein. Whjch one is this EXCEPTION?
B. left jugular and brachiocephalic veins.
C. right jugular and brachiocephalic veins. A Abducens nerve
D. left internaljugularand subclavian B. Trochlear nerve
veins. C. Oculomotor nerve
E. right internal jugular and subclavian D. Ophthalmic artery
veins. E. Ophthalmic division of the trigeminal
nerve

58. Pain, touch, temperature and proprioceptive


modalities for the temporomandibular joint
. are carried by way of which of the following
nerves?
63. The capacity of the tongue'for forceful
A Auriculotemporal movement depends on striated muscle
B. Auditory supplied by which of the following cranial
nerves?
C. Temporal
D. Superior alveolar
E. Inferioralveolar A V
B. \111
C. DC
D. X
E. XJI
59. The left recurrent laryngeal nerve is closely
related at its beginning to the

A left subclavian artery.


B. arch of the aorta and the left pulmonary 64. How many lobes does the right lung usually
veins. have?
C. ligamentum arteriosum and the arch of
the aorta. A 1
D. pulmonary trunk and the left B. 2
brachiocephalic vein. C. 3
E. ligamentum arteriosum and the left D. 4
brachiocephalic vein. E. 5

60. Which of the followingcranial nerves contain 65. Enlargement of the third ventricle and both
parasympathetic preganglionic fibers? lateral ventricles is caused by obstruction of
the
A II, III, IV, and V
B. III, IV, V, and VI A cerebral aqueduct.
C. III, V, V~~,and IX B. foramen of Magendie.
D. III, VII, IX, and X C. foramina of Luschka.
E. VII,IX, X, and XI D. interventricular foramina of Monro.
.,
,

66. Melanocytes are derived from which of the 70. Highlyskilled, discrete motor activity of the
following? hand is dependent on which of the following
cortical areas of the hemisphere?
A Ectoderm
B. Endoderm A
C. Mesoderm Transverse temporal gyrus
B. Angular gyrus of the parietal lobe
D. Dermatomes C.
E. Precentral gyrus of the frontal lobe
Neural crest D. Paracentral lobule on the medial
surface

67. When both dentitions of the human are 71. During swallowing, muscular contraction
considered, which of the followingrepresents, results in movements that seal off the
oropharynx from the nasopharynx. Which of
in years, the life span of the dental lam~~ the following muscles cause movements that
A result in a fold in the posterior wall of the
B.. 31 4' c:>C.o~ pharynx?

-
C.
D. 9 ~~
5 ~ 0 ..'. A Levator veli palatini
E. 12\-t~ B. Tensor veli palatini
C. Palatopharyngeus
D. Musculus Uvulae
- E. Palatoglossus.

68. Cementum differs from dentin in that


---- cementum 72. From which of the following is the periodontal
ligament derived?
A contains more inorganic material than
- dentin. A Dental sac
8. is not formed followingeruption of the 8. Enamel organ
tooth. C. Dental papilla
C. can contain cells, whereas dentin D. Epithelial root sheath
- contains cells as well as cell E. Outer enamel epithelium
processes.
D. is produced by cells of the periodontal
ligament, but dentin is produced by pulp 73.
- cells.
Compared with intertubular dentin, peritubular
dentin is characterized by which of the
E. contains some elastic fibers, whereas following?
dentin contains only collagenous fibers.
, A Greater stainability
B. Higher quantity of fluids
C. Lesser content of inorganic salts
D. Greater content of inorganic salts
- E. Greater content of large collagen fibers
69. A dentist willuse screw-type implants to
replace the maxillary incisors (Teeth #s 7, 8,
9, and 10). If these implants pass through the 74. Which of the following represent fan-shaped,
bone in this region, then which of the following hypocalcified areas that originate at the
regions willthey be entering? dentinoenameljunctionand extend into
enamel for part of its thickness?
- A Orbit
B. Mental foramen A Tufts
C. 8. Spindles
Nasal cavity
D. C. Lamellae
Maxillarysinus
E. Pterygopalatine fossa D. Hunter-Schreger bands
E. Contour lines of Owen
10
ji
!
I ,

75. The calcified bodies sometimes found in the 80. The lingula of the mandible serves as an
POL are BEST described as which of the attachment for which of the following?
following?
A Temporalis
A Cementicles 8. Stylomandibular ligament
B. Denticles C. Sphenomandibular ligament
C. Bone D. Temporomandibular ligament
D. Enamel pearls _. Tendon of the digastric muscle
E. Mineralized interstitial tissue

81. In the developing embryo the palate is


separated from the lip by a shallow sulcus in
76. Cell bodies of the somatic motor system lie in the depths of which two epithelial laminae
which of the following locations? -
arise. The outer lamina is the

A Dorsal horn A. dental lamina for primary teeth.


B. Ventral horn s. dental lamina for permanent teeth.
C. C. primordium of the parotid gland.
Autonomic ganglia
D. D. primordium of the buccal frenula.
Dorsal root ganglia -
E. Intermediolateral horn vestibular lamina.

82. 'I/hich of the following is derived from


ectomesenchyme7
77. Which of the following represents the location
of the cell bodies of pain fibers. in the A Ameloblasts
glossopharyngeal nerve? S. Odontoblasts
C. Stellate reticulum
A Otic ganglion D. Stratum intermedium
B. Nucleus ambiguus - Reduced enamel epithelium
C.- Trigeminal-. (semilunar) ganglion
D. Spinal nucleu~ of cranial nerve V
E. Superior ganglion of cranial nerve IX 83. In a newly erupted tooth, the junction between
teoth surface and the crevicular epithelium
c:Jnsists of which of the following?

78. Which of the following represents the cranial , Interstitial crevicular fluid
1"'1.
nerve that supplies derivatives of the third S. Basal lamina-like structure between
branchial arch? enamel and epithelium
c. Basal lamina-like structure between
A Facial cementum and epithelium
B. Accessory D. Basal lamina-like structure between
C. . Trigeminal dentin and epithelium
D. Hypoglossal _. Keratin fibers, running from the
E. Glossopharyngeal epithelium deeply into the enamel

84. vVhichgroup of fibers of the periodontal


79. Optic nerve fibers from the nasal half of the
retina cross the midline and enter the optic
ligament is the first to offer resistance to
movement of the tooth in an occlusal
tract of the opposite side by way of the direction?
A optic chiasma. A Alveolar crest
B. lateral geniculate body. 8. Interradicular
C. bipolar cells of the retina. C. Horizontal
D. brachium of the superior colliculus. D. Oblique
E. geniculocalcarine tract (optic ~
c:. Apical
radiations).
--
.it".:
.~~;

.,
,
85. Which of the followingBEST characterizes the 89. Salivary gland striated ducts are composed of
alveolar mucous membrane? which of the followingtypes of epithelium?

A Has no melanocytes A Simple squamous


B. Firmly bound to underlying bone B. Simple cuboidal
C. Well developed epithelial ridges C. Stratified squamous
D. Separated from the gingiva by the free D. Simple low columnular
gingival groove . E. Psuedostratified ciliated columnar
E. Appears red due to high vascularity and
thinness of epithelium

90. Which of the following muscles insert(s) onto


the neck of the condyle?

A Masseter
B. Temporalis
86. As demonstrated by the pattern of sensory C. Lateral pterygoid
innervation, which of the followingbranchial D. Medial pterygoid
arches are concerned in development of the
tongue?

A First and second only 91. Odontoblasts are characterized by


B. First, second, and third
. C. Second and third only A being located on external surfaces of
D. Second, third, and fourth roots.
E. Third and fourth only B. being shed from the tooth at the time of
eruption.
C. differentiating first at the cervical region
of a forming tooth.
D. secreting a non-fibrous matrix
'composed of chains of amino acids.
E. possessing long cytoplasmic
processes which lie within dentinal
87. The apical cytoplasm of active serous tubules.
glandular cells is typically filledwith which of
the following?

A Large amount of DNA 92. Which of the following are pure serous
B. Abundance of ribosomes glands?
C. Abundance of mitochondria
D. Abundance of lipid droplets A Sublingual glands
E. Abundance of zymogen granules B. Glands of Brunner
C. Submandibular glands
D. Glands of van Ebner
E. Glands of Blandin-Nuhn
-

88. Each of the followingembryologic structures 93. Which of the following structures contacts
is derived from the first branchial arch posteriorly with the isthmus of the thyroid
EXCEPT one. Which one is this EXCEPT/ON? gland?

A Tuberculum impar A Larynx


B. Maxillaryprocess B. Pharynx
C. Mandibular process C. Trachea
D. Intermaxillary process D. Esophagus
E. Lateral lingual swelling E. Carotid sheath

12
I'

94. The infrahyoid muscles receive their motor 98. The crescents or demilunes of the mucous
innervation from which of the following? alveoli of the sublingual gland are composed
of which of the following cells?
A . Vagus nerve
B. Supraclavicular nerves A Mucous
C. Brachial plexus B. Serous
D. Pharyngeal plexus C. Neural
E. Branches of the cervical plexus D. Striated
E. Myoepithelial

95. Which of the following types of tissues can be


demonstrated on the posterior slope of the
articular eminence?

A Hyaline cartilage
B. Fibrocartilage
C. Fibrous connective tissue
D. Articular cartilage
E. Elastic cartilage 99. A fracture of the hamulus affects the action of
which of the following muscles?

A Superior constrictor of the pharynx


96. Dentinal tubules are S-shaped in the crown of
B. Levator veli palatini
the toothdue to the .
C. Tensor veli palatini
D. Salpingopharyngeus
E. Buccinator
A incremental pattern.
B. epithelial diphragm.
C. crowding of odontoblasts.
D. . formationof peritubulardentin.
E. calcification pattern of maturing dentin.

97. Hertwig's epithelial root sheath is derived


from which of the following?

A Inner dental epithelium and stellate 100. Cell bodies of proprioceptive fibers in V are
reticulum located in the
B. Inner dental epithelium and stratum
intermedium
A chief nucleus.
C. Outer dental epithelium and stellate B. spinal nucleus.
reticulum
C. semilunar ganglion.
D. Outer dental epithelium and stratum D. geniculate ganglion.
intermedium
E. mesencephalic nucleus.
E. Inner dental epithelium and outer dental
epithelium
.,
,
RELEASED
NATIONAL BOARD DENTAL EXAMINATIONS
PART 1

TEST: ANATOMIC SCIENCES


FORM: 11
DATE: D96

Item Item Key Item Key 'Item Key


Key

1 D 26 B 51 A 76 B
2 E 27 B 52 C 77 E
3 B 28 C 53 D 78 E
4 A 29 C 54 A 79 A
5 D 30 D 55 Do 80 C

6 B 31 . D 56 B 81 E
7 A 32 A 57 D 82 B
8 A 33 A 58 A 83 B
9 C 34 E 59 C 84 E
10 A 35 D 60 D 85 E

11 A 36 B 61 B 86 B
12 A 37 E 62 D 87 E
13 C 38 B 63 E 88 D
14 D 39 A 64 C 89 D
15 B 40 A 65 A 90 C

16 A 41 E 66 E 91 E
17 B 42 C 67 - 92 D
18 B 43 D 68 D 93 C
19 C 44 D 69 C 94 E
20 E 45 C 70 C 95 C

21 D 46 B 71 C 96 C
22 C 47 D 72 A 97 E
23 B 48 E 73 D 98 B
24 B 49 B 74 A 99 C
25 A 50 B 75 A 100 E

- indicates item not scored


14
,,';
Part I Biochem istry-Physiology Exam
December 1996 .

101. Which of the following vitamins is the LEAST 106. Which of the following represents the chemical -
likely to be invoived in tooth development and substance that is the immediate source of
calcification? energy for muscle contraction?

A A A Glycogen
B. B1 B. Acetyl CoA
C. C C. ..Lactic acid
D. 0 D. Creatine phosphate
E. Adenosine triphosphate

102. The major function of serum LDL is to transport


which of the following? .

A Bile salts from the. intestine 107. Intracellular and interstitial body fluids have
B. Triglycerides from the intestine similar
C. Free fatty acids from adipose tissue
D. Cholesterol and phospholipids from A total osmotic pressures.
peripheral tissues B. colloid osmotic pressures.
E. Cholesterol, cholesterol esters, and C. sodium ion concentrations.
phospholipids from the liver D. chloride ion concentrations.
E. potassium ion concentrations.
103. The fluid-mosaic model for membrane structure
proposes that

A the outer and inner faces of the


membrane are identical. 108. Breathing a gas mixture with 5 percent CO2
B. ultimately leads to which of the foHowing?
peripheral proteins are situated only on
the outer face of the plasma membrane.
C. A Hypoventilation
integral proteins are associated with the
B. A decrease in heart rate
hydrophobic phase of the bilayer.
D. C. A stimulation of central chemoreceptors
both polar and nonpolar ends of
D. An inhibition of peripheral chemoreceptors
membrane phospholipids are within the
E. A decrease in cerebrospinal fluid
hydrophobic phase of the bilayer.
hydrogen-ion concentration

104. A protein in solution is at its isoelectric point


when the 109. Ovulation is triggered by a marked increase in
which of the following?
A pH is the same as that of the blood.
B. logarithm of the concentration is zero. A Estrogen
C. pH allows for maximum solubility. B. .Estradiol
D. pH is such that no migration occurs during C. Progesterone
electrophoresis. D. Luteinizing hormone
E. pH is produced by a 1:1 mixture of the E. Follicle-stimulating hormone
protein solution with isotonic saline.

110. Which of the following must be digested before


105. The absolute refractory period of a nerve action being in a form that can be absorbed by
potential is determined by the duration of which enterocytes?
of the following?
A Monoglycerides
A Sodium activation gate opening B. Fatty acids
B. Sodium inactivation gate closure C. Fructose
C. Potassium activation gate opening D. Glycine
D. Potassium inactivation gate closure E. Maltose
"

111. Alpha-ketoglutarate, oxygen, and ascorbic 116. Where in the autonomic nervous system is
acid are essential for which of the following norepinephrine stored?
processes?
A Preganglionic sympathetic nerve
A Incorporation of proline endings
8. Hydroxylation of proline B. Postganglionic sympathetic nerve
C. Gamma-Carboxylation of proline endings. - - --
D. Oxidative deamination of lysine C. Preganglionic parasympathetic nerve
E. Activation of procollagen peptidase endings
D. Postganglionic parasympathetic nerve
endings

112. Low Density Lipoprotein (LDL)particles get


into cells by
117. Which of the followingwill inactivate human
A simple diffusion. immune deficiency virus (HIV)reverse
8. cell-cell fusion; transcriptase?
C. active transport.
D. facilitated diffusion. A ATP
E. receptor-mediated endocytosis. B. p.zr
C. dATP
D. Fluorouracil
E. Methotrexate
113. Which of the following represents the major
pathway for metabolism of excessive
intraneuronal free norepinephrine?
118. Which of the following hormones conserves
A Hydrolysis by cholinesterase body protein, carbohydrate, and fat stores?
8. Deamination by monoamine oxidase
C. Hydroxylation by - monoamine oxidase A Cortisol
D. Hydroxylation by dopamine beta 8. Glucagon
hydroxylase C. Insulin
E. Methylation by catechol-a-methyl D. Somatotropin
transferase

114. Assumingthat P50 =26 torrs, underconditions 119. Each of the followingis involved in gene
where p02 =30 torrs, the average number of cloning EXCEPT one. Which one is this
°2 molecules bound per hemoglobin EXCEPTION?
molecule is closest to
A DNA ligase
A 0.5. B. RNA polymerase
8. less than 1. C. DNApolymerase I
C. almost 2. D. Restriction nucleases
D. greater than 2. E. Reverse transcriptase
E. greater than 3.

115. In the DNA molecule, guanine on 1 strand is 120. Which of the followinghas a high affinityfor
joined to cytosine on the complementary binding calcium and collagen in the calcifying
matrix?
strand by which of the followingbonds?

A Amide A Calcitonin
8. 1 hydrogen 8. Osteogenin
C. 2 hydrogen C. Osteonectin
D. 3 hydrogen D. Amelogenin
E. Phosphodiester E. Fibronectin
16

~
...
...

121. In the glycolytic sequence, the enzyme ~hat 126. During oxidative phosphorylation, the "-

brings about the transition from 6-carbon energetically unfavorable proton gradient is
metabolites to 3-carbon metabolites is created using energy from which of the
following?
A phosphoglucoisomerase.
B. phosphofructokinase. A ATP hydrolysis -
C. phosphorylase. B. ATP synthesis
O. hexokinase. C. ADP transport
E. aldolase. D. Electron transfers
E. The reduction of NAD+

122. Which of the following represents a striated


muscle that contains transverse tubules, a
slow rate of calcium sequestration, and is
inhibited by acetylcholine?
127. Which of the following explains why enamel is
A Cardiac harder than bone?
B. Skeletal
C. Multi-unit smooth A Enamel crystals are larger and more
D. Single unit smooth firmly packed.
B. Enamel contains amelogenins in its
organic matrix. ""
C. Enamel contains more magnesium and
carbonate.
123. Tay-Sachs disease is associ~ted with an
inborn error of metabolism involving a specific
D. Enamel crystals have more surface
area.
enzyme which normally degrades a particular
E. Enamel contains more collagen.
molecule in the gray matter. This enzyme acts
on which of the following? "

A Polysaccharides
B. Acylglycerols
C. Gangliosides
D. Fatty acids
E. Proteins
128. Which of the following represent(s) the matrix
proteins of enamel?

A Carboxylglutamic acid containing


124. Vascular smooth muscle relaxes in response proteins
to B. Type I collagen
C. Amelogenins
A hyperoxia. D. Proteoglycans
B. adenosine. E. Elastin
C. vasopressin.
D. angiotensin.
E. norepinephrine.

125. Which of the following is MOST often


associated with free fatty acid transport in 129. ADH receptors in the nephron are located on
human blood? the tubular membrane of which of the
following?
A Albumin
B. Globulin A Distal tubule
C. Cholesterol B. Proximal tubule
D. Sphingqlipid C. Ascending loop of Henle
E. Mucopolysaccharide D. Descending loop of Henle
. .
.

130. Which of the following changes promotes the 135. What single substance is effective in
formation of extracellular edema? reversing ketosis in a non-diabetic patient?

A Increase in tissue fluid hydrostatic A Urea


pressure 8. Glucose
8. Increase in plasma protein C. Insulin
concentration D. Leucine
C. Decrease in capillary hydrostatic E. Palmitic acid
pressure
D. Capillary.filtration exceeds capillary
absorption
E. Capillary absorption exceeds capillary 136. Intensity of sound is dependent mainly on
filtration
which of the following physical characteristics
of sound waves?

131. In addition to phosphoric acid, which of the A Frequency


following are the products of hydrolysis of B.
lecithin? Amplitude
C. Wavelength
D. Secondary waves
A Glycerol, fatty acids, serine E.
B. Sympathetic vibrations
Glycerol, fatty acids, choline
C. Sphingosine, acetic acid, inositol
D. Glyceraldehyde, fatty acids, choline
E. 137. ATP inhibits phosphofructokinase even
Glyceraldehyde, fatty acids, though ATP also is a substrate for the
ethanolamine
enzyme. Which of the following types of
inhibitiol1s BEST explains. this phenomenon?
132. Which of the following BEST characterizes A Allosteric
hydroxyapatite? 8. Competitive
C. Irreversible
A Has an amphiphilic surface O. Uncompetitive
8. Contains 12 ions per unit cell E. Noncompetitive
C. As found in bone and enamel contains
no ion substitutions
D. Has a higher solubility product constant
than fluoroapatite
138. Which of the following BEST explains why
proteins are able to buffer .physiologic
solutions over a wide range of pH?
133. Polyuridylic acid in a cell-free system capable
of protein synthesis results in production of A They are macromolecules of high
polyphenylalanine. In this system, polyuridylic
acid functions as molecular weight.
B. They contain many functional groups
with differing pKs.
A DNA. C.
B. They have unique tertiary structures that
transfer RNA.
sequester hydrogen ions.
C. messenger RNA. D.
D. ribosomal RNA. They have peptide bonds that are
resistant to hydrolysis.
E. mitochondrial RNA.

134. A derivative of vitamin K is the coenzyme for


139. The intrinsic factor for vitamin 812absorption
which of the following? is produced in the
A Production of menadiol A liver.
8. Esterification of retinol B. stomach.
C. Hydrolysis of peptide bonds C. pancreas.
D. Cross-linking of fibrinogen D. duodenum.
E. Carboxylation of glutamate side chains E. lacteals.
18
....

140. Which of the following bonds link the 146. Glutamate decarboxylase, an enzyme that
monomeric units of nucleic acids? catalyzes the formation of gamma-amino
butyric acid (GABA), is unique to
A Ionic
B. Peptide A bone.
C. Thioester B. skin.
D. Glycosidic C. heart muscle.
E. Phosphodiester D. nervous tissue.
E. connective tissue.

141. Decreased response of sense organs when


exposed' to a constant stimulus is called

A occlusion.
147. The largest amount.of body water can be
B. summation.
found in which of the following?
C. adaptation.
D. facilitation. A Urine
E. sensory deprivation. B. Blood plasma
C. Intracellular fluid
D. Interstitial fluid
142. Emotional feelings are MOST closely related E. Stomach and intestines
to which area of the brain?

A Thalamus
B. Brain stem
C. Cerebellum
D. 148. MOST fluid reabsorption by the kidney occurs
Hypothalamus
E. Limbic system in which of the following?

A Distal tubule
B. Proximal tubule
143. Each of the following is a glycosaminoglycan
EXCEPT one. Which one is this EXCEPTION? C. Collecting duct
D. Ascending loop of Henle.
A Chondroitin sulfate E. Descending loop of Henle
B. Dermatan sulfate
C. Hyaluronic acid
D. Heparan sulfate
E. Keratin 149. Which of the following portions of the
cardiovascular system contains the greatest
volume of blood?
144. Which of the following minerals are
A Arterioles
considered to be cariostatic? ~\) B. Capillaries
A Fluoride Phos hfA(.O~ C. Systemic veins
B. Fluoride,
C. Co~~r,
D. ~rt6~m,
~ iu~
phate
Copper
E.' \:ruoride, Lead
D.
E.
Chambers of the heart
Pulmonary vasculature

150. Each of the following enzymes functions in


145. Each of the following describes hyaluronate association with a membrane EXCEPT one.
EXCEPT one. Which one is this EXCEPTION? Which one is this EXCEPTION?

A Polyanion A Succinate dehydrogenase


B. Highly polar B. Na+/K+ATPase
C. Glycosaminoglycan C. Adenylate cyclase
D. Compact, folded structure D. Phosphofructokinase
E. Extracellular matrix component E. Coenzyme Q reductase
.,
,
151. Which of the following generates a slowly 156. Hydrolysis of sucrose by the enzyme sucrase
developing long-term response in target yields
tissues by binding to an intracellular receptor?
A glucose only.
A Glucagon 8. glucose and maltose.
8. Estrogen C. glucose and fructose.
C. Prolactin D. glucose and galactose.
D. Growth hormone E. fructose and maltose.
E. Parathyroid hormone

152. Which of the following functions as part of the


extracellular matrix?
157. Calcium ions initiate contraction in skeletal
A Mucin muscle when they
8. Heparin
C. A bind to T tubules.
Collaginase 8.
D. Chondroitin sulfate bind to troponin.
E. C. interact with actin.
Dolichol phosphate D. interact with myosin.
E. bind to sarcoplasmic reticulum.
153. Which of the following represents the pH of a
solution that has a 10-5M concentration of OH-
ion?

A 5 158. Which of the following enzymes converts


8. 7 trypsinogen to trypsin? .

C. 9
D. Determinable only if the pkais known. A Enterokinase
E. Determinable only if the base 8. Peptidase
composition is known. C. Secretin
D. Pepsin

154. Compared to slow-twitch (Type I) muscle


fibers, each of the following is more
characteristic of fast-twitch (Type II) muscle
fibers EXCEPT one. Which one is this 159. Enzymes that catalyze the anaerobic
EXCEPT/Ow? . processes of carbohydrate metabolism are
found predominantly in which part of a cell?
A They contain more mitochondria.
8. They have higher myosin ATPase A Cytoplasm
activity. 8. Membrane
C. They have fewer capillaries surrounding C. Cell wall
them. D. Nucleus
D. They have a more extensive E. Mitochondria
sarcoplasmic reticulum.
E. They contain higher concentrations of
glycolytic enzymes.
160. Each of the following characterizes a peptide
hormone EXCEPT one. Which one is this
EXCEPT/Ow?
155. Which of the following is MOST likely to cause
respiratory acidosis?
A Stored in secretory granules
A 8. Synthesized in a precursor form
Moving from high altitude to sea level C.
8. Untreated diabetes mellitis 8inds to intracellular receptors
C. D. Acts by generating a second
Hypoventilation
D. messenger
Kidney failure E.
E. Vomiting Usually transported unbound in plasma
20
:

161. Which of the following substances represents 166. Each of the following represents an amino ,-

an unsaturated fatty acid? acid found in proteins and used directly in the
reactions of protein synthesis EXCEPT one.
A Cholesterol Which one is this EXCEPTION? -
B. Palmitate
C. Stearate A Proline
D. Choline B. Arginine
-

E. Oleate C. Tryptophan
D. Asparagine
E. Hydroxylysine

162. Which of the following acids represents both a


product of bacterial glycolysis and is
cariogenic? 167. For a reaction catalyzed by an enzyme with a
Km= 1 mM, which of the following represents
A Lactic the effect on the velocity if [S] is changed from
B. Acetic 10 mM to 20 mM? (Assume that the enzyme
C. Succinic obeys Michaelis-Menten kinetics.)
D. Propionic
E. Phosphoric A Small decrease
B. Small increase
C. Twofold decrease
.D. Twofold increase
E. Twentyfold increase
163. What linkages occur in glycogen at branch
points between glucose units?

A Alpha-1,4
B. Alpha-1,6 168. Each of the following is an effect of parathyroid
C. Beta-1,3 hormone EXCEPT one. Which one is this
D. Beta-1,4 EXCEPTION?
E. Beta-1,6
A Stimulation of 1-alpha-hydroxylase in
kidney
B. Stimulation of osteoclastic activity in
164. Protein kinase regulate the activities of key bo'ne
C. Stimulation of calcium reabsorption by
enzymes through which of the following?
kidney
A Oxidation D. Inhibition of phosphate reabsorption by
B. Hydrolysis kidney
C. E. Inhibition of intestinal absorption of
Acetylation
D. calcium
Phosphorylation
E. Dephosphorylation

169. Which of the following liver enzymes, absent


165. Regulation of each of the following
mechanisms is associated with the from other tissues, gives the liver an
advantage over other cells in taking up
hypothalamus EXCEPT one. Which one is
this EXCEPTION? glucose after a meal?

A A Glucokinase
Sleep Aldolase
B. Water balance B.
C. C. Hexokinase
Body temperature D. Enolase
D. Pupillary diameter
E. E. Glucose-6-phosphatase
Carbohydrate metabolism.
.,
,

170. In which of the following conditions might the 175. Which of the following enzymes or processes
systolic blood pressure be abnormally high? ensures that the correct amino acid is
incorporated for a particular codon during
.A Cardiac shock protein synthesis?
B. Heart failure
C. Anaphylactic shock A Amino Acyl-t-RNA synthetase"
D. Decreased arterial compliance B. Ribosomal protein synthesis
E. Ventricular fibrillation C. Post-transcription splicing
D. RNA synthetase
E. Helicase
171. Which of the following occurs primarily in
adults and is characterized by increased
airway resistance, decreased diffusing
capacity of the lung, and chronicJperman~ 176. Ifproteinis catabolized for energy,then MOST
hypoxia? ~~\J. of the energy is derived from which of the
following?
A

D. C~
E.
a
B. Asthma
Em
C. Pn hYSe ~ 0~ S(;O
a lung.
Hyalin membrane disease
A
B.
C.
D.
E.
Urea production
Oxidative deamination
Transamination reactions
Cleavage of peptide bonds
Oxidation of ex-ketoacids derived from
amino acids
172. The concentration of which of the following
amino acids can be used as an estimation of
the amount of collagen present in a tissue?

A 177. Each of the following lipid classes is


Hydroxyproline
B. incorporated into membranes EXCEPT one.
Aspartic acid Which one is this EXCEPTION?
C. Proline
D. Serine'
A Cholesterol
B. Ganglioside
C. Triglyceride
D. Sphingomyelin
173. If the air temperature is 105°F and the relative
E. Phosphatidylcholine
humidity is 10 percent, then which of the
following represents the primary cause for
loss of body heat?

A Evaporation 178. The amplitude of an action potential can be


B. Conduction increased MOST easily by increasingwhich of
C. Convection the following concentrations?
D. Radiation
E. Hyperventilation A Intracellular sodium
B. Extracellular sodium
C. Intracellular chloride
D. Intracellular potassium
174. Which of the following hormones is released E. Extracellular potassium
by the zona fasciculata and exerts permissive
actions to allow insulin, glucagon and
epinephrine to work more effectively at their
target tissues? 179. Decreased arterial pressure upon standing is
compensated by
A Androstenedione
B. Somatomedin A decreased heart rate.
C. Aldosterone B. dilation of mesenteric vessels.
D. Thyroxine C. constriction of systemic arterioles.
E. Cortisol D. dilation of venules.
22

~-
:

180. Synaptic vesicle contents are released at the 184. Each of the following combinations lists the .-

neuromuscular junction when which of the name of the hormone, its chemical type, and
following occurs? its major tissue of origin EXCEPT one. Which
one is this EXCEPTION?
A Hyperpolarization of motor end-plate
B. Shortening of skeletal muscle Hormone Chemical Tissue
Release of ATP from nerve terminal --
C. Type
D. Entry of calcium at nerve terminal
E. Release of acetylcholine from motor A ACTH Steroid Adrenal
end-plate cortex
B. Vasopressin Peptide Posterior
pituitary
C. Epinephrine Catecholamine Adrenal
medulla
D. Thyrotropin Glycoprotein Anterior
181. Which of the followingcharacterizes exergonic pituitary
reactions? E. Somatostatin Peptide Hypothalamus

A Decreased entropy
B. Increased enthalpy
C. Decreased enthalpy
D. Negativefree energychange
E. Positive free energy change 185. Which of the following causes activation of the
pyloricpump,relaxationof the pylorus,and
contractionof the loweresophageal sphincter?

A Gastrin
8. Secretin
C. Pepsinogen
D. Acetylcholine
E. Cholecystokinin
182. In an aqueous solution at pH 7, a peptide,
containing 1 amino group side chain and 2
carboxyl group side chains would have which
of the followingnet charges?

A 2+
. B. 1+ 186. An animal is in negative nitrogen balance if its
C. 0
D. 1- A urine is nitrogen-free.
E. 2- 8. nitrogen intake equals output.
C. nitrogen intake exceeds output.
D. nitrogen output exceeds intake.
E. new tissue is being synthesized.

183. A subject consumes 250 ml of oxygen per


minute with a tidal volume of 400 ml and a
respiratory rate of 18 per minute. Which of the
following represents this subject's respiratory 187. Each of the following can cause glucosuria
minute volume in liters? EXCEPT one. Which one is this EXCEPT/Ow?

A 1.8 A Low insulin level


B. 4.5 8. High blood sugar level
'" C.
C. 5.4 Impaired tubular reabsorption
~i D. 7.2 D. High glomerular filtration rate
., E. 10.0 E. High renal threshold for glucose
~i
~i
23
"

,
188. The clearance rate for a substance that is 192. Which of the following represents the major
completely removed from the blood during force that causes glomerular filtration?
one pass through the kidney is equal to which
of the following? A Tubular hydrostatic pressure
B. Tubular colloid osmotic pressure
A Renal plasma flow C. Glomerular capillary hydrostatic
B. Filtration fraction pressure
C. Urinary excretion rate D. Glomerular capillary colloid osmotic
D. Glomerular filtration rate pressure
E. Tubular transport maximum

193. If the plasma clearance of a substance which


is freely filtered is less than that of inulin, then
189. A marked fall from normal in the oxygen
tension in arterial blood would stimulate the A the substance becomes bound to
receptors in the protein in the tubules.
B. there is net secretion of the substance
A central nervous system in the tubules.
chemoreceptors. c. there is a net reabsorption of the
B. aortic arch and the carotid sinus. substance in the tubules.
C'. aortic and carotid bodies. D. the substance is neither secreted nor
D. walls of the great veins. reabsorbed in the tubules.
E. respiratory center. E. the substance is secreted in the
proximal tubule to a greater degree than
it is in the distal tubule.

190. Oxygen tension is GREATEST inwhich of the


following blood vessels?
194. Hypoactivity of the posterior pituitary gland
A Aorta leads to
B. Pulmonary vein
C. Pulmonary artery A dwarfism.
D. Coronary artery B. cretinism.'
E. Coronary vein C. acromegaly.
D. diabetes insipidus.

191. Which of the followinggroups includes only 195. The metabolite, 25-hydroxycholecalciferol, is
amino acids essential for humans? derived MOST immediately from

A Valine, serine, leucine A ergosterol.


B. Leucine, lysine, glycine B. cholesterol.
C. Tyrosine, threonine, tryptophan C. 7-dehydro-sitosterol.
D. Phenylalanine, methionine, proline D. 7-dehydro-cholesterol.
E. Tryptophan, methionine, isoleucine E. 22-dihydro-ergosterol.
24
"

196. In a complete heart block, the -


electrocardiograph shows which of the
following?
--
A An increased PO intelVal
B. An increase in height of the P wave
C. One ORS complex for every three P =
waves
D. An increase in amplitude of the ORS
complex
E. Dissociation of the P wave and the ORS
complex

197. Cardiac output is expressed as a product of

A stroke volume and heart rate.


B. venous pressure and heart rate.
C. stroke volume and respiratory rate.
D. stroke volume and diastolic filling.
E. venous pressure and coronary blood
flow.

198. MOST of the CO2in blood is combined as

A H2CO3.
B. HCO3-.
C." GH3-COOH.
D. carbonic acid.
E. carbaminohemoglobin.

199. A deficiency of choline in the diet can cause


abnormalities in the metabolism of

A lipids.
B. proteins.
C. minerals. '
D. carbohydrates.
E. nucleoproteins.

200. Which of the following is a function of the


enterogastric reflex?

A Decreases the opening of the


gastroesophageal sphincter
B. Increases motility of the esophagus
C. Decreases motility of the stomach
D. Increases motility of the ileum
E. Increases gallbladder emptying
'..
RELEASED
NATIONAL BOARD DENTAL EXAMINATIONS
PART 1

TEST: BIOCHEMISTRY-PHYSIOLOGY
FORM: 12 ~
DATE: D96

.
Item Key Item Key Item Key. Item ,Key

101 B 126 D 151 B 176 E


102 E 127 A 152 D 177 C
103 C 128 C 153 C 178 B
104 D 129 A 154 A 179 C
105 B 130 D 155 C 180 D

106 E 131 B 156 C 181 D


107 A 132 D 157 B 182 D
108 C 133 C 158 A 183 D
109 D 134 E 159 A 184 A
-110 .E 135 B 160 C 18.5 A

111" B 136 B 161 E 186 D


112 E 137 A 162 A 187 E
113 B 138 B 163 B 188 A
114 D 139 B 164 .D 189 C
115 D 140 E 165 D 190 B

116 B 141 C 166 E 191 E


117 B 142 E 167 B 192 C
118 C 143 E 168 E 193 C
119 B 144 - 169 A 194 D
120 C 145 D 170 D 195 D

121 E 146 D 171 - 196 E


122 A 147 C 172 A 197 A
123 C 148 B 173 A 198 B
124 B 149 C 174 E 199 A
125 A 150 D 175 A 200 C

- indicates item not scored


26 .."
-~
,;~
Part I Microbiology-Pathology Exam
Decef11ber J 996

1. Which of the following is CORRECT for obligate 6. An autosomal dominant trait showing 50
anaerobic microorgansims in the oral cavity? percent penetrance will be phenotypically
expressed in what percent of the offspring?
A They do not exist in this area.
B. Only gram-positive organisms are found. A 0
C. They are normal flora and opportunistic. B. -. 25
D. They are seldom isolated in the laboratory. C. 33
E. They can be completely controlled by D. 50
using antibiotics. E. 75

J 2. Malignant epithelial cells have an increased


number and wider distribution of which of the 7. Respiratory syncytial virus (RSV) infection differs
following receptors? frominfluenzainfectionsin that
'Iroooo A RSV can be treated with amantadine.
A Il-1
B. laminin B. RSVcauses disease primarily in infants.
C. Histamine C. influenza virus is difficultto transmit.
D. D. RSVcan be prevented with effective
-- Complement
vaccines.
E. Immunoglobulin
E secretory IgAis not effective in preventing
disease.
. -
3. Each of the following is a risk factor in .
atherosclerosis EXCEPT one. Which one is this
\000& EXCEPTION? 8. Which of the followingrepresent(s) the MOST
commonsource of pulmonary embolism?
A Heredity
B. Alcoholism A Esophageal varices
- c. Hypertension ~
B. Endarteritis' ._-

D. Diabetes mellitus C. lymphangitis .


E. Hyperlipoproteinemia D. Buerger's disease
E. Thrombophlebitis
--
4~ Which of the following is the single MOST
--- numerous group of microorganisms in the oral
cavity? 9. Aflatoxin is produced by

A Enterococci A Mucor.
B. Staphylococci B. Candida. i
C. Anaerobic streptococci C.
D. Facultative streptococci D.
Tricophyton.
Penicillium.
!
E. Beta-hemolytic streptococci E. \I
- Aspergillus.
I
I
l
5. Each of the following characterizes a chlamydiaI 10. Which of the following conditions predisposes
infection EXCEPT one. Which one is this
to lung cancer by causing squamous /
EXCEPTION?
metaplasia of bronchial epithelium?
A large numbers of asymptomatic carriers A Bronchiectasis
B. Frequent co-infection with gonorrhea B. Bronchial asthma
C. The ability of the organism to survive in the C. Chronic bronchitis
host extracellularly D. Bronchial carcinoid
D. The greater likelihood that younger E. g;
Pulmonary emphysema
women will acquire salpingitis

~,
..
..

11. . The likelihood that oral bacteria play an 16. A fungus that causes systemic disease, most
important role in gingival inflammation is commonly of the lungs, and is characterized
evidenced by which of the following? by its production of tuberculate
chlamydospores in culture is
A An increase in salivary hyaluronidase
B. An increased number of bacteria in A Microsporum canis.
saliva B. Mycoplasma hominis.
-

C. An increase of neutralizing antibodies in C. Leptospira pomona.


saliva D. Actinomyces israelii.
D. A reduction of inflammation with E. Histoplasma capsulatum.
reduction of plaque

17. Which of the following genera is MOST likely


12. In addition to Neisseriameningitidis which of involved in bacillary dysentery?
the following is a significant cause of
meningitis? A Vibrio
B. Shigella
A Streptococcus faecalis C. Entamoeba
B. Streptococcus pyogenes D. Salmonella
C. Streptococcus pneumoniae E. Escherichieae
D. Staphylococcus aureus
E. Staphylococcus epidermidis'

18. Prostacyclin and thromboxane are products of


13. Which of the following are MOST antigenic? which pathway?

A Lipids A Complement cascade


B.. Haptens B.. :..Cycl.ooxyg!:!nase
C. Proteins C. Lipoxygenase
D. Nucleic acids D. Fibrinolytic
E. Carbohydrates

14. Which of the following characterizes victims of 19. The MOST important viral cause of
fatal, acute carbon monoxide poisoning? gastroenteritis in children less than 2 years
old is
A Cherry red blood
B. Acute renal failure A rotavirus.
C. Massive liver necrosis B. echovi(1Js.
D. A hypercoagulability state C. rhinovirus.
E. Anemia and generalized white cell D. coxsackievirus.
depletion E. cytomegalovirus.

20. MOST bacterial endotoxins are composed of


15. Which of the following represents the chief which of the following?
complication of mumps in the adult male?
A Pure proteins
A Orchitis B. Pure carbohydrates
B. Prostatitis C. Heteropolysaccharides
C. Glomerulonephritis D. Mucoprotein complexes
D.. Chronic nonspecific sialadenitis E. Lipo prote in-polysaccharid e complexes
..
"
21. Which of the following is the principal factor 26. Which of the followingdescribes the function
underlying localized edema in inflammation? of adjuvants?
A Lymphatic obstruction A Enhance secretion of IgA
B. Obstruction of venous outflow B. Enhance antibody response
C. Reduced intravascular osmotic C. Stimulate complement synthesis
pressure D. Desensitize to a given antigen
D. Protein leakage into tissue spaces E. Activate mast cell degranulation
E. Increased capillary permeability

22. The MOST common source of massive 27. A patient lapses into a state of fluctuating
hematemesis in alcoholics is (are) levels of consciousness or coma several
hours after sustaining blunt trauma to the
A peptic ulcer. head. This finding is consistent with which of
B. acute gastritis. the following conditions?
C. esophageal varices.
.Q:,,:;.Mallory-Weiss syndrome. - ,,'- A Subdural hematoma
E. acute hemorrhagic pancreatitis. B. Cerebral concussion
C. Cerebral infarction
D. Acute encephalitis
E. Alzheimer's disease
23. A 17-year -old patient has periodontitis
involvingthe anterior teeth with sparse
plaque. Which of the followingis the probable
primary pathogen?
28. Which chemical substance is usually
A Actinobacillus actinomycetemcomitans secreted by pheochromocytomas?
B. Bacteroides forsythus
C. Fusobacterium nucJeatum ,A Catecholamine j

D. Porphyromonas gingivaJis B. Aldosterone I


I
E. Prevotella intermedia C. Cortisone II
D. Insulin
E. Renin I

24. Which of the following represents the MOST


reliable postmortem indicator of left venticular
cardiac failure? 29. Rubella and toxoplasmosis are similar in that
both
A Ascites
B. Venous congestion A are potentially teratogenic.
C. Enlargement of the spleen B. primarilyaffect the elderly.
D. Peripheral edema of the ankles C. are prevented by vaccinations.
E. Chronic passive congestion of the lungs D. have non-human animal reservoirs.
E. can not be treated with antibiotics.

25. Which of the following represents the MOST


frequent cause of a clinically palpable breast 30. Which of the followingprevent(s) the
mass in an adult woman? synthesis of peptidoglycans?

A Sarcoma A Lysozymes
B. Fibroadenoma B. Penicillin
C. Adenocarcinoma C. Tetracyclines
D. Fibrocystic disease D. Erythromycin
E. Intraductal papilloma E. Teichoic acids
~
j
4
:

31. Which of the following is MOST likely to cause 36. Which of the following do striated muscle, .-

a sudden arrest of heart function? smooth muscle, and cardiac muscle have in
common?
A Mitral stenosis -
B. Angina pectoris A Hyperplasia of these elements is
C. Constrictive pericarditis common.
-
D. Cardiac tamponade B. They have a limited capacity to
E. Subacute bacterial endocarditis regenerate.
C. Hypertrophy is a common response to
inJury. .
D.
They need a constant high °2
32. If a bacteria were susceptible to both penicillin concentration to function.
and erythromycin, then it would not be
appropriate to treat the patients with both
antibiotics at the same time because
37. Which of the following cells are thought to be
A penicillin is inactivated by erythromycin.
B. erythromycin exerts its effect MOST important in the control of metastase~
extracellularly.
C. erythromycin enhances beta-Iactamase A B cells GO~~
activity. B. Macrophages
C. Killer(K)cells ~O
r\ c:»
D. erythromycin is primarily an anti-fungal D. Cytotox~"i-!tlhh\C\ttes
agent.
E. E. NaturC1\~P<"iK)cells
penicillin is only effective against
growing cells.

38. A deficiency in which of the following cells can


33. Which of the following represents a DNA predispose to candidiasis?
gyrase inhibitor with a broad spectrum of
activity? A Basophils
B. Eosinophils
A 'Polymyxin C. Macrophages
B. Rifampicin D. Plasma cells
C. Ethambutol E. T lymphocytes
D. Cycloserine
E. Ciprofloxacin

39. T lymphocytes are MOST directly involved in


34. which of the following?
Which of the following BEST describes the
similarity between Streptococcus pneumoniae
A Immediate hypersensitivity
and Cryptococcus neoformans?
B. Immune complex injury
A C. Complement synthesis
Commonly found in soil samples
B. D. Contact dermatitis
Readily prevented by a vaccine
C. E. Antibody formation
Readily treated by tetracycline
D. Antiphagocytic polysaccharide capsules

40. Cerebral embolism occurring as a


35. The presence of which of the following in a complication of myocardial infarction is MOST
patient's serum affords protection agaiAst indicative of which of the following?
hepatitis B?
A Mural thrombosis
" A B. Phlebothrombosis
Anti-HBcAg
B. Anti-HB sAg C. Decreased stroke volume
C. Anti-HAV D. Lack of collateral circulation
D. Anti-HBeAg E. Disseminated intravascular coagulation

5
'..
j]

46. In the process of necrosis, a reduction in the


41. Natural killer cells capable of destroying
size of the nucleus and a condensation of
malignant cells MUST be first activated with nuclear material is known as
which of the following?
A pyknosis.
A IL-2
B. karyolysis.
B. Kinin
C. karyorrhexis.
C. Tryposin D. metachromasia.
D. Complement E. hyperchromatism.
E. Histamine

42. The major cariogenic property of 47. In myasthenia gravis, autoantibodies are
Streptococcus mutans is associated with its
directed against which of the following
ability to produce which of the following structures?
enzymes?
A Sarcomere
A Hyaluronidase B. Myofascia
B. Chondroitinase
C. Myocyte nucleus
C. Aminopeptidase D. Sarcolemmal membrane
D. Glucosyltransferase E. Acetylcholine'receptor
E. Fructosyltransferase

43. The oral lesions of herpangina can often be


48. Each of the following represents a common
distinguished from hand-foot-and-mouth
opportunistic infection associated with HIV
disease by which of the following? EXCEPT one. Which one is this EXCEPTION?
A Their physical appearance A Candidiasis
B. Their intraoral locations
B. Hairy leukoplakia
C. Their density and number C. Adenovirus conjunctivitis
D. Their gradual confluence D. Cryptosporidium enterocolitis
E. Pneumocystic carinii pneumonia

44. Interleukin-1 and tumor necrosis factor in


inflammation are produced by which of the
following?
49. Acute biliary obstruction produces a rise in
A Platelets
B. Mast cells A urobilinogen in urine.
C. Lymphocytes B. urobilinogen in stool.
D. Plasma cells C. conjugated bilirubin in urine.
E. Activated macrophages D. conjugated bilirubin in serum.
E. unconjugated bilirubin in serum.
j
';~1

45. A viremia is characterized by which of the


following? 50. Which of the following is the usual cause of .~
~
chronic pyelonephritis?
A Infected cells are transformed.
B. Infected cells are no longer releasing A Systemic hypertension
virus. B. Type IV hypersensitivity
c. The virus is most easily spread via C. Chronic glomerulonephritis
gastrointestinal secretions. D. Bacteremic seeding of the kidneys
D. The virus is most susceptible to E. Infection resulting from urinary reflux
circulating antibodies.

6
,.,.

51. If serial dilutions of human saliva are 55. Which of the following is the first inflammatory .-
inoculated on blood agar plates, then cell to appear in large numbers at the site of
incubated in air for 24 hours at 37°C, which of injured tissue?
the following groups of microorganisms will'
A -
grow in the greatest number? Macrophage
B. Plasma cell .
A Actinomycetes C. Neutrophil -
B. Streptococcus D. Lymphocyte
C. Lactobacillus E. Monocyte
D. Staphylococcus
E. Fusobacterium

56. Which of the following is an antimicrobial


agent that primarily inactivates cellular DNA?
52. High levels of Porphyromonas A Phenols
gingivalis-specific antibodies are seen in B. Chlorhexidine
MOST of the serum samples from patients C. Ethylene oxide
with adult periodontitis. These antibodies are D. 70 percent isopryl alcohol
which of the following? E. Alkaline glutaraldehyde
A IgM
B. IgG
C. IgE
D. IgA 57. Which of the following is MOST resistant to
E. IgD sterilizing conditions?

A Oral yeast
B. Hepatitis B virus
C. Bacterial endospore
D. Mycobacterium tuberculosis
53. Excessive deposits of copper in liver cells,' E. Human immunodeficiency virus
degenerative changes in the brain, and a
greenish-brown ring at the outer margin of the
cornea characterize which of the following?
58. Anaerobic bacteria lacking catalase are
A Galactosemia
generally
B. Phenylketonuria
C. Wilson's disease A motile.
D. Diabetes mellitus B. sensitive to H2O2.
E. T ay-Sachs disease C. able to ferment lactate.
D. resistant to penicillin.
E. producers of °2 and H2Ofrom H2O2'

54. MOST antibiotic resistance in bacteria is 59. A 6-year-old boy has minute white specks on
caused by which of the following? the oral mucosa adjacent to his first molars. A
bluish-red ring surrounds these spots. He
'i
A Genes that are carried on plasmids appears to have a cold and his eyes are red
B. Depression of a previously existing and runny. There is a blotchy reddish rash
gene pool behind his ears and on his face. This child
C. An ever increasing mutation rate in has which of the following?
bacteria
D. Unequal distribution of antibiotics into A Eczema
body spa~es B. Measles
E. Induction of specific mutations by C. Chickenpox
specific antibiotics D. Scarlet fever

7
""". -"

.,
,
60. In anemia resulting from drug-induced bone 65. The spores of Bacillus anthracis are
marrow suppression, the peripheral blood destroyed by
smear shows erythrocytes to be
A refrigerating (-7°C for 48 hours).
A hypochromic - microcytic. B. autoclaving (121°C for 20 minutes).
B. hypochromic - normocytic. C. pasteurizing (61.7°C for 30 minutes).
C. normochromic - normocytic. D. immersing in boiling water (100°C for
D. normochromic-microcytic. 10 minutes).
E. hyperchromic - macrocytic. E. placing in a hot air oven (121°C for 20
minutes).

61. A hemorrhagic tendency is seldom seen in 66. Congestion in the early stages of
which of the following conditions? inflammation is caused by which of the
following?
A Scurvy
B. Acute leukemia A Ischemia
C. Renal insufficiency B. Venous dilation ,
D. Hepatic insufficiency C. Active hyperemia
t

E. Secondary thrombocytopenia D. Venous constriction


E. Lymphatic obstruction I
J

I
i
I

62. In which of the following is the largest amount 67~ The finding of yeast cells and . -I
I;
of genetic information transferred from one chlamydospores in the oral mucosa suggests
cell to another? which of the following?

A F-dudion A Candida albicans


B. Conjugation B. Histoplasma capsulatum
C. Transduction C. Blastomyces dermatitidis
D. Transformation D. Cryptococcus neoformans
E. Recombination E. Trichophyton mentagrophytes

68. Which of the following kinds of mutations is


63. In a dry-heat oven, which of the following caused by base analogues?
temperatures is sufficient for achieving
sterilization in 1-2 hours? A Deletions
B. Inversions
A 81°C C. Frameshifts
B. 100°C D. Transitions
C. 121°C E. Transversions
D." 160°C

69. Which of the following acids is the chief


64. Which of the following is the site of oxidative product of carbohydrate metabolism of
phosphorylation in bacteria? Streptococcus mutans?

A Nucleus A Acetic
B. Ribosome B. Formic
C. Cytoplasm C. Lactic
D. Cell membrane D. Butyric
E. Mitochondrion E. Propionic ,

8
..
70. Which of the followingorganisms represents 75. Laboratory studies reveal megaloblasts in ,-

a significant secondary invader of carious bone marrow, and a hyperchromic, macrocytic


lesions? anemia in peripheral blood. Oral examination
reveals atrophic glossitis. Which of the .

A Bacillus subtilis followingis the MOST likely cause of this


B. Lactobacillus casei condition?
-
C. Streptococcus mutans
D. Staphylococcus aureus A Iron deficiency
B. Chronic external loss of blood
C. Frequent episodes of hemolysis
D. Inhibition of hematopoiesis by
71. Inflammatoryvasodilation can be evoked by sulfonamides
each of the followingEXCEPT one. Which one E. Inabilityof the stomach to form intrinsic
is this EXCEPTION? factor '

A Renin
B. Histamine
C. C3and Cs 76. Gout results from a metabolic defect in which
D. Bradykinin of the following?
I E. Prostaglandins
,i
I A
B.
Fat
Purine
C~ Pigment
I
! 72. D. Glucose
The histamine that is released by mast cells
is.responsible for the principal symptoms of E. Calcium
.\
;
which of the following?

A Delayed hypersensitivity
B. Tuberculin reaction 77. Which of the followingviral-associated
C. Contact dermatitis enzymes is unique to RNA tumor viruses?
D. Arthus reaction
E. A Neuramidase
Anaphylaxis
B. Capping enzyme
C. Reverse transcriptase
D. DNA-dependent RNA polymerase
73. Which of the following neoplasms appears
the MOST often in children?
78. Which of the following bowel diseases is
A Neuroblastoma characterized by noncaseating
B. Chondrosarcoma
granulomatous inflammation in the gut wall?
C. Adenocarcinoma
D. Multiple myeloma A Diverticulitis
E. Basal cell carcinoma B. Peptic ulcer
C. Ulcerative colitis
D. Celiac disease
E. Crohn's disease
74. When horse serum is injected intravenously
into a rabbit and again into the skin two or
three weeks later, what is the necrotizing 79. Each of the followingcells is an
reaction that occurs at the site of the second antigen-specific cell EXCEPT one. Which one
injection? is this EXCEPTION?
'.
A Atopy A B cells
8. Anaphylaxis B. Macrophages
C. Serum sickness C. T helper cells
D. Arthus phenomenon D. Langerhans cells
E. Prausnitz-Kustner reaction E. Dendritic cells

9
"

, f

80. Streptococcus pyogenes, group A, is 86. A 53-year-old patient has an indurated.


subdividedinto specificantigenictypes chronic ulcer near the inner canthus. The
principally on the basis of immunologic MOST likely diagnosis is
differences in its '

A verruca vulgaris.
A Mprotein. B. malignant melanoma.
B. streptolysin O. C. basal cell carcinoma.
C. streptolysin S. D. squamous cell carcinoma.
D. C polysaccharide. E. adenocarcinoma of the lacrimal duct.
E. hyaluronic acid capsule.

81. Which of the followingbacterial exotoxins


converts plasminogen to plasmin? 87. Which of the following bacteria causes
A M-protein
B. Coagulase epidemics of meningitis among adults? i-Q
C. Plasmalysin A Staphylococcus aureus c...f;;O~
D. B. Klebsiella pneumoniif' -;;I
Hyaluronidase C. Haemophilus iQ~)e
E. Streptokinase
D. Neisseri~ ~~~s
E. Stre\~~~r pneumoniae
82. Which of the followingis the LEAST likely
complication of nodular prostatic hyperplasia?

A Dysuria
B. Hydronephrosis
C. 88. Eosinophils are characteristically seen in
Pyelonephritis
D. Development of carcinoma which of the following conditions?
E. Urinary tract obstruction
A Bacterial infections
B. Viral infections c,

83. A positive tuberculin test always indicates C. Fungal infections


D. Parasitic infestations
which of the following?

A Active tuberculosis
B. Chronic tuberculosis
C. No exposure to tuberculosis
D. Hypersensitivity to tuberculoproteins 89. Bone pain, osteolytic lesions, plasma cell
infiltrationof marrow and synthesis of
abnormal immunoglobulins describe
84. An endocrine disorder affecting the skeleton
and kidney, that promotes metastatic A systemic lupus erythematosus.
calcification, is B. selective IgAdeficiency.
C. polyclonal gammopathy.
A hyperadrenalism. D. Paget's disease of bone.
B. hyperthyroidism. E. multiple myeloma.
C. hyperpituitarism.
D. hyperparathyroidism.
E. hyperfunction of the thymus.

90. Which of the followingcharacterizes edema


85. Which of the followingtumors is of connective associated with the nephrotic syndrome?
tissue origin?
A Polyuria
A Myxoma B. Hematuria
B. Adenoma C. Hyperlipidemia
C. Melanoma D. Hypoalbuminemia
D. Carcinoma E. Hypoprothrobinemia
E. Papilloma
10
....

91. Blood in the sputum is characteristic of each 96. The MOST important characteristic of -
of the following EXCEPT one. Which one is malignant neoplasms, which distinguishes
this EXCEPTION? them from benign neoplasms, is their
.
A Emphysema A nonencapsulation.
B. Tuberculosis B. rapid growth rate. -
C. Lobar pneumonia C. ability to metastasize.
D. Pulmonary embolism D. lack of differentiation.
E. Bronchogenic carcinoma E. excessive mitotic activity.

97. Which of the following conditions increase the


92. The majority of cases of pharyngitis are risk of developing osteosarcoma?
caused by
A Osteomalacia
A alpha-hemolytic streptococci. B. Osteoporosis
B. Staphylococcus aureus. C. Osteoblastoma
C. a variety of viruses. D. Osteitis deformans
D. herpes simplex virus. E. Osteogenesis imperfecta
E. hemophilus influenzae.

98. Which of th~ following components of


Staphylococcusis antiphagocytic,, elicits
93. Hypersensitivity to M. Tuberculosis is hypersensitivity, and causes platelet injury?
manifested by which of the following?
A Protein A
A Necrosis B. Coagulase
B. Exudation C. Beta toxin
C. Epithelioid cells D. Teichoic acid
D. Langhans' giant cells E. Polysaccharide capsule
E. Spreading of the initial focus

99. Each of the following is a histologic feature of


malignant growth EXCEPT one. Which one is'
94. Administration of tetanus toxoid provides what this EXCEPTION?
type of immunity?
A Aplasia
A Innate B. Anaplasia
B. Natural active C. Pleomorphism
C. Natural passive D. Hyperchromatism
D. Artificial active E. Abnormal mitosis
E. Artificial passive

100. Immunological resistance to MOST


intracellular pathogens is manifested as
95. Which of the following represents the MOST which of the following?
potent carcinogen?
A Lymphoid .atrophy
A Estrogen B. Humoral immunity
B. Benzpyrene C. Cellular immunity
C. Folic acid D. Wheal and flare reactions
D. Cholic acid E. Non-specific serum protection

11
,; .

..
. RELEASED
NATIONAL BOARD DENTAL EXAMINATIONS
PART 1

TEST: lVIICROBIOLOGY-PATHOLOGY
FORM: 13
DATE: D96

Item . Key Item Key Item Key Item Key.

1 C 26 B 51 B 76 B
2 B 27 A 52 B 77 C
3 B 28 A 53 C 78 E
4 D 29 A 54 A 79 C
5 C 30 B 55 C 80 A
.

6 B 31 D 56 C 81 E
7 B 32 E 57 C 82 D
8 E 33 E 58 B 83 D
9 E 34 D 59 B 84 D
10 C 35 B 60 C 85 A
.-

11 D 36 B 61 C 86 C
12 C 37 - 62 B 87 -
13 C 38 E 63 D 88 D
14 A 39 D 64 D 89 E
15 A 40 A 65 B -90 D

16 E 41 A 66 C 91 A
17 B 42 D 67 A 92 C
18 B 43 B 68 D 93 A
19 A 44 E 69 C 94 D
20 E 45 D 70 B 95 B

21 E 46 A 71 A 96 C
22 C 47 E 72 E 97 D
23 A 48 C 73 A 98 A
49 D 74 D 99 A ~
24 E
25 D 50 E 75 E 100 C

- indicates item not scored


12
Part I Dental Anatomy and Occlusion.
December 1996

.-
All test items refer to permanent teeth unless 105. 'tl11ichof the following incisorshas its mesial
"primary" is specified. and distal contact areas at the same
incisocervical level?
All test items relating to occlusion refer to a Class I
canine and molar relationship unless otherwise A Maxillarycentral
specified. Terms such as "norma'" or "ideal" are B. Mandibular central -
synonymous with the above definition. C. Maxillarylateral
D. Mandibular lateral

101. In an ideal intercuspal relation, the lingual cusp


of the maxillary second premolar contacts the
106. 't'hich of the following is correct regarding the
A distal fossa of the mandibular second contact relationship of maxillary central incisors
premolar. and maxillary lateral incisors?
B. mesial marginal ridge of the mandibular
I first molar. A Contacts are centered faciolingually.
C. 6. Contacts are centered incisocervically.
distal marginal ridge of the mandibular C.
second molar. Facial embrasures are wider than lingual
I embrasures."'~ .
I
D. distal marginal ridge of the mandibular D. Incisal embrasures are larger than
first premolar.
gingival embrasures.

102. Which of the followingincisal angles of maxiJIary 107. Tne distal contact area -ofa maxillary lateral
teeth exhibits the.greatest convexity? incisor with ideal alignment is located

A Mesioincisal of the central A near the incisal edge.


B. Distoincisal of the central 8. in the middle third.
C. Mesioincisal of the lateral C. in the incisal third.
D. Distoincisalof the lateral D. at the junction of the incisal and middle
thirds.
E. at the junction of the middle and cervical
thirds.

103. Which of the followingrepresents the geometric


form of anterior teeth when viewed from the
mesial or distal aspect?
108. Which of the followingteeth have long axes
A Rhomboidal positioned with their root apices facial and their
B. Trapezoidal crowns lingual?
C. Elliptical
D. Triangular A - Maxillaryincisors
E. Square B. Mandibular incisors
C. Maxillary premolars
D. Maxillarymolars
E. Mandibular molars
.,

104. Each of the followingcusps of the maxillary


molars is part of the maxillarymolar primary
t
cusp triangleEXCEPTone. Whichone is this
EXCEPTION? 109. In (at) which of the following is the mesial
contact area of a maxillary canine located?
A Mesiofacial
A The incisal third
B. Mesiolingual B. The junction of incisal and middle thirds
C. Distofacial
C. The middle third
D. Distolingual D. The junction of middle and cervicalthirds

13
.,
,

110. Which of the following represents the tissue 115. Maxillarytooth crowns exhibit concavities on
formed in response to stimuli produced by which of the followingsurfaces?
carious penetration of a tooth?
A The mesial of central incisor and first
A Predentin premolar
B. Primary dentin B. The mesial of canine and first molar
C. Secondary dentin C. The distal of first premolar and the
D. Interglobular dentin mesial surface of second molar
D. The mesial of first premolar and the
distal of first molar
111. Calcification of the mandibular third molars E. The distal of the first molar and the
generally begins at mesial of the second molar

A 3-4 years.
B. 5-7 years.
C. 8-1 0 years. 116. The cross-sectional view at midroot illustrated
D. 11-13 years.
E. belowis MOSTprobablythat of a
14-16 years.

112. Each of the following morphologic structures


can be seen on any incisor EXCEPT one.
Which.one is this EXCEPTION?

A Cingulum
B. Mesial marginal ridge
C. . Lingual fossa
D. Transverse ridge A mandibular canine.
B. maxillary first premolar. .

C. mandibular first premolar.


113. In contrast to maxillary canine crowns, D.
mandibular canine crowns have which of the lingual root of a maxillary molar.
E. distal root of a mandibular first molar.
following anatomically?

A Greater measurements, mesiodistally


B. Greater measurements, faciolingually
C. More accentuated marginal ridges
D. Well-defined lingual pits 117. Which of the following jaw positions is
E. Less-pronounced cingula determined almost exclusively by the behavior
of the musculature?

A Postural
B. Intercuspal
114. Illustrated below is a frontal border tracing of a C. Retruded contact
patient with a canine-guided occlusion. Which D. Protruded contact
letter on the diagram defines the only point
where posterior tooth contact occurs?

A
b ~
118. In an ideal intercuspal relation, the
mesiolingual cusp of-the maxillary first molar
contacts the mandibular first molar in (on) the ."'.

.~
A central fossa.
B. distal fossa.
A A C. mesial fossa.
B. B D. mesial marginal ridge.
C. C E. distal marginal ridge.
D. 0

14
.
~ 119. In an ideal intercuspal relation in a normal 124.The periodontal ligament fibers are primarily --
dentition, each of the following teeth contacts composed of which of the fol/owing
each other EXCEPTone. Whichone is this - connective tissues?
EXCEPTION?
-
A Hyaline
A Maxillary first premolar and mandibular 8. Elastic
second premolar C..90Ilagenous
8. Maxillary first molar and mandibular D. Fibrocartilagenous
second premolar
C. Maxillary second premolar and
mandibular first molar
D. Maxillary second molar and mandibular
third molar

120. Which of the following represents the largest


125. On the non-working side in an ideal
cusp of the mandibular first molar?
occlusion, interfering contacts on posterior
A Distal teeth will be located on which inclines of
8. Mesiofacial which cusps?
C. Mesiolingual
D. Distofacial Inclines Cusps
E. Distolingual A Outer Guiding
8. Inner Guiding
C. Cuter Supporting
121. In cervical cross-section, the root of a D. Inner Supporting
mandibular canine is described as

A triangular.
8. roughly conical.
C. flattened in a mesiodistal direction.
-D. broader mesiodistally on the lingual
than on the facial.

126. Which premolar has a facial cusp with a


122. In an ideal intercuspal relation, the tip of the triangular ridge so uniquely prominent as to
mesiofacial cusp of the maxillary second frequently separate its mesial pit from its
molar opposes which of the following? distal pit?

A Facial groove of the mandibular second A Maxillaryfirst


molar 8. Mandibular first
8. Distofacial developmental groove of the C. Maxillary second
first molar D. Mandibular second
C. Embrasure between the mandibular
first and second molars
D. Embrasure between the mandibular
second and third molars
E. Central fossa

127. Which of the following ligaments has an outer


123. Which of the following primary teeth would oblique portion which limits the extent of jaw
" exhibit a prominent cervical ridge on both the opening and initiates translation of the
facial and lingual surfaces? condyle down the articular eminence?

A A A Capsular
8. F 8. Collateral
C. J C. Stylomandibular
D. L D. Temporomandibular
E. T

15
.,
, iiI

.~

128. The arrows on the illustration below represent the 133. The drawing below illustrates the occlusal
path taken by the opposing contacting cusps. The view of which premolar?
mandibular movement indicated is

A protrusive.
8. right lateral; working side. A Maxillaryfirst
C. left lateral; working side. 8. Mandibular first
D. right lateral; non-working side. C. Maxillarysecond
E. left lateral; non-working side. D. Mandibular second

129. Which of the followingrepresents the norl1!al


eruption age (years) of the maxillarylateral 134. Which of the following illustrates the occlusal
incisors? . view of a primary maxillary right first molar?
.,~~ _.,.,.,~:'-
-. ..-.

~~
A 4-5 years &tJ
8.
C.
6-7 years
8-9 year~
~~: :"';~;;:j;
L. -'::~...:' "'qL..~/
:
~.:.:t<) W..J
to'.,--',
:

D. 10-11 years 2 3 4 5

A 1
130. Which of the followingmaxillaryteeth has the B. 2
largest cervico-occlusal crown height? C. 3
D. 4
A First premolar E. 5
B. Second premolar
C. First molar
D. Second molar

135. The drawing below illustrates a cross section


131. The largest incisal/occlusal embrasure is located at the cementoenamel junction. This section
between which of the followingteeth? is typical of which of the following teeth?
A

(~
Maxillarycentral and lateral incisors
8. Mandibular central and lateral incisors
C. Maxillarylateral incisor and canine
D. Maridibularlateral incisor and canine
E. Maxillarycanine and first premolar A Maxillaryfirst premolar
B. Mandibular second premolar
C. Maxillarymolar
132. In an ideal intercuspal relation, the arrow on the D. Mandibular molar
illustration below represents the path taken by
which cusp of which molar?

136. When a protrusive mandibular movement


(anterior teeth edge-to- edge) is achieved, the
mandibular first molar has the potential to
.~'
contact which of the following maxillaryteeth?

Cusp Molar A First and second premolars


B. Second premolar only
A Mesiofacial Second C. Second premolar and first molar
8. Mesiolingual Second D. First and second molars
C. Distofacial Second E. Second molar only
D. Distolingual Second
E. Mesiolingual Third
16
--

137. Which of the following is characteristic of 142. The root tip MOST likely to be forced into the
--

primary maxillary canines? maxillary sinus during surgical removal is that


of a maxillary
A Crown height < mesiodistal width.
B. Cusp is low and rounded. A central incisor.
C. The mesioincisal cusp ridge is shorter B. canine.
than the distoincisal cusp ridge. C. first premolar.
D. The mesioincisal cusp ridge is longer D. first molar.
than the distoincisal cusp ridge.

138. Which of the following can adversely affect the


self-cleaning quality of a dentition in no,rmal 143. The outline below of a pulp cavity indicates
alignment? that this tooth probably

A
B.
C.

D.
Contact of adjacent teeth
Efficient use of a toothbrush
Friction of food material during
mastication A
t
B.
=--
has 3 distinct cusps.
=

Too great a contour of the cervical has experienced trauma.


enamel ridge C. is that of an old person.
D. is that of a young person.

139. In ~n acquired Class III crossb1terelationship,


as the mandible retrudes, the maxillary lateral
incisor contacts which of the following teeth? 144. On permanent teeth, the greatest incisal
curvature of a cervical line is on which surface
A Central incisor of which incisor?
B. lateral incisor
C. Central and lateral incisors Surface Incisor
D. Canine and lateral incisors
A Mesial Mandibular central
B. Distal Mandibular central
C. Mesial Mandibular lateral
140. Which incisor is MOST frequently markedly D. Distal Maxillary lateral
concave on the lingual surface? E. Mesial Maxillary central

A Maxillary central
B. Mandibular central
C. Maxillary lateral
D. Mandibular lateral

145. Which of the following illustrations represents


the mesial view of a mandibular right lateral
incisor?
141. On which of the following teeth does the
occlusal anatomy often require the placement
of separate MO and DO restorations?

u ij 2
\j3
fJ
4 2 3 4
A 1 A 1
B. 2 B. 2
C. 3 C. 3
D. 4 D. 4
..'

'S
S ~
~
..
'~"f

146. In a Class II occlusal relationship, the tip of 151. The midroot cross-sectional diagram below ;}
the facial cusp of a mandibular first premolar illustratesthe root of which maxillarymolar?
lies directly below the contacting area
between which maxillary teeth? Viewed from the occlusal

A Canine and lateral incisor


B. Canine and first premolar
C. ()G
First and second premolars
D. Second premolar and first molar (0
A Right first
B. Left first
C. Right third
147. Which extrinsic muscle of the tongue D. Left third
functtons to retract the tongue?

A Hyoglossus
B. Styloglossus
C. Genioglossus
D. Palatoglossus
152. In norma', occlusion, and in a left working
movement, the mesiofacial cusp of the
maxillary left second molar passes through
which of the following mandibular structures?
148/ When compared with maxillary first
premolars, the central developmental groove A The embrasure between the left first
of maxillary seco.nd premolars is -and second premolars' . .,
B. The embrasure between the left first
A longer with less supplementary and second molars
grooves. C. The facial groove of the left first molar
B. shorter with less supplementary D. The facial groove of the left second
grooves. molar
'-- C. longer with multiple supplementary E. The distofacial groove of the left first
grooves. molar
D. shorter with multiple supplementary
grooves.

149. Which of the following teeth in the permanent 153. Which of the 'following represents the third
dentition normally have bifurcations? pair of permanent teeth to erupt in normal
sequence?
A Mandibular canines
B. Mandibular second premolars A Maxillary central incisors
C. B. Maxillary lateral incisors }:
Maxillary molars and maxillary first
C. Mandibular lateral incisors
premolars
D. Mandibular molars and maxillary first D. Mandibular canines
premolars
E. Maxillary first and second molars

154. In an ideal intercuspal position, the distofacial


150. At 9 years of age how many primary teeth cusp of a maxillary first molar opposes which
remain in the mouth? feature on a mandibular first molar?

A 0 A The distal cusp


B. 4 B. The distofacial cusp
C. 8 C. The distofacial developmental groove
D. 12 D. The mesiofacial developmental groove
E. 18

18
~
.
155. In an ideal intercuspal relation, the
mesiofacial cusps of the mandibular second
160.Which premolar MOST likely possesses a .-

crescent-shaped central developmental


molars contact the maxillary molars in (on) the groove?
, A distal fossae of the second.
, A Maxillary first
B. central fossae of the second. B. Mandibular first
C. lingual embrasures between the first C. -
Maxillary second
and second. D. Mandibular second
D. facial embrasures between the first and
second. .

E. mesial marginal ridges of the second.

161. A mandibular canine differs from a maxillary


156. From the facial view, the maxillary first molar canine in which of the following?
. has its lingualroot apex in line withwhichof
A It is longer.
the following?
B. It has a less pronounced cingulum.
A C. The crown is approximately the same
Facial groove
B. Oistofacial line angle length.
C. D. It has a cusp tip more nearly centered
Distofacial cusp tip.
D. Mesiofacial cusp tip mesiodistally when viewed from the
E. facial.
Mesiodistal diameter midpoint

157. Because of the presence of a fissured groove,


cavity preparations MOST frequently need to 162. Which of the following molars MOST
be extended from the occlusal surface to the frequently have only 3 cusps?

A facial surface of maxillary molars. A Maxillary first


B. lingual surface of maxillary molars. B. Mandibular first
C. lingual surface of mandibular molars. C. MaxilJarythird
D. lingual surface of mandibular first D. Mandibular third
premolars.

163. Mesiolingual grooves can be found on which


158. On a maxillary molar, which of the following of the following permanent teeth?
ridges is formed by the union of the distal
cusp ridge of the mesiolingual cusp and the A Maxillary canines
triangular ridge of the distofacial cusp? B. Mandibular first premolars
C. Maxillary first molars
A Cuspal D. Mandibular second molars
B. Central
C. Marginal
D. Oblique
E. Transverse 164. In an ideal intercuspal relation, the oblique
ridge of the maxilJary first molar opposes
which structure of the mandibular molar?

159. What condylar movement is performed as the A The interproximal area between first and
mandible moves from a pure protrusive second
movement from maximum intercuspal B. The developmental groove between the
position to a maximum protruded position? mesiofacial and distofacial cusps of the
first
A Translation C. The developmental groove between the
B. Rotation mesiolingual and distolingual cusps of
C. Hinge the first
D. Medial and forward D. The developmental groove between the
distofacial and distal cusps of the first.

19
..
.

165. As compared with permanent teeth, the 169. Protrusive movement is produced primarily
crowns of the primary teeth are because of contracture of which of the
following muscles?
A larger. ~
B. more bulbous and constricted. A Masseter
~
C. about the same size, but more bell- B. Mylohyoid ,~

shaped cervically. C. Temporalis


D. narrower mesiodistally in comparison D. Medial pterygoid ~

with their crown length in the anterior E. Lateral pterygoid


teeth.

166. The smudge mark labeled a, b, C represents 170. Which root surface depression on a maxillary
which of the following contacts? first molar possesses a concavity that travels
'-- from the cervical third of the crown onto the

root surface? ~~<:)


~
A Lingual c (,O
C!b.
O~
B. Facial ~
c b a
~
-

C. Mesial
'-- D. Distofac:~~~ .
E. Distolin~~ ~

"'-- A Working side


B. Non-working side
C. Protrusive
D. Lateral protrusive 171. In which of the following teeth is the mesial
~
portion MOST distinctly separated from the
remainder of the occlusal table by a
~ transverse ridge?
"""'---

167. The apices of a mandibular second molar are A Primary maxillary first molar
located B. Primary mandibular first molar .~
C. Primary mandibular second molar .~

-- A posteriorly to the pterygomandibular D. Mandibular second premolar


,f'
:,1

raphe. E. Mandibular first molar ;]


;O
B. inferiorly to the mylohyoid muscle cJ

insertion. :5;1

----- c. '1:
inferiorly to the masseter muscle 71
insertion. ~

D. inferiorly to the medial pterygoid muscle 172. The concept of using a lateral checkbite .~
:A
~ insertion. record to set a respective condylar inclination "

implies which of the following?

A That the non-working side condyle has "


--------
traveled against the posterior wall of the
168. Mandibular lateral translation (Bennett fossa
movement) occurs during the B. That the working side condyle has
~
traveled down the slope of the articular
A earliest stage of lateral movement. eminence
B. c. That the non-working side condyle has
latter stage of protrusive movement.
C. middle stage of retrusive movement. moved anteriorly and medially
-- D. That the working side condyle has
D. hinge axis movement of the mandible.
E. moved toward the medial wall ofthe
ending stage of exhibiting a border
movement. glenoid fossa

'-------

20
-
j;~
t:
,;;:
173. Which of the following represents the MOST 178. On the occlusal surface of a maxillary first
'<1
common anatomic feature that complicates molar, the total number of pits is normally the
~j periodontal maintenance in a maxillary first same as found on the occlusal surface of
c,l
J premolar? which premolar?
<

A Deep concavity on the mesial surface of A Maxillary first


the tooth B. Mandibular first
8. Intermediate furcational ridge C. Maxillary second
C. Mesiolingual developmental groove D. U-type mandibular second
D. Excessive convexity of the facial root E. Y-type mandibular second

174. The lingual cusp(s) on which of the following 179. From the incisal aspect, the crown of a
mandibular posterior teeth is (are) maxillary canine normally exhibits which of the
approximately 2/3 the height of the respective following?
facial cusp(s)?
A A trapezoidal outline
A First premolar B. Symmetry between its mesial and distal
B. Second premolar portions
C. First molar c. A mesial portion that is thinner
D. Second molar faciolingually than the distal portion
E. Third molar D. A d~stalportion that displays some
concavity in its facial outline
~,
E. Less faciolingual thickness than the,
175. In the comparison of the rhomboidal and crown of a mandibular canine
heart-shaped crown outlines of maxillary
molars, the crown portion that differs MOST in
contour and size is the

A mesiofacial. f' 180. From a proximal view, which of the following


describes the crown outline on a mandibular
8. distofacial.
posterior tooth?
C. mesiolingual.
D. distolingual. A It is designed to protect against root
fracture by having the crown structure
serve as root support.
B. It is designed to allow for a minimum
176. A hypoplasia of primary teeth which is limited
amount of chewing efficiency on the
to ,the incisal thirds of incisors, to the incisal coronal surface of the tooth.
" tips of canines, and to the occlusal portions of c. It is usually rhomboidal and has a
molars, suggests which of the following?
r design flaw that' encourages cusp
fracture.
:f A Tetracycline medication in early life D.
'J': B. It is usually trapezoidal and has a
.'" Excessive fluorides in the drinking water
C. design flaw that encourages cusp
A metabolic disturbance during the fracture.
prenatal period
D. A metabolic disturbance during infancy
"

and the early childhood periods

181. Which of the following normally describes the


general crown form of canines when viewed
177. Which of the following is the first from the facial or lingual aspect?
succedaneous tooth?
A 3-sided
A B. 4-sided
Maxillary central incisor
8. Mandibular central incisor C. 5-sided
C. D. 6-sided
Maxillary lateral incisor
D. Mandibular first molar E. 7-sided

21
...
~

~
~
182. Which of the followingteeth is MOST likelyto
have a distal coronal concavity that can pose
187. As the mouth is opened widely, the-articular
disk moves in what direction in relation to the .
'.co

';
,~

special problems in matrix placement? articular eminence?

A A '-
.;.

Maxillaryfirst premolar Laterally


8. Mandibular first premolar 8. Anteriorly
C. Maxillaryfirst molar C. Posteriorly
D. Mandibular first molar D. Medially
E. Superiorly

183. The spacing between anterior teeth in a 188. The dentist instructs the patient, who has a
5-year-old child is MOST frequently caused by severed left lateral pterygoid muscle, to open
wide. The patient's mandible will move in
which direction?
A the presence of an excessive maxillary
anterior frenum. A To the left
8. the pressure from succedaneous teeth. 8. To the right
C. thumb-sucking. C. In a straight protrusive direction
"'--- D. tongue thrusting. D. In a retrusive direction
E. the growth of the dental arches.

,-~ 189. During a working movement of the mandible,


the facial cusp ridges of the maxillary first
premolar on the working side oppose 'wnich
184. In a cervical cross section, which premolar(s) of the following mandibular structures?
---- sometimes exhibit a root outline and a pulp
chamber floor outline that are both kidney- A The facial embrasure between the
shaped? canine and the first premolar
8. The distal cusp ridge of the first
~
A Maxillary first premolar and the mesial cusp ridge of
8. Mandibular first the second premolar
C. Maxillary second C. The distal cusp ridge of the second
~
D. Mandibular second premolar and the mesial cusp ridge of
E. All premolars the first molar
D. The mesiofacial groove of the first
molar.
-----
185. On the crowns of maxillary canines, which'
lobe includes the cusp tip? 190. Which of the following BEST describes the
----- Curve of Spee?
A Mesiolingual
A The spherical configuration of the .
8. Distofacial
C. Mesiofacial composite arrangement of the occlusal
~ D. Lingual surfaces and incisal edges of the teeth
E. Middlefacial of both dental arches ~
8. The facial-lingual curvature resulting
from the facial cusps being the longest
~
in the mandibular arch, and the lingual
~
:;
cusps being the longest in the maxillary
186. In protrusive movement, the mandibular arch ~
canines in a Class II occlusal relationship C. The anterior-posterior curvature of the
articulate with which of the followingmaxillary occlusal surfaces of the teeth, as seen
teeth?
in a facial view
D. The inclination of the teeth in relation to
A Canines only
- ~
8.
the verticallongaxis ofthe body
Lateral incisors only E. The composite arrangement, of the
C. Canines and lateral incisors facia! crown surface heights of contour
D. Canines and first premolars of all the teeth in any quadrant
--

22
....

0,

'>'

~. 191. Which of the followingnormallydescribesthe 196. From the facial aspect, the crown of a primary
j pulp cavity in a mesiodistal section of a canine normally has an incisal outline that
maxillary canine? exhibits
;

A It exhibits 2 pulp horns. A 2 mamelons.


8. It is widest at its incisal limit. 8. no slopes, because it is straight.
C. It is pointed at its incisal limit. C. mesioincisal and distoincisal slopes of
D. It is widest at the mid root level. equal length.
E. It is generally wider than in a D. notching due to labial developmental
faciolingual section. depressions.
E. a mesioincisal slope that is longer than
the distoincisal slope.
192. Which fiber group of the periodontal ligament
reduces the probability of forceful impaction
into the alveolus because of a blow to the 197. A primary molar lacks an identifiable
crown?
A root trunk.
A. Horizontal 8. cervical line.
8.-'- Apical C. cervical ridge.
C. Oblique D. apical foramen.
D. Transseptal E. dentinoenamel junction.
E. Gingival

198. The masticatory function of a mandibular first


193. The occlusal surface of the primary premolar is MOST similar to that of which of
mandibular second molar closely resembles the following teeth?
the occlusal surface of which of the following
mandibular teeth? A Mandibular incisor
8. Mandibular canine
A The permanent second molar
'.
C. Maxillary first premolar
8. The permanent first molar D. Mandibular second premolar
C. The second premolar E. Mandibular molar
D. The primary first molar

194. On the crowns of maxillary premolars, the 199. The greatest difficulty in removing calculus
height of contour is normally located in the from the root trunk area of molars is seen on
cervical third on which of the following which of the following surfaces?
surfaces?
A Mesial of mandibular second
A Facial 8. Distal of mandibular second
8. Lingual C. Lingual of maxillary second
C. Mesial D. Distal of maxillary first
D. Distal E. Mesial of mandibular first

195. Which of the following represents the apex of


the triangular-shaped boundary of the 200. In viewing the crown of a canine from the
.\
,~
interproximal space? incisal aspect, one normally sees each of the
following structures EXCEPT one. Which one
A Alveolar bone is this EXCEPTION?
8. Gingival tissue
C. The marginal ridges of the adjacent A Cingulum
teeth 8. Lingual fossa
D. The proximal surfaces of the adjacent C. Cervical line
teeth D. Distal cusp ridge
E. The contact area of the adjacent teeth E. Mesiofacial developmental depression

23
.,
,
RELEASED
NATIONAL BOARD DENTAL EXAMINATIONS
..
PART 1 . 1f-
*1Ii
~'r

TEST: DENTAL ANATOMY-OCCLUSION


FORM: 14
DATE: D96

.
..
Item Key:. Item..... Key Item Key:' ,Item Key.

101 A 126 B 151 A 176 C


102 D 127 D 152 D 177 B
"--" 103 D 128 E 153 C 178 E
104 D 129 C 154 C 179 D
105 B 130 A 155 E 180 C
----

106 A ,131 C, 156 A 181 C


107 B 132 D 157 B 182 C
'"'---
108 E 133 A 158 D 183 E
109 B 134 C 159 A 184 A
110 C 135 C 160 D 185 E
'"'-- ,
111 C 136 C 161 'B 186 C
112 D 137 D 162 C 187 B
~
113 E 138 D 163 B 188" A
114 A 139 D 164 D 189 B
- 115 D 140 C 165 B 190 C

116 B 141 A 166 A 191 C


- 117 A 142 D 167 B 192 C
118 A 143 D 168 A 193 B
B 144 E 169 E 194 A ,$
119
l'.
~

120 B 145 B 170 - 195 E ,

121 C 146 C 171 B 196 E


-
147 B 172 C 197 A i[
122 A ..'"
123 B 148 D 173 A 198 B
.~
124 C 149 D 174 A 199 D
125 D 150 D 175 D 200 C
.,
-

- indicates item not scored


L---

24
RELEASED EXAMINATION

PART l ANATOMIC SCIENCES (11)


MONDAY - A.M. BIOCHEMISTRY-PHYSIOLOGY (12)
DECEMBER 1998

TEST BOOKLET

DO NOT OPEN TEST BOOKLET UNTIL INSTRUCTED TO BEGIN.

1. In addition to this test booklet, you should have an answer sheet.

2. Use a No. 2 pencil on all test materials.

3. Enter your name and reference number below.

Name
Last First Middle

Reference Number

4. Enter your name and reference number on the side of the answer sheet numbered 1 to 100 (SIDE 1).
Blacken the circle that corresponds to each digit of your reference number.

5. Enter the name of this test (ANATOMIC SCIENCES) and the number of this test (11) on the answer sheet.
Blacken the circles that correspond to the test number.

6. Check to be sure you have completed each step above.

YOU ARE READY TO BEGIN. For each test item, decide which choice is correct and blacken the corresponding
circle on the answer sheet. Record only one answer for each test item; there is no penalty for selecting an
incorrect response. You may write in the test booklet; however, your score is based on the total number of
correct answers recorded on your answer sheet. You are allowed 3 112 hours to complete this test booklet.

All test materials must be returned before or immediately upon the request of the Test Administrator. Use of a
magnifjing glass is permitted. Use of other study aids is not permitted. No test items are to be copied or notes
taken. Infraction of these rules will result in your scores being voided.

COPYRIGHT 1998
JOINT COMMISSION ON NATIONAL DENTAL EXAMINATIONS
AMERICAN DENTAL ASSOCIATION
211 EAST CHICAGO AVENUE, CHICAGO, ILLINOIS 60611
1. Which of the following does a human normally 7. Submucosal glands are usually located in the
possess before birth but not after?
A. colon.
A. Fossa ovalis B. jejunum.
B. Hepatic vein C. appendix.
C. Ligamentum teres D. duodenum.
D. Ductus arteriosus E. fundus of the stomach.
E. Crista terminalis

8. The sensory innervation to the posterior third of


2. The core of a microvillus consists of which of the the tongue reflects the embryonic origin of its
following? covering epithelium from the

A. 9 + 2 arrangement of microtubules A. occipital somites.


B. 9 triplets of microtubules B. first pharyngeal (branchial) arch.
C. Intermediate filaments C. second pharyngeal (branchial) arch.
D. Vimentin filaments D. third pharyngeal (branchial) arch.
E. Microfilaments E. fourth pharyngeal (branchial) arch.

3. Which of the following organelles is the site of 9. After a right side radical mastoid operation, a
protein synthesis? patient has facial distortion and the mouth is
drawn upward to the left. He is unable to close
A. Rough endoplasmic reticulum his right eye. Saliva tends to accumulate in his
B. Membrane bound vesicles right cheek and dribble out of the corner of his
C. Sarcoplasmic reticulum mouth. Which of the following nerves MOST
D. Mitochondrion likely was damaged during the operation?
E. Golgi complex
A. Vagus
B. Facial
4. In adults, the thyroid gland's point of origin is C. Trigeminal
seen as the D. Hypoglossal
E. Glossopharyngeal
A. copula.
B. tuberculum impar.
C. sulcus terminalis.
D. palatine tonsil.
E. foramen cecum. 10. The blood-testis barrier is maintained by

A. spermatogonia.
5. Which of the following triangles is formed by the B. spermatocytes.
superior belly of the omohyoid muscle, the C. Sertoli cells.
anterior border of the sternocleidomastoid D. Leydig cells.
muscle, and the posterior belly of the digastric E. myoepithelial cells.
muscle?

A. Superclavicular
B. Submandibular
C. Digastric 11. A patient's pain from the temporomandibular
D. Muscular joint was referred to the skin over the parotid
E. Carotid region and to the side of the head. This referral
pattern is based on the distribution of which of
the following nerves?
6. Sensations from the left face and teeth are
interpreted in which of the following lobes?
A. Auriculotemporal
B. Greater auricular
A. Left frontal C. Temporal branch of the facial
B. Right frontal D. Superior alveolar
C. Left parietal E. Inferior alveolar
D. Right parietal
E. Right temporal
12. The temporomandibularjoint capsule is supplied 17. Infections or neoplasms that spread by
by several nerves. Two of these are the lymphatics from the skin of the angle of the
auriculotemporal and the mouth MOST likely pass to which of the
following lymph nodes?
A. facial.
B. buccal. A. Lingual
C. maxillary. B. Submental
D. masseteric. C. Submandibular
E. inferior alveolar. D. Anterior cervical
E. Superficial cervical

13. If the needle is advanced too far posteriorly


during an inferior alveolar block injection, 18. Preganglionic sympathetic fibers to the head
anesthesia of the mandibular teeth will NOT have their cell bodies in the
occur, because the needle has entered the
A. paravertebral ganglia.
A. parotid gland. B. cervicothoracic ganglia.
B. pterygoid space. C. superior cervical ganglia.
C. inferior alveolar canal. D. gray rami communicantes of the thoracic
D, medial pterygoid muscle. region.
E. superior pharyngeal constrictor muscle. E. intermediolateral horns of the thoracic
spinal cord.

14. Meiosis occurs in

A. ductus epididymis.
8. uriniferous tubules. 19. In the posterior mediastinum, the thoracic duct
C. seminiferous tubules. usually lies
D. stratum germinativum.
E. germinal epithelium of ovary. A. anterior to the phrenic nerves.
B. posterior to the esophagus.
C. on the anterolateral surface of trachea.
15. The cell bodies of proprioceptive nerves carrying
information from the periodontal ligaments are
located in the

A. nucleus ambiguus. 20. Which of the following veins is formed by the


B. trigeminal ganglion. union of the internal jugular and subclavian
C. spinal nucleus of V. veins?
D. main sensory nucleus of V.
E. mesencephalic nucleus of V.
A. Superior vena cava
B. Brachiocephalic
C. Pulmonary
D. Cephalic
E. Azygos

16. Which of the following are two muscles that


prevent food from entering the nasopharynx
during swallowing?
21. Which of the following is the MOST numerous
A. Mylohyoid and the palatoglossus cell type in the normal dental pulp?
B. Styloglossus and the palatoglossus
C. Tensor tympani and the stylopharyngeus
Tensor veli palatini and the levator veli A. Adipocyte
D.
palatini B. Mast cell
E. Musculus uvulae and the inferior C. Fibroblast
pharyngeal constrictor D. Macrophage
E. Odontoblast
22. The middle meningeal artery enters the cranial 28. The hepatic triad consists of which of the
vault through the following?

A. foramen ovate. A. Central vein, hepatic vein, and bile duct


€3. foramen lacerum. B. Portal vein, bile duct, and hepatic artery
C. foramen rotundum. C. Portal vein, bile canaliculi, and hepatic vein
D. foramen spinosum. D. Central vein, bile canaliculi, and hepatic
E. superior orbital fissure. artery
E. Hepatic artery, hepatic vein, and bile duct

23. Which of the following structures is


retroperitoneal?

A. Spleen 29. Which of the following represents a popular


B. Kidney theory for the force that is considered to be the
C. Stomach MOST responsible for active tooth eruption?
D. Sigmoid colon
E. Transverse colon A. Cells and fibers in the developing PDL pull
the tooth toward the oral cavity.
B. Dentin formation in the root pushes the
24. The hamular process is an extension of the tooth into the oral cavity.
C. Vascular pressure within the pulp pushes
A. ethmoid bone. the tooth intraorally.
B. palatine bone. D. Apical fundal bone formation pushes the
C. maxillary tuberosity. tooth.
D. medial pterygoid plate. E. Crestal bone formation pulls the tooth.
E. lateral pterygoid plate.

25. The muscularis externa of the esophagus in the


lower one third consists of 30. Which of the following ligaments helps resist
posterior movement of the mandibular
A. mostly skeletal muscle, but some smooth condyles?
muscle.
B. equal amounts of smooth and skeletal A. Pterygomandibular raphe
muscles. €3. Temporomandibular
C. smooth muscle only. C. Anterior cruciate
D. skeletal muscle only. D. Sphenomandibular
E. Stylomandibular
26. The dermis may be classifed as which of the
following types of connective tissue?

A. Adipose 31. The MOST commonly accepted theory for pain


B. Modified elastic reception in the dental pulp suggests that
C. Reticular connective
D. Dense regular collagenous connective
A. nerves in the dentinal tubules have
E. Dense irregular collagenous connective
receptors for pain.
B. odontoblastic processes serve as the only
27. Which of the following structures travels just receptors for pain.
posterior to the arch of the azygos vein? C. there are numerous Meissner's corpuscles
in the pulp that respond to pain stimuli.
A. Hemiazygos vein
D. nerve endings in the periodontal ligament
enter the pulp through accessory canals to
B. Right vagus nerve
provide pain reception.
C. Right phrenic nerve
E. hydrodynamic phenomena involving fluid
D. Right lymphatic duct flux in the tubules result in stimuli that
E. Greater splanchnic nerve
activate pain receptors within the pulp.
32. In a healthy temporomandibularjoint the 38. Oxyphil cells are present in which of the
articular surfaces are covered with which of the following?
following?
A. Thymus
A. Fat B. Spleen
B. Bone C. Fundic stomach
C. Hyaline cartilage D. Neurohypophysis
D. Synovial membrane E. Parathyroid gland
E. Dense fibrous connective tissue

39. Which of the following represents the MOST


33. The hyoid bone is attached by muscles or vascular portion of the articular disc of the
ligaments to the temporomandibular joint?

A. palatine bone and the soft palate. A. Anterior thick zone (band)
B. epiglottis and the cricoid cartilage. B. Posterior thick zone (band)
C. mandible, the first rib, and the clavicle. C. Intermediate thin zone (band)
D. mandible, the styloid process, and the D. Bilaminar zone
tongue.

40. The spleen, the thymus, and lymph nodes are


34. The pterygoid plexus is usually formed by veins similar in that each
that are tributaries of which of the following?
A. filters blood.
A. Maxillary vein 6. contains lymphocytes.
B. Angular vein C. has a medulla and a cortex.
C. Facial vein D. serves as a filter for tissue fluid.
D. Occipital vein E. has afferent and efferent lymphatic
E. Anterior jugular vein vessels.

35. Which of the following represents the primary 41. Which of the following represents the MOST
supinator at the radioulnar joint? common cartilage found in the embryo and in
the adult?
A. Supinator
6. Brachialis A. Fibrocartilage
C. Biceps brachii 6. Hyaline cartilage
D. Brachioradialis C. Elastic cartilage
E. Coracobrachialis D. Calcified cartilage

36. Which of the following veins are ldcated within 42. The ossicles of the ear are located in the
the bones of the calvarium?
A. utricle.
A. Carotid B. labyrinth.
B. Diploic C. inner ear.
C. Meningeal D. middle ear.
D. lnfrahyoid E. external auditory meatus.
E. Subarachnoid

43. A terminal branch of the mandibular nerve


emerges through which of the following
foramina?
37. Loss of the gag reflex suggests damage to
which of the following cranial nerves? A. Supraorbital
6. lnfraorbital
C. Stylomastoid
A. V
D. Rotundum
B. VII
E. Mental
C. IX
D. XI1
44. Bilateral contraction of which of the following 50. The thinnest portion of the bony orbit is located
muscles acts to protrude the mandible? in the

A. Lateral pterygoid A. roof.


B. Digastric 6. floor.
C. Buccinator C. medial wall.
D. Temporalis D. lateral wall.
E. Mylohyoid E. posterior wall.

45. Which of the following organs is the NEAREST 51. The nerve of the pterygoid canal contains which
to the right kidney? of the following fibers?

A. Colon A. Taste and vision


B. Spleen B. Pressure and touch sensation
C. Stomach C. Pain and temperature sensation
D Pancreas D. Sympathetic and parasympathetic
E. Duodenum E. Proprioception and touch sensation

46. Which of the following connective tissue types is 52. If there is an organ that needs to stretch and
primarily found in the fetus, but not in the adult? return to its original shape, then which of the
following epithelia would MOST likely be
A. Loose associated with it?
6. Mesenchyme
C. Areolar A. Transitional
D. Dense regular B. Simple cuboidal
E. Dense irregular C. Simple columnar
D. Simple squamous
E. Pseudostratified columnar
47. Antidiuretic hormone is PRODUCED by cells
that reside in which of the following structures,
cells or areas? 53. Each of the following laminae help to form a
tooth EXCEPT one. Which one is this
A. Pars nervosa EXCEPTION?
6. Pars intermedia
C. Basophils of the pars distalis A. Vestibular
D. Acidophils of the pars distalis B. Successional
E. Supraoptic nucleus of the hypothalamus C. Dental

48. Which of the following structures directly 54. Which of the following anterior primary rami form
develops from the cells remaining in the the medial cord of the brachial plexus?
remnants of the preovulatory follicle after
ovulation? A. C-4 and C-5
B. C-6 and C-7
A. Theca interna C. C-5, C-6, and C-7
B. Corpus luteum D. C-8 and T-1
C. Corpus albicans E. T-2 and T-3
D. Atretic follicle
E. Interstitial gland
55. The only purely serous minor salivary gland is
located in the lamina propria of the
49. The heart is contained in which mediastinum?
A. tongue.
A. Middle B. hard palate.
B. Superior C. soft palate.
C. Anterior D. buccal mucosa.
D. Posterior E. attached gingiva.
56. The maxillary artery is MOST often a branch of 61. Which of the following cells forms a Howship's
which of the following arteries? lacuna?

A. Facial A. Osteocyte
B. External carotid B. Osteoblast
C. Internal carotid C. Osteoclast
D. Common carotid D. Chondrocyte
E. Superficial temporal E. Chondroblast

57. Which of the following structures runs directly 62. Which portion of dentin is the MOST highly
adjacent to, and parallel with, the median nerve mineralized?
in the arm?
A. lntratubular (peritubular)
A. Ulnar artery B. Intertubular
B. Radial artery C. lnterglobular
C. Cephalic vein D. Mantle
D. Brachial artery
E. Medial brachial cutaneous nerve
63. Which of the following is the posterior limit of the
vestibular side of the cheek?

A. Retromolar pad
58. Postganglionic fibers from the superior cervical B. Sulcus terminalis
ganglion innervate which of the following C. Pterygomandibular raphe
muscles? D. Stylomandibular ligament
E. Sphenomandibular ligament
A. Ciliary
B. Inferior oblique 64. When a developing carious lesion reaches the
C. Dilator pupillae dentinoenamel junction, this will often initiate the
D. Sphincter pupillae formation of which of the following at the pulp
E. Levator palpebre superioris interface of the dentin?

A. Mantle dentin
B. Primary dentin
C. False pulp stones
59. Connective tissue proper is characterized as D. lnterglobular dentin
having E. Tertiary (reparative) dentin

A. poor vascularization.
B. poor reparative ability. 65. Which nerve enters the pharynx in the gap
C. sensitivity as its main function. between the superior and middle constrictors?
D. more intercellular material than cells.
E. mast cells as the predominant cell type. A. Lingual
B. Hypoglossal
C. Glossopharyngeal
D. Inferior laryngeal
E. Superior laryngeal

Which of the following represents a segment of


the bronchial tree that is less than 1 mm in
diameter and lined by simple columnar ciliated 66. Certain nerve trunks are composed of several
epithelium with no cartilage plates in the wall? fascicles. Which of the following entirely
encloses these trunks?
A. Bronchiole
B. Alveolus A. Epineurium
C. Bronchus B. Endoneurium
D. Alveolar sac C. Perineurium
E. Bronchial septum
67. Pain in the temporomandibular joint would be 72. Which of the following represents the main
perceived by endings of the arterial supply to the brain?

A. auriculotemporal nerve in the capsule and A. Internaljugular


periphery of the disc. B. lnternal carotid
B. lateral pterygoid nerve in the capsule, disc C. External carotid
and retrodiscal pad. D. Middle meningeal
C. auriculotemporal nerve throughout the
meniscus.
D. maxillary branch of the trigeminal in the 73. Which of the following is the nerve that supplies
retrodiscal pad and capsule. the mimetic muscles?
E. inferior alveolar nerve.
A. Vagus
B. Facial
C. Trigeminal
D. Hypoglossal
68. Which of the following lymphoid aggregations is E. Spinal accessory
(are) the MOST likely to be associated with
pseudostratified columnar ciliated epithelium?
74. Which of the following is a pulpal change
A. Palatine tonsil associated with increasing age?
B. Lingual tonsil
C. Pharyngeal tonsil A. Increased cellularity
D. Peyer's patches B. Increased collagenous components
C. Development of more extensive
subodontoblastic nerve plexus
D. Development of more extensive
subodontoblastic capillary plexus
E. Development of more extensive lymphatic
69. Which of the following intercellular junctions plexus
provides the GREATEST resistance to
substances attempting to move between cells?

A. Gap junction
B. Zonula occludens
C. Zonula adherens
75. The pyramids contain
D. Macula adherens
E. Fascia adherens A. pain fibers.
B. sensor fibers.
C. upper motor neuron fibers only.
70. How is a large vein different from its companion D. lower motor neuron fibers only.
artery? The vein has E. upper and lower motor neuron fibers.

A. a smaller lumen.
B. a less extensive vasa vasorum.
C. a more prominent tunica media.
D. an indistinct endothelial lining. 76. The prochordal plate consists of
E. a more prominent tunica adventitia.
A. embryonic endoderm, mesoderm, and
ectoderm.
71. Fiber tracts passing from the thalamus to the B. a circular area anterior to the notochord in
cortex are found in which of the following? which the endoderm is fused to the
embryonic mesoderm.
A. Corpus callosum C. the cloaca1 membrane and the overlying
B. Internal capsule amnion.
C. Medial lemniscus D. endoderm of the roof of the yolk sac and
D. Lateral lemniscus embryonic ectoderm.
E. Anterior commissure E. the cardiogenic mesoderm and the
neurectoderm.
77. Which of the following structures are found in 82. Accessory root canals can be formed by
the infratemporalfossa?
A. a break in the epithelial diaphragm prior to
A. Temporalis muscle and parotid gland dentin formation.
B. Masseter and lateral pterygoid muscles B. a break in the epithelial root sheath
C. Superficial temporal artery and parotid (Hertwig) prior to dentin formation.
lymphatic nodes C. adherence of the epithelial root sheath
D. Mandibular division of V and chorda (Hertwig) to the dentinal surface.
tympani branch of VII D. an epithelial rest that lies in contact with
the dentin of the root.

78. Which of the following glands is purely mucous?


83. In the mid-palatal raphe region of the hard
A. Parotid palate, which of the following represents the
B. Palatine layer of the epithelium of the oral mucosa which
C. Sublingual is deep to the stratum spinosum?
D. von Ebnet's
E. Submandibular A. Stratum lucidum
B. Stratum basale
C. Lamina propria
D. Stratum granulosum
E. Stratum corneum
79. A patient bites the tip of his tongue. The pain
that ensues is carried by way of which of the
following cranial nerves?

A. Vagus, X 84. Information from the hypothalamus is carried to


B. Facial, VII the anterior pituitary (pars distalis) by the
C. Trigeminal, V
D. Hypoglossal, XI1 A. gamma efferent system.
E. Glossopharyngeal, IX B. hypothalamic radiation.
C. hypothalamo-hypophyseal tract.
D. hypothalamic association fibers.
E. hypothalamo-hypophyseal portal system.

80. To expose the submandibular duct by an


intraoral approach, one must cut through which
of the following? 85. On its way to the brain, the vertebral artery
passes through which of the following?
A. Mucous membrane only
B. Mucous membrane and the genioglossus
A. Foramen magnum
muscle
B. Foramen lacerum
C. Mucous membrane and the mylohyoid
C. Jugular foramen
muscle
Mucous membrane and the geniohyoid
D. Foramen spinosum
D.
muscle
E. Mucous membrane and the anterior
digastric muscle

86. In addition to the common carotid arteries, which


81. The submandibular ganglion is associated with of the following blood vessels provides
which of the following nerves? circulation to the brain?

A. Inferior alveolar A. Vertebral artery


B. Glossopharyngeal B. Thyrocervical trunk
C. Maxillary C. Costacervical trunk
D. Mylohyoid D. Superior thyroid artery
E. Lingual E. Internal thoracic artery
87. On a comparative basis, bone and cellular 92. Which of the following bones is formed primarily
cementum are similar in that both by intramembranous ossification?

A. are difficult to resorb, heal well, and A. Femur


contain cement lines. B. Tibia
B. contain concentric patterns, have C. Stapes
Volkmann's canals, and can be cancellous. D. Humerus
C. are deposited throughout life, acquire E. Mandible
blood vessels during aging, and contain
similar collagenous fibers.
D. contain cells in lacunae with canaliculi that
extend primarily toward the nutritional
source. 93. Gnarled enamel is MOSTfrequently found

88. Which of the following cranial nerves supplies A. in cusps.


B. near the cervical line.
the muscles derived from the first pair of
C. around pits and fissures.
branchial arches?
D. adjacent to contact areas.
E. equally on all surfaces of the crown.
A. Vagus
B. Facial
C. Trigeminal
D. Hypoglossal 94. An oblique facial cleft follows the li
E. Glossopharyngeal of

ry processes.
A.
B.
C.
&Y""-
lateral nasal an
medial nasal
medial an
axillary processes.
al nasal processes.
89. Cranial nerve V emerges from the D maxJfk?&nd mandibular processes.
E. 11% asal and mandibular processes.
A. pons.
B. midbrain.
C. diencephalon.
D. medulla oblongata.
E. cerebral peduncles.
95. A patient bleeds from the anterior septa1 region
of his nose. A break in the vessel walls has
caused this bleeding. These vessels originate
90. Which of the following structures may be found from which of the following arteries?
within the posterior mediastinum?
A. Angular
A. Heart B. Posterior superior alveolar
B. Phrenic nerve C. Sphenopalatine
C. Thoracic duct D. Greater palatine
D. Arch of the aorta
E. Recurrent laryngeal nerve

96. Maturation of enamel is characterized by a


percentage increase in inorganic content and a
91. Which group of fibers of the periodontal ligament percentage
offer the MOST resistance to movement of the
tooth in an apical direction? A. increase in water.
B. increase in organic content.
A. Alveolar crest C. decrease in water and decrease in organic
B. lnterradicular content.
C. Horizontal D. decrease in water and increase in organic
D. Oblique content.
E. Apical E. increase in water and decrease in organic
content.
97. Para-keratinized oral mucosa is often found on 100. In the photomicrograph below of a ground
which of the following? section of a tooth viewed by transmitted light,
some dentinal tubules appear black because
A. Oral surface of the soft palate they
Ventral surface of the tongue
Skin surface of the lips
Floor of the mouth
Attached gingiva

98. Which of the following represents the major


fibrillar component of mature dentin? A. are filled with blood.
B. are filled with minerals.
A. Elastin C. are filled with bacteria.
B. Reticulin D. have become filled with air during
C. Type I collagen sectioning.
D. Type II collagen E. contain deeply stained odontoblastic
E. Type Ill collagen processes.

99. Each of the following terms applies to a


discussion of enamel structure EXCEPT one.
Which one is this EXCEPTION?

A. Prisms
B. Perikymata
C. Striae of Retzius
D. Contour lines of Owen
E. lnterprismatic substance
NATIONAL BOARD DENTAL EXAMINATIONS
PART 1

TEST: ANATOMIC SCIENCES


FORM: 11 (All)
DATE: D98

Item Key, Item Key Item Key Item Key

1 D 26 E 51 D 76 D
2 E 27 B 52 A 77 D
3 A 28 B 53 A 78 B
4 E 29 A 54 D 79 C
5 E 30 B 55 A 80 A

6 D 31 E 56 B 81 E
7 D 32 E 57 D 82 B
8 D 33 D 58 C 83 B
9 B 34 A 59 D 84 E
10 C 35 C 60 A 85 A

11 A 36 B 61 C 86 A
12 D 37 C 62 A 87 D
13 A 38 E 63 C 88 C
14 C 39 D 64 E 89 A
15 E 40 B 65 C 90 C

16 D 41 B 66 A 91 D
17 C 42 D 67 A 92 E
18 E 43 E 68 C 93 A
19 B 44 A 69 B 94 *
20 B 45 A 70 E 95 C

21 C 46 B 71 B 96 C
22 D 47 E 72 B 97 E
23 B 48 B 73 B 98 C
24 D 49 A 74 B 99 D
25 C 50 C 75 C 100 D
BIOCHEMISTRY-PHYSIOLOGY (12)

BEFORE PROCEEDING

1. Turn your answer sheet over to the side numbered 101 - 200 (SIDE 2).

2. Enter your name on the second side of the answer sheet.

3. Enter the name of this test (BIOCHEMISTRY-PHYSIOLOGY) and the number of this
test (12) on the answer sheet. Blacken the circles that correspond to the test number.

4. Check to be sure you have completed each step above.

5. Proceed with the examination.


101. Which of the following substances, under 106. GABA increases the permeability of
physiologic conditions, is essential to the postsynaptic membranes to which of the
transformation of G-actin to F-actin? following ions?

A. ATP A. Sodium
B. Chloride B. Calcium
C. Manganese C. Chloride
D. Pyrophosphate D. Magnesium
E. Creatine phosphate E. Potassium

102. Chondroitin sulfate and hyaluronic acid are 107. The polymerase chain reaction is MOST
characterized by which of the following? useful for which of the following?

A. Cell walls of caries-related streptococci A. Preparing enzymes that synthesize


decomposition end products of oral nucleic acids
lactobacilli B. Isolating the genome of an organism
B. Byproducts of carbohydrate metabolism C. Amplifying a specific DNA sequence
by oral streptococci D. Separating polyclonal antibodies
C. Enzymes which break down gram- E. Synthesizing RNA from DNA
positive cell walls
D. Components of extracellular matrix
108. Each of the following segments of the
gastrointestinaltract consists of smooth
muscle under autonomic nervous control
EXCEPT one. Which one is this EXCEPTION?
103. Which of the following represents the normal
clearance of glucose? A. Rectum
B. Internal anal sphincter
A. 0 mglmin C. Antrum of the stomach
D. Upper esophagus
B. 50 mglmin
C. 100 mglmin E. Gastro-esophageal sphincter
D. 180 mglmin
E. 350 mglmin

109. The movement of ~ a + across


+ a membrane is
characterized by which of the following?
104. Which of the following BEST explains the
primary action of antidiuretic hormone? A. A facilitated diffusion
B. An example of a cotransport system
A. It decreases the activity of the Na-K C. In many cell membranes involves a
pump in the distal tubule. ~ a + -+~ a countertransport
+ system
B. It increases the Hz0 permeability of the D. Maintains ~ a + concentration
+ very much
collecting ducts and the distal tubules. higher in the cell than in the extracellular
C. It decreases the pore size of the distal fluid
tubules and the collecting ducts.
D. It decreases the glomerular filtration rate.
E. It inhibits the action of glutaminase. 110. The enzyme catalyzing the rate-controlling
step in the de novo synthesis of fatty acids is
regulated allosterically by the positive
105. Which of the following takes place as modulator
proinsulin is converted to insulin?
A. ATP.
A. Disulfide bonds are formed. B. NADPH.
B. Disulfide bonds are broken. C. citrate.
C. The polypeptide chain is lengthened. D. cyclic AMP.
D. A segment of the polypeptide chain is E. oxaloacetate.
removed.
111. Which of the following is required for 117. As DNA is denatured, each of the following
vitamin Dg synthesis? events takes place EXCEPT one. Which event
is this EXCEPTION?
A. Decarboxylation in the liver
B. UV activation of precursors in skin A. Total G-C content of total DNA
C. Metabolism by gut bacteria increasing
D. Deamination in the kidney B. UV light absorption increasing
C. Complementary strands becoming
random coils
2 . The maximal frequency of impulses that can D. Base stacking becoming disrupted
be carried by a nerve fiber is limited by which E. Hydrogen bonds breaking
of the following?

A. Intensity of the stimulus


8 Each of the following is a mechanism in
B. humans helping to raise body temperature
Diameter of the nerve fiber
C. Duration of the absolute refractory period EXCEPT one. Which one is this
D. Duration of the relative refractory period EXCEPTION?

A. Shivering
113. Which of the following represents the normal B. ATP hydrolysis
substrate of thrombin? C. Exothermic reactions
D. Peripheral vasodilation
A. Fibrin E. Increased thyroxine release
B. Thrombospondin
C. Prothrombin
D. Thromboplastin
119. Which of the following types of bonds link
E. Fibrinogen amino acid residues to form proteins?

A. Amide
114. A sustained, severe carbohydrate deficiency B. Anomeric
in the diet will result in which of the following? C. Epimeric
D. Hydrogen
A. Ketoacidosis E. Glycosidic
B. Severe metabolic alkalosis
C. A deficiency in prostaglandinformation
D. An inability to synthesize ascorbic acid

120. In skeletal muscle, excitation-contraction


coupling is mediated by which of the
following?
115. The anticaries effect of fluoride is LEAST
related to which of the following? A. Binding of calcium and calmodulin
B. Efflux of potassium from transverse
A. Ion exchange tubules
B. Decreased solubility C. A sudden rise in permeability to chloride
C. Facilitation of remineralization ions
D. Its presence during enamel formation D. Release of acetylcholinesterase from
E. Activation of plaque polysaccharide nerve terminals
hydrolysis E. Release of calcium from the
sarcoplasmic reticulum

116. Which of the following represents a


polyunsaturated fatty acid that is commonly 121. A mucin is which of the following types of
found in animal cell membranes? proteins?

A. Oleic A. Simple protein


B. Lactic B. Phosphoprotein
C. Sialic C. Chromoprotein
D. Stearic D. Nucleoprotein
E. Linoleic E. Glycoprotein
122. The tricarboxylic acid cycle is initiated by the 128. Under strict anaerobic conditions, the
condensation of which of the following two catabolism of one glucose molecule would
molecules? yield a net of

A. Pyruvate and malate A. 2 ATP and 2 lactic acid molecules.


B. NAD' and oxaloacetate B. 4 ATP and 2 lactic acid molecules.
C. 2 ATP and 2 pyruvic acid molecules.
C. NAD' and oxalosuccinate D. 4 ATP and 2 pyruvic acid molecules.
D. Acetyl coenzyme A and oxaloacetate
E. Acetyl coenzyme A and oxalosuccinate
129. Which of the following is LEAST descriptive of
lipids?
123. Compared to skeletal muscle contraction, A. Nonpolar
smooth muscle contraction B. Carbon-containing
C. Amphipathic
A. requires more energy for a given tension. D. Hydrophilic
B. occurs several times more rapidly.
C. can be maintained for a longer time.
D. is unaffected by the autonomic nervous
system.
130. The ability to concentrate urine varies among
124. Androgens are produced in the testis and animal species. The maximum urine
concentration that can be produced by an
animal is MOST closely related to which of the
A. adrenal cortex. following?
B. thyroid.
C. adrenal medulla.
D. pituitary. A. Renal blood flow
E. hypothalamus. B. Total number of nephrons
C. Glomerular filtration rate
D. Length of the loop of Henle
125. What thermodynamic parameter is a measure E. Diameter of the distal tubule
of randomness or disorder in a system?

A. Entropy
B. Enthalpy
C. Free energy 131. Each of the following appears in the
D. Potential energy glomerular filtrate in concentrations
E. Activation energy approximately equal to those in plasma
EXCEPT one. Which one is this
EXCEPTION?
126. A number of catabolic pathways are
allosterically inhibited by an increase in the A. Urea
concentration of which of the following? B. Glucose
C. Amino acids
A. ADP D. Steroid hormones
B. AMP E. Plasma electrolytes
C. ATP
D. NAD+
132. If an axonal membrane transiently becomes
E. Pyruvate
very permeable to Na+ ions, then the
membrane potential of the cell will approach
127. Both systemic and pulmonary circulations
have the same A. -70 mV.
B. -60 mV.
A. pulse pressure. C. -50 mV.
B. total capacitance. D. OmV.
C. diastolic pressure. E. +60 mV.
D. resistance.
E. flow rate.
133. The clinical symptoms of dermatitis, diarrhea, 138. The stimulating effect of low arterial 0 2
and dementia suggest a deficiency in which of tension on respiration is usually produced
the following vitamins? through an effect on the

A. Retinol A. carotid bodies.


B. Riboflavin B. carotid sinuses.
C. Tocopherol C. expiratory center.
D. Ascorbic acid D. inspiratory center.
E. Niacin E. alveolar nerve endings.

139. Most endogenous cholesterol in the liver is


usually converted into which of the following?
134. Streptomycin is an antibiotic which inhibits the
process of A. Glucose
B. Steroids
A. translation in eukaryotes. C. Cholic acid
B. translation in prokaryotes. D. Oxaloacetate
C. transcription in eukaryotes. E. Ketone bodies
D. transcription in prokaryotes.
E. DNA replication in prokaryotes.

140. Which of the following could be beneficial in


reducing edema in the arms of women who
135. Insulin INCREASES the activity of which of have had radical mastectomy with removal of
the following? axial lymph nodes?

A. Enolase A. Hypertensiveagents
B. Phosphorylase a B. Avoidance of all diuretic agents
C. Phosphofructokinase C. Increase interstitial oncotic pressure
D. Glucose-6-phosphatase D. Decrease interstitial hydrostatic pressure
E. Fructose 1-6 diphosphatase E. Administration of a plasma volume
expander

136. lncreased parasympathetic activity results in


141. Which of the following substances is LEAST
polar?
A. decreased salivary secretion.
B. increased cardiac contractility.
C. decreased gastric motility and tone. A. Ethanol
D. increased bronchiolar smooth muscle B. Cholesterol
contraction. C. Palmitic acid
D. Glycocholic acid

137. A patient who has emphysema is MOST likely


to exhibit which of the following changes in
functional residual capacity (FRC) and
compliance? 142. Which of the following molecules would likely
form a micelle when mixed with water and
FRC Compliance agitated?

A. Increased Increased A. Serine


B. Decreased No change B. Glycerol
C. lncreased No change C. Phospholipid
D. Decreased lncreased D. Triglyceride
E. Decreased Decreased
143. Which of the following is MOST likely to result 148, In respiratory acidosis, arterial C02 content
from increased vagal activity? and pH become abnormal. Which of the
following BEST describes their respective
A. lncreased heart rate changes?
B. lncreased stroke volume
C. lncreased cardiac output
D. Decreased cardiac oxygen consumption
E. Decreased transit time through the AV A. Increases Increases
node B. Increases Decreases
C. Decreases Increases
D. Decreases Decreases
144. Which of the following noncollagenous protein
components BEST characterizes dentin
matrix?

A. Laminin
B. Vimentin
149. Which of the following is the major protein
C. Phosphophoryn component of cementum?
D. Osteonectin
E. Fibronectin A. Elastin
B. Keratin
C. Collagen
D. Amelogenin
E. Osteonectin
2+.
145. Some carbohydrates convert Cu Ions to CU+
ions. This property is related to their ability to
act as
150. Which of the following do elastin and collagen
A. a reducing agent. have in common?
B. an oxidizing agent.
C. both a reducing agent and an oxidizing
agent. A. Easily stretched
D. neither a reducing agent nor an oxidizing B. Absence of proline
agent. C. Disulfide crosslinking
D. Triple helix structure
E. About one-third glycine
146. Coenzyme A participates in

A. formylation.
B. protein synthesis.
C. methionine activation. 151. The plasma osmolality decreases after
D. activation of carboxyl groups. infusion of

A. aldosterone.
B. vasopressin (ADH).
C. angiotensin II.
147. Which of the following is the FIRST step in the D. parathyroid hormone.
catabolism of many amino acids? E. isotonic saline solution.

A. Formation of a dipeptide with glutamate 152. Which of the following features distinguishes
B. Conjugation of the alpha amino to
active transport from facilitated diffusion?
glucuronate
C.. Transamination of the alpha amino to a
keto acid A. Specificity
D. Conjugation of the alpha carboxyl group B. Carrier-mediated
to glucuronate C. Requires metabolic energy
E. Decarboxylation of the alpha carboxyl D. Presence of a transport maximum (Tm)
group to form a primary amine
153. Each of the following secretes H C a - into the 158. Which of the following changes in lung
gastrointestinaltract EXCEPT one. Which compliance and vital capacity would be
one is this EXCEPTION? observed in the absence of pulmonary
surfactant?
A. Chief cells
B. Colon mucosa Lung Compliance Vital Capacity
C. Salivary glands
D. Stomach mucosa A. Increase Increase
B. Increase Decrease
C. Increase No change
154. The chemical energy generated by D. Decrease Increase
mitochondria1electron transport results from E. Decrease Decrease
which of the following?

A. Excess H+ in the matrix


B. An H+ gradient across the inner 159. Which of the following represents a decrease
membrane in the frequency of action potentials in an
C. The formation of thioesters in the matrix afferent neuron despite a constant stimulus
D. A conformational change in the inner intensity?
membrane
A. Refraction
B. Potentiation
155. Which of the following is necessaryfor C. Perception
de novo synthesis of cholesterol? D. Adaptation
E. Accommodation
A. NA+
B. NADH 160. Which of the following is a typical finding for a
C. NADP+ normal 23-year-old man?
D. NADPH
E. FADH2 A. Hematocrit of 45%
B. Venous blood with pH of 7.2
3
C. White blood cell count of 10,000/mm
D. Red blood cell count of 7,000,000/mm3
E. Pulse pressure equivalent to 80 mm Hg
156. The function of which of the following types of
nucleic acid is to activate and select specific
amino acids for protein synthesis? 161. Earaches may develop as a result of blockage
of the eustachian tube because

A. of anvil misalignment.
B. the ossicle can no longer vibrate.
C. fluid in the middle ear cannot escape.
D. pressure on the round window distorts
the basement membrane.
E. pressure in the middle ear is not
equalized with atmospheric pressure.

157. Which of the following is a pyrimidine base


that is present in RNA but is NOT present in 162. The activities of each of
DNA? one contribute to calculus form
one is this EXCEPTION?
A. Uracil
B. Guanine
C. Thymine
D. Adenine
E. Cytosine
163. Which of the following stimulates vagal nerve 168. A diminished oxygen tension in the
endings in the lung parenchyma and inhibits myocardium causes immediate
inspiration?
A, vasoconstriction of coronary vessels.
A. Decreased arterial pH B. vasodilation of coronary vessels.
B. Expansion of the lungs C. stimulation of chemoreceptors.
C. Decreased alveolar 0 2 tension D. inhibition of chemoreceptors.
D. lncreased alveolar C02 tension

164. Plasmid vectors suitable for cloning have


which of the following characteristics? 169. In relative insulin insufficiency, acetyl CoA is
usually channeled into
A. Must be able to replicate synchronously
with the host chromosome A. ketone-body formation.
B. Several unique recognition sequences B. cholesterol synthesis.
for one restriction enzyme C. fatty-acid synthesis.
C. Two genes conferring resistance to D. gluconeogenesis.
different antibiotics E. the Krebs cycle.
D. Large size to facilitate plasmid's entry
into cells
170. After prolonged acidosis, which of the
following represents the nitrogenous product
165. Right-sided lesions of the spinal cord result in that is excreted in high amounts in the urine?
loss of which of the following (below the level
of the lesion)? A. Urea
B. Ammonia
A. Motor activity and pain and temperature C. Uric acid
sensations on the same side D. Creatinine
B. Motor activity on the same side and pain E. Aspartic acid
and temperature sensations on the
opposite side
C. Motor activity and pain and temperature
sensations on the opposite side
D. Motor activity on the opposite side and 171. Which of the following are the immediate
pain and temperature sensations on the effects of calcitonin on serum levels of calcium
same side and phosphate?

Serum calcium Serum phosphate


166. If the molar percentage of A (adenine) in a
native DNA specimen is 22%, then what is the A. Increased Increased
molar content of G (guanine)? B. Increased Decreased
C. Decreased Decreased
D. No change Increased
E. Decreased No change

167. 172. Atoms are isotopes of each other only if


During an isotonic contraction, which of the
following bands or structures do NOT change
in width or length? A. their nuclei contain the same number of
neutrons.
B. their atomic numbers are the same, but
A. A band
their mass numbers differ.
B. I band their mass numbers are the same, but
C.
C. Muscle sarcomeres their atomic numbers differ.
D. Two consecutive Z lines
D. one is a beta emitter, but the other is an
E. Series elastic elements
alpha emitter.
173. Sodium fluoride inhibits glycolysis by affecting 178. The Henderson-Hasselbalch equation shows
which of the following? that

A. Amylase A. dilution of a buffer increases its pH.


B. Enolase B. pH = pka when an acid is 0.1 N.
C. Phosphatase
D. Phosphorylase C. pH = pka when an acid is half
neutralized.
D. pH is independent of the dissociation
constant of the acid.

174. Hyperventilation alters the acid-base balance 179. Triglyceride absorbed into the lymphatic
of arterial blood by system is transported to the liver as which of
the following?
A. increasing C02 and increasing pH.
B. increasing C02 and decreasing pH. A. Very low density lipoprotein
B. Low density lipoprotein
C. decreasing C02 and decreasing pH. C. Chylomicrons
D. decreasing C02 and increasing pH. D. Liposomes
E. Micelles

180. Blocking the hypothalamic-hypophyseal


175. Which of the following collagen synthesis venous portal system increases the secretion
post-translational events occurs of which of the following?
extracellularly?
A. Prolactin
A. Glycosylation B. Oxytocin
B. Hydroxylation C. ACTH
C. Intermolecular cross-linkage D. TSH
D. Disulfide bond formation
181. Which of the following represents the plasma
protein that transports ferrous iron?

A. Ferritin
176. If the pH becomes lower than the isoelectric
6. Myoglobin
point of a protein, then how will the protein C. Hemoglobin
respond in an electrophoreticsystem? It will D. Transferrin
E. Cytochrome-c
A. become denatured.
B. migrate to the negative pole.
C. migrate to the positive pole. 182. Which of the following sweeteners is
D. remain stationary and unchanged. non-nutritive as well as non-cariogenic?
E. separate into its different monomeric
forms. A. D-fructose
B. Saccharin
C. Galactose
D. Sorbitol
E. Maltose
177. The level of nonprotein nitrogen in the blood is
due principally to the level of which of the
following? 183. Epinephrine causes an elevation in CAMP
levels in muscle cells which in turn activate
A. Urea
6. Ammonia A. ATPase.
C. Creatine B. adenyl cyclase.
D. Arginine C. glycogen synthetase.
E. Uric acid D. glycogen phosphorylase.
E. glycogen phosphorylase phosphatase.
184. Acromegaly is due to an excessive production 189. Pieces of dog parotid gland and pancreas are
of which of the following? transplanted under the skin of the donor
animal. After re-establishment of circulation,
A. Thyrotropin ingestion of food will result in secretion from
B. . Gonadotropin which of the following?
C. Somatotropin
D. Adrenocorticotropin A. Parotid and pancreatic transplants
B. The fragment of the pancreas only
C. The fragment of the parotid only
185. Intravenous injections of KC1 solution would D. Neither fragment
increase the secretion of which of the
following?
190. Which of the following conditions is indicated
A. lnsulin during isovolumetric ventricular contraction of
B. Cortisol the heart?
C. Calcitonin
D. Aldosterone A. All valves are open.
E. Parathyroid hormone B. All valves are closed.
C. The aortic and pulmonary valves are
open and the mitral and tricuspid valves
186. Frank-Starling's law of the heart states that the are closed.
D. The aortic and pulmonary valves are
A. initial length of cardiac muscle fibers closed and the mitral and tricuspid
affects the strength of contraction. valves are open.
B. strength of contraction depends on the
strength of the ventricular muscle.
C. heart rate depends directly on thickness
of the ventricle.
D. strength of contraction depends on the 191. Which of the following controls the excitability
strength of stimulus. of the muscle spindle?

A. Load on the muscle


B. Gamma efferent system
C. Alpha efferent discharge
187. When arterial pressure increases, D. Length of the extrafusal fibers
pressoreceptors discharge and

A. increase cardiac rate and strength of


contraction.
B. cause vasoconstriction throughout the 192. In the denervated heart, adjustments to
peripheral circulatory system. increased workload are mediated by
C. excite sympathetic nerves and inhibit mechanisms associated with
parasympathetic nerves.
D. inhibit tonic activity of sympathetic A. increased end systolic volume.
nerves and excite parasympathetic B. decreased end systolicvolume.
nerves. C. decreased end diastolic volume.
D. increased end diastolic volume.

188. The rate of diffusion across the alveolar wall is


inversely proportional to which of the 193. Which of the following is secreted more during
following? the absorptive state than during the
postabsorptive state?
A. The surface area for gaseous exchange
B. The thickness of the alveolar wall A. lnsulin
C. The difference in the partial pressures of B. Glucagon
the gas C. Cortisol
D. The solubility of the gas D. Thyroxine
E. Epinephrine
194. Bulging veins in the neck result from which of 199. The largest amount of glomerular filtrate is
the following? reabsorbed in the

A. Pulmonary edema A. proximal convoluted tubule.


B. Hemolytic anemia B. descending loop of Henle.
C. Systemic hypotension C. ascending loop of Henle.
D. Congestive heart failure D. distal convoluted tubule.
E. Intermittent claudication E. collecting duct.

195. A parathyroidectomized animal will exhibit 200. How many anomeric carbons are present in a
which levels of calcium and phosphate ion fructose molecule?
concentration in the plasma?

Calcium Phosphate ion


concentration

A. Low High
B. Normal Low
C. High Low
D. Normal Normal
E. Low Low

196. Which term of the Michaelis-Menten equation,

can be determined from the y intercept of a


Lineweaver-Burk plot?

197. Absence of which of the following blood


enzymes drastically reduces blood CO2
carrying capacity?

A. Carbonic anhydrase
B. Alkaline phosphatase
C. Pyruvate carboxykinase
D. Histidine decarboxylase
E. Serum glutamic-oxaloacetate
transaminase

198. Glucosuria with hyperglycemia usually occurs


in which of the following?

A. Pellagra
B. Addison's disease
C. Diabetes mellitus
D. Diabetes insipidus
E. Parkinson's disease
NATIONAL BOARD DENTAL EXAMINATIONS
PART 1

TEST: BIOCHEMISTRY-PHYSIOLOGY
FORM: 12 (A12)
DATE: D98

Item Key Item Key Item Key Item Key

101 A 126 C 151 B 176 B


102 D 127 E 152 C 177 A
103 A 128 A 153 A 178 C
104 B 129 D 154 B 179 C
105 D 130 D 155 D 180 A

106 C 131 D 156 D 181 D


107 C 132 E 157 A 182 B
108 D 133 E 158 E 183 D
109 C 134 B 159 D 184 C
110 C 135 C 160 A 185 D

111 B 136 D 161 E 186 A


112 C 137 A 162 * 187 D
113 E 138 A 163 B 188 B
114 A 139 C 164 C 189 B
115 E 140 E 165 B 190 B

116 E 141 B 166 B 191 B


117 A 142 C 167 A 192 D
118 D 143 D 168 B 193 A
119 A 144 C 169 A 194 D
120 E 145 A 170 B 195 A

121 E 146 D 171 C 196 B


122 D 147 C 172 B 197 A
123 C 148 B 173 B 198 C
124 A 149 C 174 D 199 A
125 A 150 E 175 C 200 B
DENTAL ANATOMY AND OCCLUSION (14)

BEFORE PROCEEDING

1. Turn your answer sheet over to the side numbered 101 - 200 (SIDE2).
2. Enter your name on the second side of the answer sheet.

3. Enter the name of this test (DENTAL ANATOMY and OCCLUSION)and the number
of this test (14) on the answer sheet. Blacken the circles that correspond to the test
number.

4. Check to be sure you have completed each step above.

5. Proceed with the examination.


All test items refer to permanent teeth unless 105. Through which of the following grooves of the
"primary" is specified. mandibular first molar does the maxillary
mesiolingual cusp pass in a lateral excursive
All test items relating t o occlusion refer t o Class I movement on the working side?
canine and molar relationship unless otherwise
specified. Tenns such as "normal" or "ideaIJ'are A. Facial
synonymous with the above definition. B. Lingual
C. Central
D. Distofacial

106. In an ideal occlusion, the facial cusps of the


101. Which of the following are the LASTprimaly posterior mandibular teeth oppose which
teeth to erupt? landmark of the maxillary dentition?

A. Maxillary canines A. Facio-occlusalline


B. Mandibular canines B. Central fossa line
C. Mandibular first molars C. Linguo-occlusalline
D. Maxillary second molars D. Curve of Spee
E. Mandibular second molars E. Curve of Wilson

107. The upper compartment of the


temporomandibular joint is that space
102. Which of the following is the correct sequence between the
of dental tissues from SOFTEST to
HARDEST? A. condylar head and the disc.
B. inferior and superior retrodiscal lamina.
A. Dentin, cementum, enamel C. retrodiscal tissue and capsular ligament.
B. Cementum, dentin, enamel D. disc and the articular fossa and
C. Dentin, enamel, cementum eminence.
D. Cementum, enamel, dentin

108. If a maxillary first molar has a fourth pulp


103. The crown of which premolar is wider canal, it is located in which of the following
faciolingually than mesiodistally, and has two roots?
cusps that are approximately equal in height?
A. Mesiofacial
A. Maxillary first B. Distofacial
B. Mandibular first C. Palatal
C. Maxillary second D. Fourth
D. Mandibular second

104. In the intercuspal position, the lingual cusp of


a mandibular first premolar usually occludes 109. Which of the following representsthe general
crown form of canines, as viewed from the
labial aspect?
A. with the mesial marginal ridge of the
maxillary second premolar and the distal
marginal ridge of the first premolar. A. Hexagonal
B. in the lingual embrasure between B. Triangular
maxillary canine and first premolar. C. Rhomboidal
C. with the lingual surface of the maxillary D. Pentagonal
first premolar. E. Diamond-shaped
D. in the lingual embrasure between
maxillary premolars.
E. with no maxillary tooth.
110. On the crown of the maxillary canine, the 115. When compared to the maxillary central
height of contour is normally located in the incisor, the maxillary canine normally exhibits
cervical third of which of the following which of the following?
surfaces?
A. Shorter root
A. Labial B. Wider crown mesiodistally
6. Lingual C. Thicker crown labiolingually
C. Both labial and lingual D. Longer crown incisogingivally
D. Neither labial nor lingual E. Shorter crown and shorter root length

111. Which of the following surfaces of a tooth is 116. Which of the following teeth have proximal
always next to an adjacent tooth? contact areas at approximately the same
levels cervicoincisally,or cervico-occlusally,
A. Distal on the mesial and the distal?
B. Facial
C. Palatal A. Maxillary central incisors
D. Occlusal B. Mandibular central incisors
E. Proximal C. Maxillary canines
D. Mandibular canines

112. The protrusive pathway of the mandibular


cusps on the maxillary posterior teeth is 117. In the intercuspal position, the facial aspect of
toward the the mesial cusp ridge of a mandibular canine
opposes which of the following maxillary
A. mesial. structures?
6. facial.
C. distal. A. DL of a canine
D. lingual. 6. DF of a canine
C. ML of a lateral incisor
D. DL of a lateral incisor
E. DL of a central incisor

113. The non-working pathway of the maxillary


cusps on the mandibular posterior teeth is 118. For an amalgam restoration, it will be MOST
toward the difficult to obtain close matrix-band adaptation
on which of the following surfaces?
A. distofacial.
6. distolingual. A. Mesial of a maxillary first premolar
C. mesiofacial. B. Distal of a maxillary first premolar
D. mesiolingual. C. Mesial of a maxillary second premolar
D. Distal of a mandibular second premolar
E. Mesial of a mandibular first molar
114. Which of the following illustrations represents
the distal view of a maxillary first molar?

119. From an occlusal view, the arrangement of


permanent teeth of the maxillary and
mandibular arches are parabolic in shape. In
one segment of the dentition, however, four
teeth are aligned in a straight line. In what
region is this segment located?

A. Maxillary anterior
6. Maxillary posterior
C. Mandibular anterior
D. Mandibular posterior
120. A posterior tooth has a faciolingual crown 126. Which premolar poses the GREATEST
dimension greater than its mesiodistal crown problem when root canal therapy or extraction
dimension. In addition, it has two roots. This are being considered?
tooth is MOST likely a
A. Mandibular first
A. maxillary first premolar. B. Mandibular second
B. mandibular first premolar. C. Maxillary first
C. mandibular second premolar. D. Maxillary second
D. mandibular first molar.
E. maxillary second molar.
127. Which of the following ranks the roots of the
maxillary first molar in order from LARGEST to
121. Which primary molar typically has a SMALLEST?
transverse ridge, an oblique ridge, and a
distolingual groove? A. Palatal, mesiobuccal,distobuccal
B. Palatal, distobuccal, mesiobuccal
A. Maxillary first C. Mesiobuccal, palatal, distobuccal
B. Maxillary second D. Distobuccal, mesiobuccal, palatal
C. Mandibular first
D. Mandibular second

122. In the sagittal plane, which of the following 128. In an ideal intercuspal relation, which of the
represents the thickest section of the articular following maxillary cusps will oppose the
disc? mesiofacial groove of the mandibular first
molar?
A. Anterior border
B. Posterior border A. Facial cusp of the second premolar
C. Bilaminar zone B. Mesiofacial cusp of the first molar
D. Intermediate zone C. Distofacial cusp of the first molar
E. Retrodiscal area D. Mesiolingual cusp of the first molar
E. Distolingual cusp of the first molar
123. Which of the following premolars usually has
two lingual cusps?

A. Maxillary first
B. Maxillary second
129. When proximal surfaces of adjacent teeth
diverge from an area of contact, an
C. Mandibular first
embrasure is formed
D. Mandibular second
A. lingually only.
124. Which of the following anterior teeth exhibits B. lingually and facially only.
the MOST deviation in crown morphology? C. lingually and occlusally only.
D. lingually, facially, occlusally, and
A. Maxillary central incisor cervically.
B. Maxillary lateral incisor
C. Mandibular lateral incisor
D. Maxillary canine
E. Mandibular canine
130. The pulp chamber of a mature tooth contains

125. Which of the following periodontal ligament A. cells that continue to form primary
fibers provide the major support to the tooth dentin.
during function? B. an enamel lining for thermal protection.
C. blood vessels and nerves.
A. Apical D. cernentoblasts.
B. Oblique E. ameloblasts.
C. Horizontal
D. Alveolar crest
E. lnterradicular
131. Between which of the following teeth in the 137. How many pulp horns would be expected on a
primary dentition is the mandibular "primate" newly erupted Tooth #18?
space usually found?

A. Central incisors
6. Central and lateral incisors
C. Lateral incisor and canine
D. Canine and first molar
E. First and second molars
138. In which of the following areas of a healthy
mouth is the alveolar process the thinnest?
132. During a lateral movement of the mandible.
the articular disc is tightly attached to the head A. Lingual to the maxillary central incisors
of the condyle by which of the following 6. Facial to the mandibular central incisors
ligaments? C. Lingual to the maxillary canines
D. Lingual to the mandibular first molars
A. Superior retrodiscal E. Facial to the maxillary second molars
B. Inferior retrodiscal
C. Sphenomandibular
D. Stylomandibular 139. On which of the following surfaces of
E. Collateral permanent teeth is pit and fissure caries
MOST likely to occur?

133. Which of the following teeth has its distal A. Facial surfaces of maxillary first molars
proximal contact area located entirely in the 6. Lingual surfaces of maxillary first molars
incisal or the occlusal one-third? C. Facial surfaces of mandibular first
premolars
A. Maxillary canine D. Lingual surfaces of mandibular first
B. Mandibular canine molars
C. Maxillary lateral incisor E. Proximal surfaces of mandibular incisors
D. Mandibular lateral incisor
E. Mandibular second premolar
140. When in its proper position relative to the
plane of occlusion, the crown of a mandibular
134. The occlusal outline of a mandibular first second molar inclines
molar is usually similar to a
A. distally and facially.
A. circle. 8. mesially and facially.
6. square. C. mesially and lingually.
C. rhomboid. D. distally and lingually.
D. pentagon.

141. Assuming occlusion and alignment are


135. Which of the following teeth is the MOST likely normal, the arrow on the illustration below
to have a bifurcated root? represents the path taken by which cusp of a
second molar?
A. Maxillary central incisor
6. Maxillary lateral incisor
C. Mandibular lateral incisor
D. Maxillary canine
E. Mandibular canine

136. The occlusal table of any posterior tooth


makes up what percent of the total Cusp
buccolingual dimension of the tooth?
A. Mesiofacial
A. 25-35 percent 6. Mesiolingual
B. 40-50 percent C. Distofacial
C. 55-65 percent D. Distolingual
D. 70-80 percent E. Cusp of Carabelli
142. In an ideal intercuspal relation, which of the 147. In a protrusive movement, the maxillary left
following maxillary teeth have single central incisor can potentially come in contact
antagonists? with which of the following mandibular teeth?

A. Central incisors A. #22 and #23


B. Lateral incisors B. #23 and #24
C. First premolars C. #24 and #25
D. First molars D. #25 and #26
E. Third molars

148. Which primary molar has a crown somewhat


143. Which of the following represents the structure resembling a permanent premolar, but the
on the crown of a maxillary canine which is root form is typical of a permanent molar?
located immediately to the mesial of the
mesiolingualfossa? A. Maxillary first
B. Maxillary second
A. Lingual ridge C. Mandibular first
B. Mesial marginal ridge D. Mandibular second
C. Distal marginal ridge
D. Distolingual fossa
E. Mesial developmental groove

149. Which of the following plays the greatest role


in disoccluding the posterior teeth in latero-
protrusive movements?
144. The epithelial attachment is always an actual
part of the tooth's A. Anterior guidance
B. Posterior guidance
A. anatomical crown. C. Bennett side shift
B. clinical crown. D. Intercondylar distance
C. gingival line.
D. cervical line.
E. periodontium.

150. Which of the following represents the


145. The dentition of a normally developed EARLIEST age by which the roots of the
6 ID-year-old child usually consists of which maxillary first premolar are completely
of the following teeth? formed?

A. 8 primary and 8 permanent A. 8-9 years


B. 10 primary and 10 permanent B. 10-11 years
C. 18 primary and 6 permanent C. 12-13 years
D. 20 primary D. 14-15 years

46. Which of the following represents the


movement of a tooth through the surrounding
tissue so that the clinical crown gradually
appears longer? 151. Viewed from the occlusal, the basic coronal
outline of a mandibular second premolar is
A. Eruption
B. Attrition A. square.
C. Mastication B. circular.
D. Exfoliation C. rhomboidal.
E. Longitudinal development D. pentagonal.
E. rectangular.
Which of the following BEST describes the 156. Enamel rods in a primary tooth extend
location of the cervical line on a mandibular
lateral incisor? A. from the dentinoenamel junction apically
in the cervical third of the crown.
A. On the distal, it is more apical than on B. from the dentinoenameljunction
the mesial. occlusally in the cervical third of the
B. On the lingual, it is more incisal than on crown.
the facial. C. in the same direction as in a permanent
C. On the mesial, it is more apical than on tooth in the cervical third, but not in the
the distal. incisal or occlusal thirds of the crown.
D. On the facial, it is more apical than on D. in a diffused and gnarled manner in the
the lingual. cervical third of the crown.
E. On the facial and the lingual, it is at the
same level.
157. Which of the following molars has the
LARGEST mesiodistal measurement of its
crown?

A. Maxillary first
153. In a normal occlusion, the lingual cusp of the B. Mandibular first
maxillary second premolar occludes with C. Maxillary second
which fossa of which mandibular tooth? D. Mandibular second

Fossa Tooth

A. Distal Second premolar


B. Mesial Second premolar 158. Which of the following teeth would MOST
C. Central First molar likely be congenitally absent?
D. Mesial First molar
A. Maxillary central incisor
B. Mandibular central incisor
C. Maxillary lateral incisor
D. Mandibular lateral incisor
E. Maxillary canine
154. Ligaments associated with the TMJ serve to

inhibit the normal masticatory cycle.


protect surrounding and supporting
tissues from damages. 159. Which of the following posterior teeth normally
assist musculature in producing exhibit two root canal orifices?
movement.
D. allow for retrusive movement to occur. A. Mandibular first premolars
E. provide elastic potential to allow for B. Maxillary first premolars
border movements. C. Maxillary second premolars
D. Maxillary first molars
E. Mandibular first molars

155. From a facial view, where would the 160. Which of the following describes the proximal
cementoenamel junction be MOST apically contact relationship between a maxillary
positioned on the crown of a primary central incisor and a maxillary lateral incisor?
mandibular first molar?
A. Contact is offset to the lingual.
A. Mesial one-third B. Contact is centered incisocervically.
B. Middle one-third C. Lingual embrasure is larger than the
C. Distal one-third facial embrasure.
D. Equally positioned in all thirds D. lncisal embrasure is the largest of all the
embrasures.
161. A crown concavity, which is confluent with a 166. From the facial view, which premolar has a
longitudinal groove of the root, is a rather mesial cusp ridge longer than its distal cusp
common feature of which surface of which ridge?
premolar?
A. Maxillary first
Surface Premolar B. Maxillary second
C. Mandibular first
A. Mesial Maxillary first D. Mandibular second
B. Distal Maxillary first
C. Mesial Mandibular first
D. Distal Mandibular first 167. Which of the following characteristics is
E. Mesial Mandibular second common to maxillary first and second
premolars?

A. The lingual cusp tip is offset to the


162. A peg-shaped crown form is an occasional mesial.
anomalous occurrence of which of the B. The lingual cusp tip is offset to the distal.
following incisors? C. The facial and lingual cusps are equal in
size and prominence.
A. Maxillary central D. The mesial root concavity extends
B. Maxillary lateral across the cervical line onto the cervical
C. Mandibular central third of the crown.
D. Mandibular lateral E. The lingual height of contour is in the
cervical third of the crown.

163. The crown of a mandibular lateral incisor,


compared to the crown of a mandibular central 168. In a molar, where do root canals usually join
incisor in the same mouth, is BEST described the pulp chamber?
as
A. At the level of the furcation
A. widermesiodistally. B. At varying levels, dependent upon age
B. narrower mesiodistally. C. Within the middle third of the crown
C. more symmetrical. D. Within the cervical third of the crown
D. having a smoother lingual surface. E. Apical to the cementoenamel junction

4 64. Which of the following characteristics of a 169. The lingual aspect of a mandibular first molar
mandibular first molar distinguishes this tooth presents with
from a mandibular second molar?
A. a lingual groove extending onto the
A. Number of roots lingual surface and ending in the cervical
B. Number of lingual cusps third of the crown.
C. Presence of a lingual groove B. a large mesiolingual cusp and much
D. Position of the lingual cusps smaller distolingual cusp.
E. Number of developmental grooves C. the lingual surface of each cusp
possessing a slightly convex shape in
the occlusal third.
D. three lingual cusps.
E. two lingual grooves.
165. When compared with the incisal embrasure
between the maxillary central and lateral
incisors, the incisal embrasure between the 170. On the crown of a maxillary canine, which of
maxillary central incisors is the following structures is located immediately
to the mesial of the mesiolingual fossa?
A. larger.
B. smaller. A. Lingual ridge
C. the same size. B. Distal marginal ridge
D. determined by the position of the C. Mesial marginal ridge
epithelial attachment. D. Distolingual fossa
E. determined by the height of curvature of E. Mesial developmental groove
the cervical line.
171. The transseptal group of periodontal fibers 176. When the posterior teeth are in a crossbite
travel from one root to an adjacent root relationship, which of the following cusps are
surface. considered supporting cusps?

This pathway occurs via the facial aspect of A. Maxillary facial and mandibular facial
the alveolus. B. Maxillary facial and mandibular lingual
C. Maxillary lingual and mandibular facial
A. Both statements are TRUE. D. Maxillary lingual and mandibular lingual
B. Both statements are FALSE.
C. The first statement is TRUE, the second
is FALSE. 177. The central pit of the maxillary first molar is
D. The first statement is FALSE, the second formed at the bases of the triangular ridges of
is TRUE. which cusps?

A. Mesiobuccal, mesiolingual,and
When the mandible moves to the right side, distobuccal
which of the following represents the B. Mesiobuccal, mesiolingual, and
excursive tooth contacts which could occur on distolingual
the right molars? C. Mesiobuccal, distobuccal, and
distolingual
A. Non-working side contacts D. Mesiolingual, distobuccal, and
B. Working side contacts distolingual
C. Lateral contacts
D. Occlusion
178. Which of the following grooves on a
mandibular first premolar originates in an
occlusal pit and extends onto a proximal
173. Which of the following representsthe eighth surface?
tooth from the midline in each mandibular
quadrant at age 26? A. Mesiolingual developmental
B. Distolingual developmental
A. Second molar with incomplete root C. Mesial marginal
B. Second molar with complete root D. Linguogingival
C. Third molar with incomplete root
D. Third molar with complete root
E. Not erupted
179. The physiologic rest position of the mandible
174. The usual order of eruption of primary teeth is established when the
following the central incisors is
A. muscles of mastication are in tonic
A. first molars, lateral incisors, second equilibrium.
molars, canines. B. maxillary and mandibular teeth make the
B. lateral incisors, first molars, second greatest occlusal contact.
molars, canines. C. condyles are in their most retruded
C. lateral incisors, first molars, canines, position.
second molars. D. condyles are in their most protruded
D. lateral incisors, canines, first molars, position.
second molars.

175. During an endodontic procedure on 180. In an ideal intercuspal relation, the


Tooth #21, clinical symptoms lead the dentist mesiolingualcusps of maxillary molars
to suspect the presence of a second pulp occlude with which of the following anatomic
canal. Which direction is the MOST likely structures of mandibular molars?
location for the suspected canal?
A. Mesial fossae
A. Facial B. Distal fossae
B. Distal C. Central fossae
C. Lingual D. Mesial marginal ridges
D. Mesial
181. Which of the following occurs in a right lateral 186. In the intercuspal position, the facio-incisal
movement? aspect of a mandibular central incisor opposes
which of the following structure(s) of which
A. The right condyle primarily rotates. maxillary incisor?
B. The right condyle moves down the
eminentia. Marginal ridge(s) Maxillary incisor
C. The facial cusps of the mandibular left
side pass under the maxillary left facial A. Facio-incisal Lateral
cusps. B. Mesial and distal Lateral
D. The facial cusps of the mandibular right C. Distal Lateral
side pass under the maxillary right D. Mesial Central
lingual cusps. E. Distal Central

187. Excessive calcified tissue formation at the root


apices is known as which of the following?
182. Some degree of protection for lips, cheeks,
and tongue is afforded by the A. Concrescence
B. Enamel pearls
A. contact of adjacent teeth. C. Hypercementosis
B. deflecting function of triangular ridges. D. Cementa1 pearls
C. facial and lingual heights of contour of
the teeth.
D. horizontal and vertical overlapping of the
188. In the midroot cross-section, which root of a
teeth. first molar has the largest area?

A. The distofacial of a maxillary


B. The mesiofacial of a maxillary
C. The lingual of a maxillary
D. The mesial of a mandibular
183. Which of the following structures calcifies first E. The distal of a mandibular
in an anterior tooth?

A. Cingulum 189. Which of the following teeth usually has the


B. Cervical ridge steepest cusp inclines?
C. Marginal ridge
D. lncisal ridge A. Maxillary first premolar
E. Root apex B. Maxillary first molar
C. Mandibular second premolar
D. Mandibular first molar
E. Mandibular second molar
184. When compared to a maxillary first molar, the
crown of a second molar normally exhibits a
190. Which of the following teeth normally exhibits
A. larger distofacial cusp. five major cusps?
B. shorter distolingual groove.
C. greater mesiodistal crown width. A. Primary mandibular first molar
D. greater occlusocervical crown width. B. Primary maxillary second molar
C. Maxillary first molar
D. Mandibular first molar
E. Mandibular second molar

185. Which of the following are the principal muscle


fibers that retrude the mandible? 191. In the diagram of the envelope of motion, the
maximum opening position is the MOST
A. Superficial rnasseter
B. Posterior belly of the digastric A. centrally-located.
C. Posterior fibers of the temporal B. anterior.
D. Posterior fibers of the internal pterygoid C. superior.
D. posterior.
E. inferior.
192. The primary mandibular first molar normally 196. The third tooth from the midline normally
exhibits which of the following? erupts prior to the fourth tooth from the midline
in
A. An oblique ridge
6. A facial pit A. the mandibular arch of the primary
C. Three roots dentition.
D. A distal groove B. the maxillary arch of the primary
E. A distal triangular fossa dentition.
C. the mandibular arch of the permanent
dentition.
D. the maxillary arch of the permanent
dentition.
193. Which of the following BEST characterizes a
of mandibular second molar?
197. At what age is a primary maxillary canine
usually exfoliated?
A. Exhibits four cusps
B. Exhibits a DF groove
A. 6 to 7 years
C. Exhibits a distal cusp
B. 8 to 9 years
D. Exhibits one fused root
C. 10 to IIyears
E. Similar to the five-cusp variety
D. 12tol3years
mandibular third molar
E. 14 to I 5 years

198. Which of the following grooves on a maxillary


first molar runs from the mesial pit to the
194. Assuming occlusion and alignment are central pit on the occlusal surface?
normal, the arrow in the illustration below
represents the path taken by a A. Transverse of the oblique ridge
B. Distal oblique
C. Distolingual
D. Faciolingual
E. Central

199. The maxillary lateral incisor is usually equal to


or larger than the maxillary central incisor in
which dimensions?
A. cusp of canine.
B. facial cusp of first premolar. A. Root width (faciolingually)
C. lingual cusp of first premolar. B. Crown width (mesiodistally)
D. facial cusp of second premolar. C. Crown width (faciolingually)
E. lingual cusp of second premolar D. Root length (cervicoapically)
E. Crown length (cervicoincisally)

195. When a patient moves from maximum 200. Which of the following canines is the last
intercuspal position to place the anterior teeth anterior tooth to erupt and at what age does it
into an edge-to-edge relationship, the erupt?
condyles of the mandible have moved
A. Mandibular; 9-10 years
A. backward. B. Maxillary; 9-10 years
B. forward. C. Mandibular; 11-12 years
C. downward. D. Maxillary; 11-12 years
D. forward and downward.
E. backward and downward.
NATIONAL BOARD DENTAL EXAMINATIONS
PART 1

TEST: DENTAL ANATOMY-OCCLUSION


FORM: 14 (A14)
DATE: D98

Item Key Item Key Item Key Item Key

101 D 126 C 151 D 176 B


102 B 127 A 152 A 177 A
103 C 128 B 153 A 178 A
104 E 129 D 154 B 179 A
105 B 130 C 155 A 180 C

106 B 131 D 156 B 181 A


107 D 132 E 157 B 182 D
108 A 133 D 158 C 183 D
109 D 134 D 159 B 184 B
110 C 135 E 160 C 185 C

111 E 136 C 161 A 186 D


112 A 137 C 162 B 187 C
113 A 138 B 163 A 188 D
114 D 139 B 164 E 189 A
115 C 140 C 165 B 190 D

116 B 141 D 166 A 191 E


117 D 142 E 167 A 192 E
118 A 143 B 168 E 193 A
119 D 144 E 169 C 194 D
120 A 145 C 170 C 195 D

121 B 146 A 171 C 196 C


122 B 147 B 172 B 197 C
123 D 148 A 173 D 198 E
124 B 149 A 174 C 199 D
125 B 150 C 175 C 200 D
PART l MICROBIOLOGY-PATHOLOGY (13)
MONDAY - P.M. DENTAL ANATOMY AND OCCLUSION (14)
DECEMBER 1998

TEST BOOKLET

DO NOT OPEN TEST BOOKLET UNTIL INSTRUCTED TO BEGIN.

1. In addition to this test booklet, you should have an answer sheet.

2. Use a No. 2 pencil on all test materials.

3. Enter your name and reference number below.

Name
Last First Middle

Reference Number

4. Enter your name and reference number on the side of the answer sheet numbered 1 to 100 (SIDE 1).
Blacken the circle that corresponds to each digit of your reference number.

5. Enter the name of this test (MICROBIOLOGY-PATHOLOGY) and the number of this test (13) on the
answer sheet. Blacken the circles that correspond to the test number.

6. Check to be sure you have completed each step above.

YOU ARE READY TO BEGIN. For each test item, decide which choice is correct and blacken the corresponding
circle on the answer sheet. Record only one answer for each test item; there is no penalty for selecting an
incorrect response. You may write in the test booklet; however, your score is based on the total number of
correct answers recorded on your answer sheet. You are allowed 3 112 hours to complete this test booklet.

All test materials must be returned before or immediately upon the request of the Test Administrator. Use of a
magnifying glass is permitted. Use of other study aids is not permitted. No test items are to be copied or notes
taken. Infraction of these rules will result in your scores being voided.

COPYRIGHT 1998
JOINT COMMISSION ON NATIONAL DENTAL EXAMINATIONS
AMERICAN DENTAL ASSOCIATION
211 EAST CHICAGO AVENUE, CHICAGO, ILLINOIS 60611
1. A patient who has anemia, poorly localized 6. Which of the following commonly produces a
abdominal pain, and wrist and foot drop probably positive heterophile antibody test?
is manifesting a toxic state induced by which of
the following? A. Adenovirus
B. Cytomegalovirus
A. Carbon tetrachloride C. Epstein-Barrvirus
B. Carbon monoxide D. Parainfluenza virus
C. Bismuth E. Herpes simplex virus type 1
D. Mercury
E. Lead

2. A patient with shingies (herpes zoster) is MOST 7. Which of the following inflammatory responses is
likely to have had a previous episode of which of associated with an infection caused by pyogenic
the following? pathogens?

A. Measles A. Catarrhal
B. Roseola B. Granulomatous
C. Chickenpox C. Pseudomembranous
D. Viral meningitis D. Acute suppurative
E. Infectious mononucleosis

8. Which of the following bacterial genera is LEAST


3. Normal human cells contain gene sequences likely to be found among normal anaerobic flora of
homologous to virus genome sequences known to the gingival sulcus?
induce cancer in animals. What term applies to
these gene sequences when found in human A. Mycobactenurn
cells? B. Fusobactenum
C. Prevotella
A. Protooncogenes D. Actinobacillus
B. Viral oncogenes E. Potphyrornonas
C. Retroviral genes
D. Analogous chromosomes
E. Homologous chromosomes 9. If Type A blood is accidentally transfused into a
Type B recipient, the immediate hemolytic
reaction would be the result of which of the
following?

4. Which of the following bacterial vaccines is given A. IgM against the A antigen
routinely to children in the U.S.A.? B. IgA against the A antigen
C. IgG against the B antigen
A. BCG D. T cytotoxic cell activation
B. Cholera E. Sensitization to antigens other than A or B
C. Polyvalent pneumococcus
D. Measles, mumps, rubella
E. Diptheria, pertussis, tetanus

10. Which of the following is the MOST appropriate


alternative for treating a patient allergic to
penicillin who has an infection caused by bacteria
5. Which of the following organisms is commonly sensitive to penicillin?
located on the dorsum of the tongue?
A. Rifampin
A. Actinomyces israelii B. Amoxicillin
B. Streptococcus salivanus C. Erythromycin
C. Lactobacillus acidophilus D. Tetracycline
D. Prevotella rnelaninogenica E. Cephalosporin
E. Streptococcusmutans
11. In the past 30 years, the mortality rate of which of 16. A patient with rheumatic heart disease is MOST
the following forms of cancer in women has likely to develop congestive heart failure due to
shown a marked increase? which of the following?

A. Uterine cervix A. Heart murmur


B. Colorectal B. Valvular insufficiency
C. Stomach C. Digitalis toxicity
D. Breast D. Cor pulmonale
E. Lung E. Aschoff bodies in coronary arteries

12. Normal clotting time and normal platelet count


accompanied by prolonged bleeding time 17. Which of the following represents the phenotypic
suggest which of the following? expression of both alleles in a gene pair?

A. Hemophilia A. Penetrance
B. Steatorrhea B. Lyonization
C. Polycythemia Vera C. Codominance
D. Thrombocytopenic purpura D. Hybridization
E. Excessive aspirin ingestion E. Heterozygosity

13. Dry heat sterilization of dental instruments 18. Hereditary angioedema is the result of which of
requires the following?

A. 3 minutes at 2 7 0 ' ~ (132'~). A. Lack of a thymus


B. Deficiency in C1 esterase inhibitor
B. 20 minutes at 2 5 0 ' ~ (121'~). C. Developmental arrest of lymphocytes
C. 30 minutes at 3 2 0 ' ~ (160'~). D. Defective VH gene recombination to DJH
D. 60 minutes at 3 4 0 ' ~ (171'~). E. Bare lymphocytes (no Class llll antigens)
E. 6 hours at 1 0 0 ' ~(38'~).

14. Successful immunization with a hepatitis B 19. Legionella pneumophilia is commonly transmitted
vaccine results in the development of serum by
antibodies to which of the following?
A. sexual contact.
A. HBsAg B. water contaminated with human waste.
B. HBcAg C. aerosolization of organisms in contaminated
C. HBeAg soil.
D. HBV-DNA polymerase D. direct contact with respiratory droplets from
a contaminated person.
E. aerosolization of organisms in air
conditioning cooling towers.

15. Which of the following represents a simple 20. Which of the following BEST represents the
autosomal dominant disorder? pathogenesis of jaundice in patientswith
hepatitis A?
A. Type 1 diabetes mellitus
B. Type 2 diabetes mellitus A. Massive hemolysis
C. Gardner syndrome B. Portal hypertension
D. Cleft palate C. Damage to liver cells
E. Gout D. Massive fibrosis of the liver
E. Obstruction of main bile ducts
21. Which of the following pathologic changes is 27. Uncomplicated healing of a wound by secondary
irreversible? intention, observed microscopically at three days,
is MOST likely to show evidence of which ofthe
A. Fatty change in liver cells following?
B. Karyolysis in myocardial cells
C. Glycogen deposition in hepatocyte nuclei A. Mature cicatrix
D. Hydropic vacuolization of renal tubular B. Keloid formation
epithelial cells C. Granulomatous inflammation
D. Lack of acute inflammation
E. Ulceration of the epithelial surface
22. Which of the following is the major VIRAL cause
of birth defects in infants in developed countries?
28. Which of the following represents the MOST
A. Measles common autopsy finding in instances of sudden
B. Cytomegalovirus death as a result of myocardial infarction?
C. Herpes simplex
D. Rubella A. Valvular prolapse
E. Varicella-zoster B. Coronary thrombosis
C. Fibrinous pericarditis
D. Aneurysm of the heart wall
23. Warts are caused by which of the following? E. Rupture of the coronary vessel

A. Papillomaviruses
B. Herpesviruses
C. Rhinoviruses
D. Adenoviruses 29. Which of the following represents the
E. Rotaviruses characteristic cellular infiltrate in delayed type
hypersensitivity?
24. To which of the following molecules does HIV A. T cells and macrophages
bind? B. Eosinophils
C. Neutrophils
A. CD4 D. B cells and plasma cells
B. CD8 E. Mast cells and basophils
C. MHC Class I
D. MHC Class II
E. immunoglobulin 30. Which type of oral candidiasis is likely to produce
a firmly adherent white plaque on the oral
mucosa?
25. Which of the following viral diseases has the A. Acute pseudomembranous
LONGEST incubation period? B. Chronic hyperplastic
C. Chronic atrophic
A. Herpetic gingivostomatitis D. Acute atrophic
B. Common cold
C. Influenza
D. Measles
E. Rabies
31. If a foreign antigen enters the body through the
26. Which of the following typically results in sudden skin, which of the following portions of the
death? lymphoid system is the antigen MOST likely to
reach FIRST?
A. Bronchiectasis
B. Pneumoconiosis A. Liver
C. Pulmonary emphysema B. Spleen
D. Pulmonary saddle embolus C. Thymus
E. Idiopathic pulmonary fibrosis D. Lymph nodes
E. Mucosal-associated lymphoid tissue
32. Each of the following is commonly associated with 38. Which of the following genera of bacteria is MOST
congestive heart failure EXCEPT one. Which commonly found on the skin?
one is this EXCEPTION?
A. Candida
A. Dyspnea B. Brucella
B. Cyanosis C. Bacillus
C. Anasarca D. Streptococcus
D. Ankle edema E. Staphylococcus
E. Passive congestion of the liver

39. Which of the following is unique to gram-negative


33. The appearance of new strains of human bacteria?
immunodeficiency virus are primarily the result of
A. Flagella
A. natural selection. B. Capsules
B. genomic recombination. C. Ribosomes
C, errors in translation. D. Endotoxins
D. errors in transcription. E. Endospores

34. Bleeding esophageal varices are a common 40. Inadequate immobilization after a bone fracture is
complication of which of the following conditions? MOST likely to result in which of the following?

A. Cirrhosis A. Osteornyelitis
B. Hiatal hernia B. Osteoarthritis
C. Pancreatic carcinoma C. Pseudoarthrosis
D. Chronic active hepatitis D. Heberden's nodes
E. Hepatocellular carcinoma E. Rheumatoid arthritis

41. Phage conversion is responsible for which of the


following?
35. Which of the following is the primary site of origin
of neuroblastoma? A. Transformation of bacteria
B. Production of beta-galactose
C. Production of pyrogenic toxin
A. Brain
D. Ability of Rhizobium species to fix nitrogen
B. Kidney
C. Stomach
D. Mandible 42. Which of the following represents the BESTway
E. Adrenal gland
to verify heat sterilization?

36. In the process of cell death, lysosomal enzymes A. Using a chemical monitor
function mainly to B. Recording sterilizer pressure
C. Recording duration of exposure
D. Recording sterilizer temperature
A. autolyze necrotic cells.
E. Testing with biological indicator
B. mediate cell degeneration.
C. act as a major target for cell injury.
D. activate the complement sequence. 43. Which of the following diseases is MOST often
characterized by hyphae growing in and around
vessels?

A. Candidiasis
37. The antigen-binding site of antibody molecules is B. Mucormycosis
localized in the C. Blastomycosis
D. Histoplasmosis
A. Fab fragment. E. Coccidioidomycosis
B. Fc receptor.
C. J-chain.
D. constant region.
44. In either an allergic reaction or a parasitic 49. The process of gene transfer between bacterial
infection, which of the following cells increase in cells that involves the uptake of naked DNA
number in the bloodstream? molecules is classified as which of the following?

A. Basophils A. Conjugation
B. Eosinophils B. Transduction
C. Plasma cells C. Translocation
D. Atypical lymphocytes D. Transformation
E. Nucleated erythrocytes E. Gene conversion

45. Which of the following viruses is characterized by 50. Obesity, striae, moon face, buffalo hump, and
latency and by the clinical symptoms that can osteoporosis are manifestations of which of the
follow trauma, fever, or nerve damage? following?

A. Variola A. Myxedema
B. Influenza B. Acromegaly
C. Rubella C. Addison's disease
D. Herpes simplex D. Cushing's syndrome
E. Coxsackievirus A E. Diabetes insipidus

51. The capsule of Streptococcus rnutans is an


important virulence factor that
46. Which of the following is the MOST important
function of bacterial pili in causing human A. has proteolytic activity.
infectious disease? B. enhances oral accumulation.
C. causes circulatory collapse.
D. prevents phagocytic digestion.
A. Making the bacteria motile
E. is necessary to cause bacteremia.
B. Allowing bacteria to adhere to human cells
C. Transfering DNA from one bacterium to
another 52. Which of the following genera form endospores?
D. Destroying phagocytic macrophages and
neutrophils
A. Proteus and Escherichia
B. Clostridiurn and Bacillus
C. Porphyromonas and Streptococcus
D. Mycobacteriurn and Actinornyces
E. Staphylococcus and Corynebactenurn
47. Host responses against encapsulated
Streptococcuspneurnoniae are chiefly mediated
by which of the following? 53. Which of the following chemical agents has the
broadest antimicrobial spectrum of activity?
A. IgE
B. IgD A. Phenol
C. Opsonins B. Ethanol
D. Cytotoxic T lymphocytes C. lsopropanol
E. Delayed hypersensitivity D. Glutaraldehyde
E. Quaternary amine

54. Which of the following viruses does NOT establish


48. MOST rickettsia1 diseases can produce severe a latent infection?
illness in humans because rickettsiae
A. Poliovirus
A. produce potent exotoxins. B. Epstein-Barr
B. cause extensive CNS damage. C. Herpes simplex
C. are destructive for epithelial cells. D. Varicellazoster
D. are destructive for endothelial cells. E. Cytomegalovirus
55. Cor pulmonale is usually the direct result of 60. A patient states that he immediately begins
wheezing and has difficulty breathing when
A. viral pneumonia. exposed to penicillin. Which ofthe following
B. myocardial ischemia. classes of antibodies is responsible for the
C. systemic hypertension. difficulty in breathing?
D. resistance to blood flow through the lungs.
E. streptococcal hypersensitization. A. IgA
B. IgE
C. IgG
D. IgM
E. IgD
56. Which of the following represents one function of
macrophages in immunity?
61. Multiple drug resistance is related MOST closely
to bacterial
A. Produce IL-2
B. Produce antibody
Present antigen to antigen-specific T cells A. plasmids.
C.
Mediators of antibody dependent cellular B. chromosomes.
D.
cytotoxicity C. transduction.
D. transformation.
E. viruses in the bacterial wall.
57. What is the role of lactobacilli in coronal caries?
62. An enterotoxin formed by Staphylococcus aureus
A. Primary pathogen in most cases causes
B. Highly pathogenic in animal models
C. Causes progression of existing lesions
Highly adherent colonizer of smooth A. food poisoning.
D.
B. a rash on the skin.
surfaces
Consumes lactic acid produced by C. neuromuscular paralysis.
E.
D. spasms of voluntary muscles.
streptococci

63. The basic chemical structure of endotoxins


includes which of the following?
58. Which of the following represents the formation of A. Capsular polysaccharide
an intravascular blood clot? B. Acid stable peptidoglycan
C. H and L chain glycoproteins
A. Thrombosis D. Lipoteichoic acid, polysaccharide, protein
B. Hemorrhage E. Somatic 0 polysaccharide, core
C. Infarction polysaccharide, lipid A
D. Embolization
E. Transudation
64. Which of the following causes whooping cough?

A. Haemophilus intluenzae
B. Streptococcuspneumoniae
C. Bordetella pertussis
D. Brucella melitensis
59. Which of the following statements is CORRECT E. Klebsiella pneumoniae
regarding glioblastoma multiforme?
65. Which of the following disorders is LEAST likely to
A. The tumor is most common before puberty. be included in the differential diagnosis of a
B. It is classified as a type of meningioma. patient with acute appendicitis?
C. It is the most common tvpe of astrocvtoma.
D. Its prognosis is generaliy more favorable
A. Crohn's disease
than Grade I astrocytoma.
It is derived from the epithelial lining of the B. Duodenal peptic ulcer
E.
C. Meckel's diverticulitis
ventricles.
D. Pelvic inflammatory disease
E. Gastroenteritis with mesenteric adenitis
66. Dimorphism in microorganisms is characterized 71. Penicillin is usually non-toxic to human cells
by the capability to produce because human cells LACK which of the
following?
A. both sexual and asexual spores.
B. both a yeast phase and a mycelial phase. A. Mitochondria
C. two distinct types of clinical infections. B. Peptidoglycans
D. both by budding and by production of C. Topoisomerases
spores. D. Nuclear membranes

72. Anaerobic bacteria obtain their energy


67. During a Type I hypersensitivity reaction, requirements from which of the following
leukotrienes and prostaglandin are generated reactions?
from which of the following?
A. Respiration
A. Thymus B. Fermentation
B. Histamine C. Atmospheric oxygen
C. Arachidonic acid D. Oxidative phosphorylation
D. Bradykinin and other kinins E. Catalase and superoxide dismutase
E. Granules of mast cells and basophils

73. In order for a virus to infect a host cell, it must first


68. Under which of the following circumstances does adsorb onto the cell surface. This process
a graft versus host reaction occur? necessitates

A. When nonvital tissues are grafted A. phagocytic activity by the cell.


B. When viable lymphoid cells are present in B. an inhibition of the cellular secretion of
the graft interferon.
C. When the recipient and the donor are C. an insertion of virally specified glycoproteins
syngeneic into the host-cell membrane.
D. When the graft is contaminated with gram- D. a specific binding of the virus to a
negative microorganisms cell-membrane receptor.
E. When the donor and the recipient are of
different races

69. Which of the following is associated with an 74. Neisseria gonorrheae has affinity for which of the
increased risk of breast cancer? following structures?

A. History of bearing multiple children A. Skin


B. Estrogen deficiency B. Nerve cells
C. Silicone implants C. Plasma cells
D. Fibroadenoma D. Muscle fibers
E. Obesity E. Mucous membrane

75. Which of the following is MOST likely caused by


70. Which of the following are directly associated with chronic blood loss due to a long-standing peptic
destruction of glomerular basement membranes ulcer?
in a patient with glomerulonephritis?
A. Aplastic anemia
A. Eosinophils B. Hemolytic anemia
B. Lymphokines C. Pernicious anemia
C. IgE antibodies D. Myelophthisic anemia
D. Polymorphonuclear leukocytes E. Iron deficiency anemia
76. An afebrile patient with conjugated 81. Individuals with severe, uncomplicated pulmonary
hyperbilirubinemia and an absence of emphysema might be expected to show which of
urobilinogen MOST likely has which of the the following?
following conditions?
A. Cough
A. Cholelithiasis B. Chest pain
B. Aplastic anemia C. Normal or increased total lung capacity
C. Hemolytic anemia D. Difficulty when breathing in (inspiration)
D. Acute hepatitis
E. Alcoholic cirrhosis

82. Which of the following is an oncogenic RNA


virus?
77. Which of the following genera of fungi is MOST
frequently recovered from healthy mucous A. Rotavirus
membranes? B. Retrovirus
C. Herpesvirus
A. Candida D. Paramyxovirus
B. Aspergillus E. Papillomavirus
C. Histoplasma
D. Blastomyces
E. Coccidioides 83. Candidiasis in the adult oral cavity may signify a
change in the balance of oral microbiota. This
particular change is often seen in persons who
are taking which of the following drugs?

Which of the following disinfectants is effective A. Antiviral


against herpes simplex viruses but NOT B. Antifungal
rhinoviruses? C. Antibacterial

A. Phenolics
B. lodophores
84. In addition to Kaposi's sarcoma, which other
malignant neoplasm is often observed in AIDS?
C. Glutaraldehydes
D. Sodium hypochlorite
E. 70% isopropyl alcohol A. Mycosis fungoides
B. Testicular carcinoma
C. Neuroblastoma
D. Non-Hodgkin's lymphoma
E. Rhabdomyosarcoma
79. The chemotactic accumulation of inflammatory
cells that occurs at the sites where immune 85. Which of the following represents a malignant
complexes are deposited is MOST probably due tumor arising from mesenchymal tissue?
to the presence of
A. Sarcoma
A. C5a. B. Adenoma
B. Factor B. C. Carcinoma
C. IL-2. D. Hamartoma
D. IgA. E. Choristoma
E. IgE.

86. Certain enzymes are responsible for suppuration.


80. In which of the following organs or tissues are These enzymes derive chiefly from
ascending infections common?
A. serum.
A. Bone B. tissue.
B. Heart C. neutrophils.
C. Kidney D. lymphocytes.
D. Peritoneum E. plasma cells.
E. Subcutaneous connective tissue
87. Which of the following represents the classic 92. Which of the following represents the epithelial
microscopic lesion of rheumatoid arthritis? change MOST predictive of cancer?

A. Tophus A. Dysplasia
B. Pannus B. Metaplasia
C. Aschoff body C. Acanthosis
D. Heberden node D. Parakeratosis
E. Wire-loop lesion E. Hyperkeratosis

88. A 43-year-old woman exhibits radiographic 93. Acute leukemias are MOSToften seen in which of
evidence of an osteolytic lesion of the humerus. the following age groups?
Her serum calcium level is elevated.
Microscopically, the bone lesion shows numerous A. Under 20 years
giant cells. Which of the following represents the B. 20-40 years
MOST probable diagnosis? C. 40-60 years
D. 60-80 years
A. Renal rickets E. Over 80 years
B. Fibrous dysplasia
C. Osteitis deformans
D. Hyperparathyroidism 94. Which of the following is a major component of
nephrotic syndrome?

89. Adult respiratory distress syndrome might be A. Anemia


caused by each of the following EXCEPT one. B. Hematuria
Which one is this EXCEPTION? C. Hypertension
D. Proteinuria
A. Shock E. Red cell casts in the urine
B. Heroin overdose
C. Viral pneumonia
D. Cigarette smoking 95. Which of the following is the MOST common skin
E. Breathing 100 percent 0 2 cancer in humans?

A. Malignant melanoma
B. Basal cell carcinoma
C. Squamous cell carcinoma
D. Sebaceous adenocarcinoma
90. Which of the following is thought to be of MOST E. Transitional cell carcinoma
significance in the
an uncontrolled diabetic?
96. Which of the following represents the MOST
A. Genetics probable cause for prolonged bleeding time in a
patient with leukemia?

A. Decreased factor Vlll


B. lncreased factor IX
endothelium C. lncreased megakaryocytes
D. Decreased level of serum calcium
E. Decreased number of blood platelets

91. Rheumatic fever can be a sequelae to which of


the following streptococcal infections? 97. An abscess in the oral cavity with central necrosis
extruding pus containing sulfur granules would be
A. Pyoderma which of the following?
B. Diphtheria
C. Scarlet fever A. Furuncle
D. Dental caries B. Tuberculosis
E. Streptococcal gangrene C. Actinomycosis
D. Vincent's infection
98. Patients with which of the following malignancies
have the poorest prognosis?

A. Lung cancer
6. Malignant melanoma
C. Pancreatic carcinoma
D. Carcinoma of the colon
E. Squamous carcinoma of the tongue

99. Asymptomatic carriers are a major hazard of


which of the following?

A. Plague
6. Shigellosis
C. Salmonellosis
D. Typhoid fever
E. Legionnaire's disease

100. Acid phosphatase is elevated in which ofthe


following malignancies?

A. Multiple myeloma
6. Carcinoma of the breast
C. Squamous cell carcinoma
D. Carcinoma of the thyroid
E. Metastatic carcinoma of the prostate
NATIONAL BOARD DENTAL EXAMINATIONS
PART 1

TEST: MICROBIOLOGY-PATHOLOGY
FORM: 13 (A13)
DATE: D98

Item Key Item Key Item Key Item Key

1 E 26 D 51 B 76 A
2 C 27 E 52 B 77 A
3 A 28 B 53 D 78 E
4 E 29 A 54 A 79 A
5 B 30 B 55 D 80 C

6 C 31 D 56 C 81 C
7 D 32 C 57 C 82 B
8 A 33 D 58 A 83 C
9 A 34 A 59 C 84 D
10 C 35 E 60 B 85 A

11 E 36 A 61 A 86 C
12 E 37 A 62 A 87 B
13 D 38 E 63 E 88 D
14 A 39 D 64 C 89 D
15 C 40 C 65 B 90 *
16 B 41 C 66 B 91 C
17 C 42 E 67 C 92 A
18 B 43 B 68 B 93 A
19 E 44 B 69 E 94 D
20 C 45 D 70 D 95 B

21 B 46 B 71 B 96 E
22 B 47 C 72 B 97 C
23 A 48 D 73 D 98 C
24 A 49 D 74 E 99 D
25 E 50 D 75 E 100 E
I
National Board Dental Examinations
TEST PACKET f-fL4
Compiled, released items from approximately 1998-2005
6061 I 3 12-440-2795 Fax: 3 12-440-2820
21 1 East Cl~jcagoAvenue Chicago, Illi~~ois
E-Mail: ASDA@ASDAnel.org Website: v,rw\v.ASDAnet.org

TEST PACKET I-M

National Board Dental Examination Part I


(Compiled, released items from
approximately 1998-2005)

Con tents Page

Randornly ordered test items ...........................................................3

Answer Key.................................................................................... 22
1 VVhich of the iollowing nerves innervates the 6 Which of the following muscles participates in
11-~ferior
buccal n-iucose of tl-ie floor of tlie oral flexion at tile gleno-liurneral and the huriiero-ulliai
cavity? joints?

A Facial A Deltoid
B. Trigeminal B. BI-achialis
C Lir~gual C Biceps brachii
D. Mylohyoid D. Triceps brachii
E Hypoglossal E. Coracobrachialis

2. Each of the follov~ingstructures increases the 7. Cell bodies of sympathetic f~bers~nthe nerve of
surface area of the small intestine EXCEPT one. the pterygo~dcanal come from which of the
Whicl? one is the EXCEPTION? following?

A. Villi A. Facial nerve


B. Rugae B. Superior cervical ganglion
C. Microvilli C. Greater petrosal nerve
D Brush border D. Glossopharyngeal nerve
E. Circular folds E. Otic ganglion

3. The ~nferiorparathyroid gland develops from which 8 In the upper limb, which of the following represents
of the following structures? a hallmark of lymphatic vessels?

A. Second pharyngeal arch A. Contain valves


B. Fourth pharyngeal arch B. Follow the veins
C. Fifth pharyngeal arch C. Always travel in pairs
D. Third pharyngeal pouch D. Only found on the anterior surface of the
E. Fourth pharyngeal pouch limb
E. Contain fenestrations to allow passage of
fluids into the interstitiurn
4. Which of the following controls parathyroid gland
function?
9. Which of the following nerves is associated witti
A. Thyroxin
the second pharyngeal arch?
0. Calcitonin
C. Blood levels of calcium A. Vagus
D. Blood levels of magnesium B. Glossopharyngeal
E. Thyroid stimulating hormone C. Accessory
D. Mandibular
E. Facial
5. The splenic artery is a branch of which of the
following arteries?
10. The nluscles of facial expression are derived from
A. Celiac
which of the following?
B. Lefl hepatic
C. Left gastric A. Frontonasal process
D. Musculophrenic B. First arch
E. Superior mesenteric C. Second arch
D. Third arch
E. Fiflh arch
11 Eact~of the following cranial nerves is associated 16. The nerves of the inter11althoracic wall lie
wttl-I the cavernous sinus EXCEPT one. Whiclh one immediately deep to which layer of the wal!?
is the EXCEPTION?
A External intercostal
A. Facial B. Internal intercostal
B. Abducens C. Transversus thoracis
C. Trochlear D. Parietal pleura
D. TI-igeniinal E. Subcostals
E. Oculoniotor

17. Whicti of the following represents the condition in


12. Which of the following nerves innervates the taste which a radiograph of a 10-year-old patient's
buds of the anterior two-thirds of the tongue? maxilla shows that two succedaneous teeth are
absent?
A. Vagus
B. Facial A. Mesiodens
C. Trigeminal E. Gemination
D. Hypoglossal C. Dens in dente
E. Glossopharyngeal D. Latent odontogenesis
E. Partial anodontia (hypodontia)

13. A deviation of the tongue, when protruded, away


from the midline results from damage to which of 18. Where is the channel of communication between
the following cranial nerves? the maxillary sinus and the nasal cavity situated?

A. V A. In the superior nasal meatus


B VII E. In the sphenoethmoidal recess
C. X C. In the middle nasal meatus, at the semilunar
D. XI hiatus
E. XI1 D. At the anterior extremity of the inferior nasal
meatus
E. At the posterior extremity of the inferior nasal
14. Which of the following exhibits phagocytic activity meatus
in the central nervous system?

A. Ependymal cell 19. In the temporomandibular joint, a very dense


B. Microglial cell collection of organized elastic fibers is found in
C. Oligodendrocyte which of the following areas of the articular disc?
D. Fibrous astrocyte
E. Protoplasmic astrocyte A. Anterior band
B. Posterior band
C. Intermediate band
i5 Each of the following cell layers is derived from D. Posterior-inferior lamina of the bilaminar
oral epithelium EXCEPT one. Which one is the zone
E. Posterior-superior lamina of the bilarninar
EXCEPTION?
zone
A. Dental follicle
G. Stellate reticulum
C. Stratum intermediuni 20. Which of the following represents a muscle that
D. Inner enamel epitheliurn might assist in depressing the mandible?
E. Outer enamel epithelium
A. Masseter
B. Stylohyoid
C. Teniporalis
D. Geniohyoid
E. Medial pterygoid
21. The organic cotnponent of denlil-I IS comprised 26. Cells that will form file veriebrae have their origin
prin~arilyof in wlhicl-I oi ttie followirig?

P,. keratin. A Notocliord


E. reticular fil~ers. B. Neural arch
C. type I collagen. C Derniamyotome
D type ll and Ill collage11 D Two pairs of soniites
E, oxytalan f~bers. E Intermediate mesoder~nalplale

22 Each of the following structures leaves the cranium 27. The nasolacrinial duct drains into which of the
by way of the jugular foramen EXCEPT one. Which following?
one is the EXCEPTION?
A. Middle meatus
A. Cranial nerve IX B. Inferior meatus
B. Cranial nerve X C. Superior meatus
C. Cranial nerve XI D. Sphenoethmoidal recess
D. Cranial nerve XI1
E Sigmoid sinus - internal jugular vein
28. The carotid sheath encloses each of the following
structures EXCEPT one. Which one is the
23. Each of the following venous channels has direct EXCEPTION?
connections with the pterygoid venous plexus
EXCEPT one. Which one is the EXCEPTION? A. Vagus nerve
B. External carotid
A. Maxillary vein C. Internal jugular vein
B. Vertebral vein D. Common carotid artery
C. Deep facial vein
D. lnfraorbital vein
E. Posterior superior alveolar vein 29. Following eruption and initial occlusal contact in
the oral cavity, a tooth will continue to erupt in
order to compensate for occlusal wear. In response
24. Stimulation of the lesser petrosal nerve in an adult to this continuous eruption, which of the following
causes secretion by which of the following glands? is deposited at the apex of the root?
A. Parotid A. Primary dentin
B Lacrimal B. Secondary dentin
C. Sublingual C. Reparative dentin
D. Submandibular D. Cellular cementum
E. Glands of the hard palate E. Acellular cementum

25. Ameloblasts will not differentiate from 30. Which of the following cells is most likely to be
preameloblasts unless they engaged in mitosis at any given time?

A. contact dentin A. Osteocyte


8. are touched by odontoblast processes B. Macrophage
C. are touched by stratum intermedium C Plasma cell
D. contact stellate reticulum D. Chondrocyte
E, contact neural CI-est mesoderm E. Basal keratinocyte
31. Ftbrocarlrlage nornially occurs III 37. S l i a r p e ) ~f~bers
'~ from the periodontal ligament
inserl into which of the following structures?
A. epiglottis
6. external ear. A. Bundle bone and dentin
C. eprpliyseal plate. B. Cortical plates and denti11
D interverlebral d ~ s c s . C. Bundle bo11eand cen-~entur??
E. C-shaped rlngs in the wall of the trachea D. Cortrcal plates and cementum
E. Havers~anbone and cementum

32 There is a distinct change in the type of surface


epithelium at the junction of the 38. In an adult, a Babinski sign indicates damage to

A. oropliarynx and esophagus A the dorsal horn.


B. esophagus and stomach. B, lower motor neurons.
C. fundus and pylorus. C. upper motor neurons.
D. duodenum and jejunum. D. the lateral spinothalamic tract.
E. ileum and colon. E. the dorsal spinocerebellar tract

33. In which of the following would fenestrated 39. Cell bodies of neurons mediating proprioception
capillary plexi be found? from the periodontal ligament of the maxillary frrst
molar lie primarily in the
A. Predentin
6. Dental pulp A. semilunar ganglion.
C. Cellular cementum B. motor nucleus of V.
D. Periodontal ligament C. spinal nucleus of V.
E. Reduced enamel epithelium D. chief sensory nucleus of V.
E. mesencephalic nucleus of V.

34. Submucosal glands are usually located in the


40. In hydrocephalus, excess cerebral spinal fluid is
A. fundus of stomach. found within which of the following?
B. duodenum.
C. appendix, A. Ventricle
D. jejunum. B. Subdural space
E. colon. C. Cisterna rnagna
D. Subarachnoid space
E. Superior sagittal sinus
35. Difficulty in raising the shoulder might be the result
of damage to which of the following cranial
nerves? 41. Intelligence and sensory-motor functions of a
patienl appear to be intact. However, the patient
A. V lacks self-discipline and has exhibited anti-social
8. VII
behavior. He is unable to plan for the future or to
C. X
organize behaviors into logical sequences. A lesion
D. XI
E. XI1 in which of the following areas is likely?

A. Frontal lobe
8. Occipital lobe
36. A stab wound creating a pneuniothorax on the left C. Internal capsulc
side will usually result in collapse of which of the D. Temporal lobe
following? E. Parietal l o l ~ e

A. The lefl lung only


B. The right lung only
C. Both lungs
D The rib cage on the left side
E 7he lefl lung and pericardial sac
42. The neurons of the central nervous systeni that 47. Mucosa of the anterior two-th~rdsof the tongue
iriiiervate rnuscles derived fror-1.1branr:Iiia! arches develops prrniarily from
are found 111which of the following nuclei?
A. Rathke's pouch.
A Nucleus arnbiguus B. tuberculum inipar.
B iiypoglossal nucleus C. foregut endoderni.
C Dorsal motor nucleus of X D, tiypobranchial eminence
D Nucleus of Edinger-Westphal E. lateral lingual swellings.
E. Superior salivatory nucleus

48. Initially, the developing hear1 is


43. Wt-iich of the following represents the basis for the
topical application of fluorides in dental caries A induced by the notochord.
prevention? B, positioned posterior to the notochord.
C. positioned anterior to the prochordal plate.
A. Fluoride penetrates the enamel through the D. between the prochordal plate and the
lamellae. notochord.
B Keratin content of the enamel is made more
insoluble.
C Fluoride coats the tooth forming a uniform 49. Melanocytes migrate to the lamina propria of the
protective barrier. oral mucosa from which of the following?
D, The primary cuticle, being less calcified,
absorbs the fluoride. A. Myotome
E. Acid solubility of the surface enamel is B. Dermatome
reduced by the fluoride. C. Sclerotome
D. Neural crest
E. Rathke's pouch
44. Which of the following represents the primary
mineral component of alveolar bone in the
periodontiurn? 50. Blood from the cephalic vein drains into which of
the following veins?
A. Osteoid
B Elastin A. Basilic
C. Collagen B. Braciocephalic
D. Hydroxyapatite C. Axillary
E. Ground substance D. Internal thoracic
E. Superior vena cava

45. Which of the following represents the primary


function of cementum? 51. Coupled respiration requires each of the following
EXCEPT one. Which one is the EXCEPTION?
A Attach Sharpey's fibers
B. Protect the root from caries A. ADP
C. Repair traumatic injuries of the root B. Oxygen
D Supply nutrition to the periodontal ligament C. Carbon dioxide
E. Maintain the width of the periodontal D. Electron donor
ligament E. Inorganic phosphate

46. The junction between the 100th surface and the 52. If the anticodon on transfer-RNA is 5'ACG3', then
crevicular ep~theliumis composed of which of the which of the following is its corresponding codon
following? on messenger-RNA?

A. Cementoid A. 5' CGT 3'


B. T~gtitjunction 6. 5' CGU 3'
C. Intermediate filaments C. 5' TGC 3'
D. Basal lar-nina-like structure D. 5' UAG 3'
E. Interstitial crevicular fluid E. 5' UGC 3'
53 Wllicl? of the following is the nlajor storage forrn of 58. Eacl.1 of the following is an anticoagcilant EXCEPT
irori iri the body? one. Which one is 1I1e EXCEPTIOId?

A Transferrir? P,. Heparin


B. Wemosider~n B. Vitamin K
C Apoferritir-r C. Dicurnarol
D. Hen?ogIohin D. Sodium citrate
E Ferrilin E. Any antithrombin substance

54. When a mammaliar? red blood cell is placed in 0.3 59 The characteristic of the aorta that is n~ost
per cent sodium chlor~de;water moves across the responsible for the maintenance of diastolic blood
cell membrane by which of the following pressure is its
processes?
A. wide lumen.
A. Osmosis 0. elastic distensibility.
B. Hernolysis C. proximity to the heart.
C. Pinocytosis D. great peripheral resistance
D, Active transport E. active contraction.
E. Diffusion

60. Colloid osmotic pressure of the blood is important


55. Free fatty acids in plasma are because it

A, metabolically inert. A. nourishes blood cells.


B. mainly bound to [beta]-lipoproteins. B. aids in blood clotting.
C. independent of epinephrine secretion. C. prevents loss of erythrocytes from the blood.
D. mainly associated with serum albumin D. prevents excess loss of fluid from capillaries.
E. prevents entrance of tissue fluid into
capillaries.
56. Each of the following is required for normal blood
d o t formation EXCEPT one. Which one is the
EXCEPTION? 61. VVhich of the following is most likely to promote
depolymerization of extracellular matrix?
A. ~a++
6. Plasmin A. Cortisone
C. Thrombin B. Collagenase
D. Vitamin K C. Chymotrypsin
E. Proteolysis D. Hyaluronidase

57. Which of the following combines with heparin to 62. Which porlion of the nervous system contains the
inhibit blood clotting? cardiac, vomiting, and vasomotor centers?

A. Anti-thrombin Ill A. Medulla


B Platelets B. Thalamus
C. Fibrinogen C. Cerebrum
D. Plasminogen D. Cervical region of the spinal cord
E. Thr-ombin
63 The concentration of wliich of the following anill-lo 68. Prior to surgery. an anxious patrent has a higher
acids L ~ I -be
I used to dcter~i-line
whether or not systolic blood pressure than previously noted.
dentrn has contan7inaled a purif~edenamel Which of the following represents the most likely
preparation? reason?

A. Lysirie A Cardiac shock


B. PI-oline B. Anaphylactrc shock
C. Hydroxyproline C. Atrial f~brillation
D. Phosphotyrosine D. Ventricular f~brillation
E. Phosphoserine E. Decreased arterial compliance

64 Calcium is transported in the blood in combination 69. Reduced renal blood flow can cause hypertensiori
with which of the following? by which of the following?

A. Albuniin A. Release of renin


B. Citrate B. Reflex vasoconstriction
C. Hemoglobin C. Retention of potassium in blood
D. Beta-globulin D. Increased output of epinephrine

65. Sympathetic stimulation most likely produces which 70. Which of the following describes the effect of a
of the following? drug that inhibits renal carbonic anhydrase?

A. Glycogen synthesis A. It decreases urea clearance.


B. Bronchial dilation B. It increases the Tm for glucose.
C. Decreased mental activity C. It increases the acidity of the urine.
D. Decreased metabolic rate D. It decreases sodium reabsorption in the
E. lncreased blood flow to the skin proximal tubule.

66. Which of the following is the same value for 71. The composition of plaque is most similar to which
intracellular and interstitial fluid? of the following?

A. [Na+] A. Bone
s. [CI-1
B. Enamel
C. Calculus
c. [K+] D. Oral bacteria
D. Colloid osmotic pressure E. Saliva
E. Total osmotic pressure

72. Which of the following substances is the


67. Which of the following explains why an increased predominant source of ATP at MODERATE levels
illllng of the ventricle during diastole causes a (for greater than 60 minutes) of activity?
more forceful hear1 beat? The increased ventricular
volume A. Amino acids
B. Fatty acids
A. din~inishesthe refractory period of the C. Carbohydrates
ventricle D. Proteins
B. increases end-diastolic fiber length.
C. facilitates conduct~onin the heart.
D. produces a less than o p i ~ m aload.
l
73 Which of the follov~ingsubstances has a clear-aric~ 78. Peptide horniones generally exerl iheir e f f e ~ i
ratr: that IS greater than the glomcrular iiitration wthicfr of the foliovdrr-iy?
It-iroi~yi-I
rate (GFR)'?
A. Intracellular receptors and stin-~ulatirigproteit-i
/A. Para-aniinohippuric acid (PAtI) synthesis
B Potassiunl B. Receptors on the cell nien?brar?earid
C. Glucose stimulating protein synthesis
D. Urea C. Intr-acellular receptors and altering the
specif~cactivity of certair? enzymes
D. Receptors on the cell membrane and
altering the specifrc activity of certain
74 What is the biological significance of the extensive
enzymes
degeneracy of the genetic code?

A. Alters the amino acid sequence of proteins


encoded by the DNA 79. During the period of isovolurnetric contractiori,
B. Mrnirnizes the deleterious effect of mutations which of the following happens?
C. Maximizes the beneficial effect of mutations
D. Increases chain termination A. The semilunar valves are open.
E Leads to active proteins B. The leR ventricular pressure is rising rapidly.
C. The aortic pressure is slightly less than the
left ventricular pressure.
75. Which of the following types of blotting can be
D. The right ventricular pressure is greater than
the left ventricular pressure.
used to identify DNA restriction fragments?

A Eastern
B. Southern 80. Glucose reabsorption in the nephron is usually
C. Northern completed in which of the following?
D. Western
A. Distal tubule
B. Loop of Henle
76. The catabolism of which of the following results in C. Collecting duct
D. Proximal tubule
no energy production in the form of ATP?

A. Lipid
8. Protein 81. Which of the following represents the correct size
C. Nucleotide and characteristic of the nerve fibers that conduct
D. Carbohydrate sensory input of pain from the oral-facial region?

A. Large diameter, myelinated


77. Which of the following substances is released by B. Small diameter, myelinated
blood platelets and causes platelets to stick C. Large diameter, unmyelinated
D. Small diameter, unn~yelinated
together?
E. Intermediate diameter, myelir-iated
A. Thrombin
B. Fibrinogen
C. Phospholipids 82. As blood flows through the periodontal niembrane.
D. Thromboxane A2 tr-emendous pressures occur in response to
E. Prostacyclin 12 (Pg12) forceful occlusion. Blood flow is temporal-ily
reduced, but platelets do not aggregate because of
the presence of

A. ~ a " .
B. ADP.
C. thrombin.
D. throniboxane A2.
E. prostacyclin (Pg12).
8:: I417 exampit: of synergism is the effectc of 88. \r\lhic.h of ltle followirtg n-rolecules causes the
activaiiur I of 17-ryosir-Ikirlase 3 r d the coi.liraclio~-Ioi
1%. ir-isul~i-~
arid glucagot? on blood glucose.
st?-toothn-iuscle'i
B. estroger-r a n d progesterone- on uterilie
mot~irty. A. Actir-I
C, growth tiornior~earid tliyr-oxrrie on skeletal B. Tropor-tin
g rov\lth C. Caln-ioduliri
D. antidiuretic hornlone and aldoster-one on D. Calcitonin
potassrum excr-etrori E. Cholecalcifer.ol

84. The jaw jerk reflex is an exaniple of which of the 89. Which of the following is the primary difference
following reflexes? between juxtameduliary and cortical nephrons?

A. Load A. Renal plasma flow


0. Flexor B. Filtration fraction
C. Withdraviiai C. Length of the distal tubule
D. Dynamic stretch D. Length of the proximal tubule
E. Length of the thin segment of the loop of
Henie
85 The stretch reflex 1s an example of whrch of the
foilowing reflexes?
90. Which of the following is an essential element
A. Withdrawal
found in all cytochromes?
8. Nociceptive
C. Polysynaptic A. Co
D. Monosynaptic B. Cu
C. Fe
D. Mg
86. A patient bites down rapidly on an unexpected E. Zn
hard surface while chewing. Cessation of motor
unit recruitment in jaw closing muscles is caused
by stimulation of 91. Acute cyanide poisoning would most likely lead to
which of the following?
A. rnuscle spindles.
6. mucosai mechanoreceptors. A. lncreased oxygen concentration in arteriai
C. periodontal mechanoreceptors. blood
D. nociceptors in the dental pulp. 8. lncreased carbon dioxide concentration in
venous blood
C. Decreased oxygen extraction by peripheral
87. Which of the following statements about tissues
norepinephrine is cor-rect? D. lncreased oxygen exir-action by peripheral
tissues
A. Causes cardiac acceleration E. Decreased carbon dioxide concentration in
6. Causes general vasodilation arterial blood
C. Causes vasodilation In vessels of the skin
D. Has a negative inotropic effect on the heart
E Is the preganglionic sympathetic 92. Chondroitin sulfate is a major component of vdliich
neurotransmitter of the following?

A. Hair
0. Mucin
C. Cartilage
D. Bacterial ceil walls
E. Blood group substance
93. \riltiich of It-IC followirrg cliaracter~zesbotli active 99. Wt-iicti niain product of protein riitroger?metabolisnl
transporl and facilitated diffusion? is lourid ill human urine'?

A. Hydrolysis of ATP A. Urea


B. Conipetitive inhibition B. Ammonia
C Transport bidirectional C. Creatine
D Transport against a concentration gradient D. Uric acid
E. Creatin~ne

94. W h ~ c hof the following has the most etlect in


stiniulating respiration? 100. Each of the following is expected to be active
during fatty acid biosynthesis EXCEPT one. Which
A Increase in blood pH one is the EXCEPTION?
B. Decrease in arterial p 0 2
C. Increase in arterial pC02 A. Tricarboxylic acid cycle
D. Decrease in blood pH
B. Pyruvate dehydrogenase
C. Amino acid catabolism
E. Decrease in arterial pC02
D. Beta oxidation
E. Glycolysis

95. Which of the following proteins is involved with


bacterial aggregation and subsequent elimination 101. Which of the following statements best describes
from the oral cavity? the roots of a maxillary first molar?

A. The mesiofacial root usually tips mesially in


A. Gustin
its apical one-third.
0. Statherin
B. The palatal root has a concave facial
C. Lactoferrin
surface.
D. Secretory IgA
C. The palatal root is flattened and concave on
its mesial and distal surfaces.
D. The distofacial root is flattened and concave
96. Thyroid hormones are synthesized from which of on its facial surface.
the following amino acids?

A. Tyrosine 102. Which of the following represents the location of


B. Dopamine the lingual height of contour on the crown of the
C. Histidine mandibular second premolar?
D. Threonine
E. Tryptophan A. Middle third
B. Occlusal third
C. Same third as that tooth's buccal height of
contour
97. Which of the following enzymes is responsible f o ~ D. Same third as the lingual height of contour
inactivating catecholamines? on the crown of the maxillary premolars
E. Same third as the lingual height of contour
A. Phosphodiesterase on ttie crown of the mandibular first premolar
B. Monoamirie oxidase
C. Amino decarboxylase
D. Tyrosine hydroxylase 103. When viewed from the facial aspect, the crown of
the mandibular first premolar has an occlusal
outline that nornially exhibits which of the iollowing
98. Protein and RNA synthesis occur in each of the characteristics?
following phases of the cell cycle EXCEPT one. A. Cusp tip which is offset to ttie distal
Wliich one is the EXCEPTION? B. Cusp tip which is centered mesiodislally
C. Disto-occlusal slope which is longer than ttie
A. Go niesio- occlusal slope
B. GI D. Mesio-occlusal slope which is longer than
C. S the disto- occlusal slope
D. G2
E. M
9
04.?-vdo pulp carials are most cornrnonly found in the 110 The part of the tooth w~liich.at a given mon-rent, is
exposed to the oral cavity 1s
A. rool of a mandibular cer-itral incisor.
B, facial root of a maxillary f~rstpremolar. A. the anatomic crown.
C drstal root of a mandibular f~rstmolar. B the clinical crown.
D mesial root of a mandibular frrst molar. C. the functional clrnical crown and root.
E rnesiofacial rool of a n-~axillaryfirst molar. D. measurably larger r r i young persons than irn
older persons.
E, not effected by periodontal health.
105. In the triangle formed by the projection of the
orifrces of the canals of a maxillary first n~olar,the
111. The anatomic crown IS that portion of a tooth that
A. line connecting mesial with lingual is longest. is
B. line connecting distal w ~ t hlingual is longest.
C. line connecting mesial with distal is longest. A. in occlusion.
D. angle at the distolingual canal is obtuse. B. exposed to oral fluids.
E. angle at the mesiofacial canal is obtuse. C. coronal to the cervical line.
D. occlusal to the gingival margins.

106 Which of the following attaches the root surface to


the alveolar bone? 112. A patient has an extremely wide, notched tooth in
the mandibular left central incisor position. Clinical
A. Cementum and radiographic examinations reveal 28 teeth
B. Attached gingiva have erupted, but four third molars have not
C. Dentinal tubules erupted. Which of the following conditrons exists?
D. Periodontal ligament
E. Cementoenamel junction A. Fusion
B. Dens in dente
C. Concrescence
107. Which of the following anatomic structures is found D. Dilaceration
just lingual to the maxillary central incisors? E. Gemination

A. Incisive foramen
B. Canine eminence 113. When compared to a maxillary first molar, the roots
C. Maxillary tuberosity of a maxillary second molar
D. External oblique ridge
E. Greater palatine foramen A. are longer.
B. are more divergent.
C. are fewer in number.
108. Which of the following papillae would normally be D. have less potential for fusion.
found in the buccal vestibule? E. are greater in distal inclination

A. Parotid
6. incisive 114. Which of the following teeth is the most likely to
C. Fungiform have a bifurcated root?
D. Interdental
E. Circumvallate A Maxillary central incisor
B. Maxillary lateral incisor
C. Mandibular lateral incisor
109. The bulk of a tooth consists of D. Maxillary canine
E. Mandibular canine
A. pulp.
B. crown.
C. dentin. 115. Which of the following primary teeth does a second
D. enamel. prernolar replace when it erupts into the oral
E. cernentuni cavity?

A. Canine
B. First premolar
C. Second premolar
D. First molar
E. Second molar
I VVIircli of ti-lc followirng describes the contact '122. 11.) a right virorkirig ~nioven-rent,thc Ilngual cusp of a
Ielatior->st-lipbetween a rnaxillar-y cer lira1 irncisor a116 maxiliar-y rrght second premolar passes througt-I
a maxillary lateral rnclsor? wlhich of the following mandibular structures?

A Contact is offset to the lingual. A. Facral gr-oove of the right first molar
B Contact is centered irncisocervically. B. Lingual groove of t!ie right frrst molar
C. Lingual embrasure is larger than the facial C. Ernbrasure between the right firsf premolar
embrasure. and the right second prernolar
D lncisal embrasure is the largest of all the D. Embrasure between the right second
embrasures. premolar and the right flrst n~olar

1'17. In the horizontal plane, as the mandible moves it1 a 123. The proximal contact areas between anterior teeth
lateral excursion, the midline of the mandible are incisal to the middle third of the teeth. Which of
nioves the following is an EXCEPTION to this rule?

A. straight anteriorly. A. The mesial contact of a maxillary canine


B straight posteriorly. B. The distal contact of a maxillary canine
C. straight laterally. C. The mesial contact of a maxillary lateral
D. anteriorly and laterally. incisor
E. posteriorly and laterally. D. The mesial contact of a mandibular lateral
incisor
118. A patient presents with symptoms of an acute
abscess on the maxillary left lateral incisor. There 124. When the mandible performs a laterotrusive
are no clinical signs of decay or restoration. movement, the laterotrusive-side condyle moves
Radiographically, which of the following will most primarily about which of the following axes?
likely be shown as the cause of the abscess?
A. Vertical
A. Agenesis 6. Sagittal
3. Dilaceration C. Horizontal
C. Concrescence D. Transverse
D. Dens in dente
E. Enamel pearls
125. During a visual examination of a patient who has a
severe clenching habit, which muscle would one
119. Each of the following can be found in the dental expect to see enlarged?
pulp EXCEPT one. Which one is the EXCEPTION?
A. Masseter
A. Nerve tissue 8. Temporalis
B. Blood vessels C. Medial pterygoid
C, Cementoblasts D. Lateral Pterygoid
D. Lymph vessels

126. From the occlusal aspect, which of the following


120. Which of the following molar roots is wide represents the most frequently seen form of a
faciolingually and concave on both mesial and maxillary second molar?
d~stalsurfaces?
A. Round
A. Distofacial of a maxillary flrst 8. Square
B. Lingual of a maxillary frrst C. Trapezoidal
C Mesial of a mandibular flrst D. Rhomboidal
D Distal of a mandibular frrst E. Hearl-shaped

121. The d~stolingualcusp of which of the following 127. Initiat~onof calciflcat~onfor- the mandibular centr-a1
molar-s might be absent? incisor normally occurs a1

A. Maxillary frrst A. 3-4 months of age


B. Mandibular f ~ r s l 8. 1 year of age.
C Mandibular third C. 2-3 years of age.
D. Maxillary third D. 4-5 years of age.
E. 6-7 years of age.
121: The oblique ridge of the rnaxillal-y molar-s extends 134. V\k~er-tin its norn.ral posilioi? relative to tihe arc11
Ijt-tweerl wt-~iclitwo cusps? fo~rl-Ithe cro\yr-Iof a rrrar-ldibular frrct rntolar ir-tcli~rler-

A Mes~oiacialand distofacial A. drstally and facially.


B Mesiolingual and distolingual B niesially and facially.
C Ivlesiolrngual and distofacial C. mesially and lingually
D Mesiofacial and inesiolingual D distally and lingually.
E Mesiofacial and distolingual

135. Which of the following teeth might possess three


129. A! age 8, the maxillary first molar has cusps?

A a mesial contact with the primary frrst molar. A. Maxillary second pr-eniolar and maxillary frrst
B a distal contact with the second molar. molar
C, no mesial contact. B Maxillary second premolar and mandibular
D, no distal contact. flrst premolar
E. not yet erupted. C. Maxillary first molar and mandibular second
molar
D. Maxillary second molar and mandibular f~rst
130. Of the primary maxillary teeth, the c e ~ i c aridge
l premolar
would stand out most prominently as a distinct E. Maxillary second molar and mandibular
entity on which surface of which molar? second premolar
&r Surface
136. Each of the following grooves originates in the
A, First Distofacial
central pit of the maxillary second molar (four cusp
B. First Mesiofacial
type) EXCEPT one. Which one is the
C. Second Distofacial
EXCEPTION?
D. Second Mesiolingual
E. Second Distolingual A. Buccal
B. Central
C. Distolingual
131. Which groove of the mandibular first molar does
D. Transverse grooves of oblique ridge
the maxillary mesiolingual cusp pass through in a
lateral excursive movement on the working side?
137. When the mandible moves from maximum
A. Buccal
intercuspal position distally, any tooth contacts that
B Lingual
occur are called
C. Central
D. Distobuccal A. retrusive contacts.
B. protrusive contacts.
C. mediostrusive side contacts.
132. Which of the following premolars frequently has
D. laterotrusive side contacts.
only one pulp horn?
E. lateral protrusive contacts.
A Maxillary first
B Mandibular first
138. Between which of the following permanent teeth is
C. Maxillary second
the lingual ernbr-asuresmaller than the facial
D. Mandibular second
embrasure?

133. Tlhe incisal embrasure is the snlallest between


A. Maxillary frrst premolar and maxillary second
premolar
wlilch of the following two teeth?
6. Maxillary second molar and maxillary thir-d
A. Maxillary central incisors molar
B Mandibular central incisor-s C. I\/iandibular first molar 2nd mandibular
C. Maxillary central and lateral incisors second molar
D. Maxillary lateral incisor and canine D. Mandibular f~rstprenlolar and mandibular
E Mandibular lateral Incisor and central incisor second premolar
139 On a mandibular frst molar, the dlstofacial groove 144. Wliicli of the followirig represents the slructure in
serves as an esc;apeway for the niesiolir?gual c u s l ~ the I-r-iaxillaryalveolar bone that n~axillarypremolar
of tile rr-~axillaryfirst molar durirlg which of the roofs occasionally penetrate?
following mandil~ularmovernents?
A. Antrum
A. Working B. Nasal septum
B Non-working C. Frontal sinus
C. Protrusive D. Zygomatic arch
D. Centric slide E. Mandibular fossa

140. Which of the following primary grooves uniting in 145. Which of the following jaw positions is determined
the distal pit o n the occlusal surface of the almost exclusively by tooth contact?
mandibular second molar represents the one that
normally has no counterparl in the distal pit of the A. Rest position
f~rstmolar? B. Terminal hinge
C. Maximum opening
A. Distolingual D. Maximum protrusive
B. D F triangular E. lntercuspal positlon
C. DL triangular
D. Distal marginal
E. Distal portion of the central 146. In a CUSP TO FOSSA contacting relationship in
intercuspal position, the maxillary first premolar is
most likely to articulate with which of the following
141. Which of the following best describes the cervical mandibular teeth?
margin on the facial crown surface of the maxillary
first molar? A. Lateral incisor
B. Lateral incisor and canine
A. Straight C. First premolar
B. Evenly convex toward the apex D. Canine and first premolar
C. Evenly convex toward the occlusal E. First premolar and second premolar
D. Irregularly convex toward the apex
E. Irregularly convex toward the occlusal
147. Which of the following is least likely to contribute to
or affect stability of the dental arch form?
142. Which of the following represents the most
common type of root anomaly on the mandibular A. Periodontal health
f~rstpremolar? B. Plane of occlusion
C. Occlusal contact forces
A. Dwarfing D. Interproximal contact form
B. Elongation E. Forces exerted by the lips and tongue
C. Bifurcation
D. Concrescence
E. Trifurcation 148. When viewed from the frontal plane and
progressing posteriorly, the axial inclination of the
crowns of maxillary posterior teeth
143. Which of the following teeth represents the one
most likely to present with three roots? A. remains vertical.
0. inclines lingually.
A. Mandibular central incisor C. inclines distally.
B. Mandibular second premolar D. inclines mesially.
C. Mandibular canine E. inclines buccally.
D. Maxillary f~rstpreniolai-
E. Maxillary second premolar
149. When viewed from the sagittal plane, the axial
inchnation of Ihe anterior teeth

A. rernains vertical.
B. inclines facially.
C. inclines mesially.
D. inclines distally.
E. inclines lingually.
150 T i - ~ epresence of n?snielor-ison a 19-year-old 156, VVhicti of the following pathogetis is tl-re single
patlent suggests whict? of the follovdiiig condilioris'? rnosi coninion cause of sexcially trarisniitted
disease it? the U.S.?
A Fluorosis
B. li/ialforniat~o~.r A. Treponenia pallidurn
C. Malnutritiori B. Haeniophilus ducreyi
D. Delayed el-uption C. Chlaniydra trachomatis
E. Anterior open bite D. Herpes simplex virus
E. Huniari immunodeficiency virus

151. In osteonialacia, bones are weak because of the


failure o i 157. Which of the following represents the mechanism
of action of diphtheria toxin?
A. remodeling of bone.
B. bone matrix formation. A. Activates CAMP
C. osteoblast proliferation B. Causes cytolysis
D. bone matrix calcification. C. Inhibits translation
E. interstitial growth of cartilage D. Inhibits transcriptron
E. Inhibits DNA replication

152. Which of the following is the most common


location for an atherosclerotic induced aneurysm? 158. Each of the following fluids is considered one that
can transmit HIV EXCEPT one. Which one IS the
A. Thoracic aorta EXCEPTION?
B. Abdominal aorta
C. Coronary arterles A. Semen
D. Arch of the aorta B. Serum
E. Common iliac arteries C. Saliva
D. Amniotic fluid
E. Breast milk
153. Each of the following statements about poliovirus
infections is correct EXCEPT one. Which one is
the EXCEPTION? 159. Which of the following represents the anaerobic
organism that is cultured from gingival scrapings
A. Most infections are subclinical. and that forms black colonies on hemin-containing
B. Paralysis is an uncommon outcome of culture media?
infection.
C. There are 3 types of poliovirus, making 3 A. Sarcina lutea
infections possible. B. Bacillus anthracis
D. V ~ r u sis predominantly shed from the body C. Veillonella alcalescens
and transmitted in respiratory secretions. D. Porphyromonas gingivalis
E. Some damaged neurons may be repaired, E. Alpha-hemolytic streptococcus
restoring lost functions.

1GO. Beta 1-4 linkages connect N-acetyl glucosamine


154. Which of the following can result if an individual and N-acetyl muraniic acid. The resulting polymer
having reactivation of latent varicella zoster virus is found in bacterial
transmits virus to a seronegative individual?
A. flagella.
A. Shingles B. capsules.
B Chickenpox C. cell walls.
C. lierpes labialis D. metachroniatic granules
D. Infectious mononucleosis
E. Iierpetic gingivostoniatitis
161. Rough pneuniococci that are grown in the
presence of DNA from smooth pneuniococci
155. Which of the following conditions might be an initial develop capsules. This process IS termed
manifestation of early, acute l i l V infection?
A. translation.
A. Kaposi's sarcoma B. transduction.
B. Wasting syndrome C. transformation
C. Oral hairy leukoplakia D. conjugation.
D. Mononucleosis-like syndrome
E Pneumocyslis carin~ipneumonia
I During a17 outbreak of gastroenteritis caused by 168. Nongor~ococcaluretl-iritis is oiler-I caused by
Salriionella; a strairi of the species sudder-~ly microorganisms 01 which of the followiriy genera?
appears to be resistanl to several anlil~ioiics.
VVliich of the following best explains why? A. Chlamydia
B. Treponerna
A. Parent strain has undergone several C Neisseria
mutations D. Hemophilus
B Parent strairi has acqulred a plasmid.
C. There are several specres of Salmonella
present, each resistant to one antibiotic. 169. Which of the following is least likely to ~ ~ r o d u c e
0. New strain is a species of Escherichia coli, acute abdominal symptoms?
which has acquired virulence by mutation.
A. Cliolelithiasis
B. Acute pancreatitis
163. Which of the following karyotypes is found in C. Ampulla of Valer cancer
Turner syndrome? D. Carcinoma of the tail of the pancreas

A. 45, XO
B. 45, YO 170. Which of the following represents the arthritis that
C. 46,XX is usually associated with aging?
D. 47,XXY
E. 47. XYY A. Osteoarthritis
B. Gouty arthritis
C. Rheumatoid arthritis
164. Which of the following viruses are frequently D. Psoriatic arthritis
associated with eye infections?

A. Mumps and measles viruses 171. Multiple, lytic lesions of bone characterize each of
B. Parainfluenza and rubella viruses the following conditions EXCEPT one. Which is the
C. Coxsackievirus and rhinoviruses EXCEPTION?
D. Adenoviruses and reoviruses
E. Herpes simplex virus and adenoviruses A. Multiple myeloma
6. Metastatic carcinoma
C. Osteogenesis imperfects
165. In humans, the mumps virus is transferred by D. Hyperparathyroidism
E. Langerhans (eosinophilic) granulomatosis
A. food.
B. feces.
C. flies. 172. Osteoporosis can be associated with each of the
D. saliva. following EXCEPT one. Which is the EXCEPTION?

A. Prolonged coriicosteroid administration


166. Human leukocyte antigen (HLA) Class I molecules B. Prolonged immobilization
are found on which of the following? C. Chronic malnutrition
D. Hypervitaminosis D
A All nucleated celis E. Advanced age
B. CD4+ T cells only
C. Epithelial cells only
D. HLA Class 11-1- cells only 173. In general, enzyme-deficiency diseases are
E. Mesenchymal cells only inherited by which of the following modes?

A. Polygenic
167. Soft and hard tissue necrosis char-acterizes wliicli 8. X-linked dominant
of the following fungal diseases? C. Auioson-iai recessive
D. Autosornal dominant
A. Mucormycosis
B. Cryptococcosis
C. Histoplasmosis
D. Coccidioidomycosis
E. Cand~diasis
174 Wtiict-i of the follovi/ing anemias results Iron? drug- 180 Wliich of the following vir-uses IS most likely l o be
~ridilcedbone mar-row supl~~essiorr? isolated front feces?

k Aplastic A. Rubella
El Sickle cell B. Hepatitis C
C Pernicious C. Influenza A
D Hemolytic. D. Coxsackievirus
E. Myelophthisic E. Herpes simplex

175 Multiple drug resistance is associated with 181. Cellular tropisnl by viruses is dependent on which
of the following?
A. plasmids.
B. recombination. A. Host cell DNA homology to viral DNA
C. point mutations. B. Temperature of host cell incubation
D. specialized transducing phages. C. Gamma-interferon production
E. generalized transducing phages D. Cell surface receptors
E. V~ralenzyme synthesis

176. Which of the following statements best describes


bacterial transformation? 182. A 5-year-old child with vesicular lesions limited to
the palate and the posterior oropharyngeal mucosa
A. DNA coding for RNA synthesis
has an oral temperature of 1 0 I 0 F (38OC). The
B. RNA coding for protein synthesis
most probable diagnosis is
C. Acquisition of an inheritable trait by bacteria
mediated by DNA A. chickenpox.
D. Acquisition of an inheritable trait by bacteria B. herpangina.
mediated by RNA C. recurrent oral herpes.
E. Binding of 30s and 7 0 s ribosomes D. herpetic gingivostomatitis.
E. hand-foot-and-mouth disease
177. Plasmid-mediated antibiotic resistance has been
observed in diseases caused by each of the 183. Which of the following cells are the most
following EXCEPT one. Which one is the radiosensitive?
EXCEPTION?
A. Neurons
A. Strepfococcus pyogenes B. Fibroblasts
8. Staphy/ococcus aureus C. Lymphocytes
C. Neisseria gonor~hoeae D. Chondrocyles
D. Bordefella pe~tussis E. Epithelial cells
E. Haeri70pt7ilus influenzae

184. During repair, which of the following substances is


178. Each of tlie following can produce a genotypic essential for precollagen fibers to transform into
change EXCEPT one. Which is tlie EXCEPTION? collagen fibers?
A. Conjugation A. Corlisone
6. Lysogenizat~on B. Carotene
C. Transformation C. Ascorbic acid
D. Exposure to tetracycline D. Prothrombir?
E. Thromboplastin
179. The human imn~unodef~ciency virus preferentially
infects which of thc follov\iing ceils7 185. Which of the follovding conditions represents ari
iritox~cationrather- than an infection?
A. NK
B. Helper T A Anthrax
C. Cytotoxic T B. Botulisni
D. Suppressor 7 C. Chancroid
D. Bacteroidosis
E. Salmonellosis septicemia
186 7-he tensile strengtil of a I-iealing n?ucosal wound 192. Metastatic calcif~cationsare n?osl likely to be the
deperids on wl-iich of the followirig? result of

A. Wound hormones A. gallstones.


B Epithelial regeneration B atherosclerosis.
C. For-n~atior-I
of collagen frbers C. hyperparathyroidisni.
D. Act~vationof frbrir~olysis D, osteogenic sarcoma.
E. Formation of elastic frbers E. lymphatic spread of lung cancer.

187. The chemotactic accumulation of leukocytes at the 193. Difficulty in swallowing refers to
site of immune complex deposition is a result of
A. dyspnea.
A. steroids. B. achalasia.
B histamine. C. dysphagia.
C. complement. D. hemetemesis.
D, antihistamines.

194. Each of the following viruses is potentially capable


188. Which of the following has the least ability to of causing cell transformation EXCEPT one. Which
regenerate? one is the EXCEPTION?
A. Bone A. Retrovirus
B. Liver B. Herpesvirus
C. Striated muscle C. Picornavirus
D. Collagen D. Hepatitis B virus
E. Smooth muscle E. Human papillomavirus

189. Healing is retarded by each of the following 195. Which of the following terms refers to the presence
conditions EXCEPT one. Which one is the of digested blood in the stool?
EXCEPTION?
A. Hemosiderosis
A. Trauma B. Hemochromatosis
B. Immobilization C. Hematoma
C. Infection D. lcterus
D. Hemorrhage E. Melena
E. Ischemia

196. Chlamydia trachoniatis might cause each of the


190. Escape of plasma from capillaries into an area of following diseases EXCEPT one. Which one is the
inflammation is favored by an increase in which of EXCEPTION?
the following?
A. Trachoma
A Hyaluronidase B. Lymphogranuloma venereuni
B Leukocyte migration C. Inclusion conjunctivitis
C. Vascular permeability D. Non-gonococcal urethritis
D. Osmotic pressure of plasma E. Primary atypical pneumonia
E. Number of endothelial junctions

197. Which of the following conditions is most


191. Each of the following conditions predisposes a commonly associated with acute pancreatit~s?
patient to develop cancer EXCEPT one. Which one
is thc EXCEPTION? A. Chronic alcohol abuse
B. Diabetes ~nellitus
A. Asbestosis C. Physical trauma
0. Anthracosis D. Viral infection
C. Hepatitis C E. Hypercalcernia
D. Gardner's syndrorne
E. ulcerative colitis
198 Each of ttie following is attributable to hepatic
failure EXCEPT one. Which one is the
EXCEPTION?

A TI-emor
B Gynecomastia
C. Mallory bodies
D. Hypoalburninemia
E. Spider telangectasra

199. Red hepatization refers to which of the following?

A. Early stage of acute hepatitis


B. Late stage of acute hepatitis
C. Cirrhosis caused by hemochromatosis
D Congestion of the liver caused by chronic
right sided heart failure
E. Stage of lobar pneumonia

200 Acute passive congestion often accompanies


which of the following?

A. Edema
B. Inflammation
C. Contusion
D. Hematoma
NATIONAL BOARD PART I RELEASED ITEM KEY
ITEMS 101-200
NATIONAL BOARD PART I RELEASED ITEM KEY
ITEMS 1-100

The above key may be used to determine correct and incorrect answers; however, for this
itern set, it is not possible to convert a raw score number correct to a standard score or-
associated passlfail status.

You might also like